Download as pdf or txt
Download as pdf or txt
You are on page 1of 132

The Official SAT Online Course

Help | Profile | My Organizer | My Bookmarks | Logout

1/1

Answers and Explanations

Test Sections

Back to Score Report

Section 1

Essay
Online - Practice Test #3

Section 2

Section 3

Section 4

Section 5

Section 7

These sample essays were originally handwritten by students but are shown typed here for ease of reading. The essays are displayed exactly as students wrote them, without any corrections to spelling, punctuation, or syntax. One handwritten sample essay is provided to illustrate the need for legible and clear handwriting.

Section 8

Section 9

Exemplars:

Section 10

Essay Prompt

Think carefully about the issue presented in the following excerpt and the assignment below.

eg

is
Back to Score Report

te

Do people learn who they are only when they are forced into action? Plan and write an essay in which you develop your point of view on this issue. Support your position with reasoning and examples taken from your reading, studies, experience, or observations.

re

The people we call heroes do not usually start out as unusual. Often they are ordinary people subject to ordinary human weaknessesfear, doubt, and self-interest. In fact, they live ordinary lives until they distinguish themselves by having to deal with an injustice or a difficult situation. Only then, when they must respond in thought and in action to an extraordinary challenge, do people begin to know their strengths and weaknesses.

Copyright 2006 The College Board. All rights reserved.

Privacy Policy

Terms of Use

Contact Us

nR

file://E:\\c2.htm

2006-11-12

The Official SAT Online Course

1/22

Help | Profile | My Organizer | My Bookmarks | Logout

Answers and Explanations

Test Sections

Back to Score Report

Section 1

View Answers and Explanations


Online - Practice Test #3

Section 2

Section 3

Section 4

Section 5

Section 7

Responding to criticism that the script was rambling and -------, the new screenwriter revised the dialogue for greater succinctness and -------.
(A)

Section 8

engaging. . simplicity

Section 9

(B)

subjective. . ambiguity

Section 10

(C) muddled. . clarity

(D) terse. . emptiness

(E)

difficult. . abstraction

nR

Explanation for Incorrect Answer A : Choice (A) is incorrect. "Engaging" means interesting. "Simplicity" means uncomplicated. If one were to insert these terms into the text, the sentence would read "Responding to criticism that the script was rambling and engaging, the new screenwriter revised the dialogue for greater succinctness and simplicity." The missing terms describe a second negative quality of the script and a corresponding positive quality of the revised version. A screenplay would not be criticized for being engaging.

Explanation for Incorrect Answer B : Choice (B) is incorrect. "Subjective" means dependent on personal taste or views. "Ambiguity" means uncertainty. If one were to insert these terms into the text, the sentence would read "Responding to criticism that the script was rambling and subjective, the new screenwriter revised the dialogue for greater succinctness and ambiguity." The missing terms describe a second negative quality of the script and a corresponding positive quality of the revised version. If a screenplay were criticized for being subjective, the screenwriter would probably revise it to make it less ambiguious.

Explanation for Incorrect Answer D : Choice (D) is incorrect. "Terse" means brief or succinct. "Emptiness" means, in this context, meaninglessness. If one were to insert these terms into the text, the sentence would read "Responding to criticism that the script was rambling and terse, the new screenwriter revised the dialogue for greater succinctness and emptiness." The missing terms describe a second negative quality of the script and a corresponding positive quality of the revised version. A screenplay cannot be both rambling and terse, and one would not revise it to give it greater emptiness.

Explanation for Incorrect Answer E : Choice (E) is incorrect. "Difficult" means hard. "Abstraction" means theoretical. If one were to insert these terms into the text, the sentence would read "Responding to criticism that the script was rambling and difficult, the new screenwriter revised the dialogue for greater succinctness and abstraction." The missing terms describe a second negative quality of the script and a corresponding positive quality of the revised version. If a screenplay is difficult, then adding more abstraction would only make it more difficult.

eg i

st e

Explanation for Correct Answer C : Choice (C) is correct. "Muddled" means confused or aimless. "Clarity" means clearness. If one were to insert these terms into the text, the sentence would read "Responding to criticism that the script was rambling and muddled, the new screenwriter revised the dialogue for greater succinctness and clarity." The missing terms describe a second negative quality of the script and a corresponding positive quality of the revised version. A muddled script would need to be revised for clarity.

re

d
2006-11-12

ANSWERS

AND EXPLANATIONS

file://E:\\c3.htm

The Official SAT Online Course

2/22

During the 1990s, Shanghai benefited from an architectural -------, the result of a dramatic increase in innovative and artistic building.
(A)

intransigence

(B)

plentitude

(C) desecration

(D) stagnation

(E)

renaissance

ANSWERS

AND EXPLANATIONS

Explanation for Correct Answer E : Choice (E) is correct. "Renaissance" means a rebirth or revival. If one were to insert this term into the text, the sentence would read "During the 1990s, Shanghai benefited from an architectural renaissance, the result of a dramatic increase in innovative and artistic building." An "architectural renaissance" describes a period of renewed excitement in the field of architecture, and properly describes the result of a dramatic increase in new artistic construction.

Explanation for Incorrect Answer C : Choice (C) is incorrect. A "desecration" means an outrageous mistreatment of somethingthe vandalizing of a sacred building, for instance. If one were to insert this term into the text, the sentence would read "During the 1990s, Shanghai benefited from an architectural desecration, the result of a dramatic increase in innovative and artistic building." Desecration would not benefit a city.

nR
(A)

Explanation for Incorrect Answer D : Choice (D) is incorrect. "Stagnation" means a lack of movement or development. If one were to insert this term into the text, the sentence would read "During the 1990s, Shanghai benefited from an architectural stagnation, the result of a dramatic increase in innovative and artistic building." Stagnation is the exact opposite of a dramatic increase in innovation.

Many subatomic nuclear particles are ------- and nearly -------: they are hard to track as well as to detect.

unstable . . explosive

(B)

elusive . . imperceptible

(C) minute . . immobile

(D) charged . . reactive

(E)

tenuous . . indivisible

ANSWERS

Explanation for Correct Answer B : Choice (B) is correct. "Elusive" means hard to find. "Imperceptible" means difficult

eg

Explanation for Incorrect Answer B : Choice (B) is incorrect. A "plenitude" means an abundance. If one were to insert this term into the text, the sentence would read "During the 1990s, Shanghai benefited from an architectural plenitude, the result of a dramatic increase in innovative and artistic building." An abundance of architecture would be the result of an increase in construction. But this would not account for the "innovative and artistic" quality of the architecture.

AND EXPLANATIONS

is te r

Explanation for Incorrect Answer A : Choice (A) is incorrect. "Intransigence" means being uncompromising or stubborn. If one were to insert this term into the text, the sentence would read "During the 1990s, Shanghai benefited from an architectural intransigence, the result of a dramatic increase in innovative and artistic building." It is unlikely that "intransigence" would be the result of an increase in architectural innovation; innovation is in some ways the opposite of stubbornness.

ed

file://E:\\c3.htm

2006-11-12

The Official SAT Online Course

3/22

to perceive or detect. If one were to insert these terms into the text, the sentence would read "Many subatomic nuclear particles are elusive and nearly imperceptible: they are hard to track as well as to detect." The statement following the colon explains or expands on the statement preceding it. The first missing term indicates that the particles are hard to track: this perfectly defines "elusive." The second missing term indicates that they are hard to detect: this perfectly defines "imperceptible."

Explanation for Incorrect Answer A : Choice (A) is incorrect. "Unstable" means likely to change suddenly. "Explosive" means likely to blow up. If one were to insert these terms into the text, the sentence would read "Many subatomic nuclear particles are unstable and nearly explosive: they are hard to track as well as to detect." The statement following the colon explains or expands on the statement preceding it. The first missing term indicates that the particles are hard to track: this does not define "unstable."

Explanation for Incorrect Answer C : Choice (C) is incorrect. "Minute" means extremely small. "Immobile" means not moving. If one were to insert these terms into the text, the sentence would read "Many subatomic nuclear particles are minute and nearly immobile: they are hard to track as well as to detect." The statement following the colon explains or expands on the statement preceding it. The first missing term indicates that the particles are hard to track: this does not precisely define minute. The second missing term indicates that they are hard to detect: this does not define immobile. Anything that does not move should be easy, not difficult, to detect.

nR
(A)

Explanation for Incorrect Answer E : Choice (E) is incorrect. "Tenuous" means flimsy. "Indivisible" means not capable of being broken down into smaller parts. If one were to insert these terms into the text, the sentence would read "Many subatomic nuclear particles are tenuous and nearly indivisible: they are hard to track as well as to detect." The statement following the colon explains or expands on the statement preceding it. The first missing term indicates that the particles are hard to track: this does not define "tenuous." The second missing term indicates that they are hard to detect: this does not define "indivisible."

The crafty child tricked his innocent brother, a particularly ------- and trusting boy, into committing a mischievous prank.

guileless

(B)

intrusive

(C) astute

(D) opportunistic

(E)

circumspect

ANSWERS

Explanation for Correct Answer A : Choice (A) is correct. "Guileless" means innocent and naive. If one were to insert this term into the text, the sentence would read "The crafty child tricked his innocent brother, a particularly guileless and trusting boy, into committing a mischievous prank." The missing term characterizes a brother who is innocent, trusting, and able to be tricked. "Guileless" suggests such a person.

Explanation for Incorrect Answer B :

eg i

AND EXPLANATIONS

st

Explanation for Incorrect Answer D : Choice (D) is incorrect. "Charged" means full of energy. "Reactive" means readily responsive to a stimulus. If one were to insert these terms into the text, the sentence would read "Many subatomic nuclear particles are charged and nearly reactive: they are hard to track as well as to detect." The statement following the colon explains or expands on the statement preceding it. The first missing term indicates that the particles are hard to track: this does not define charged. The second missing term indicates that they are hard to detect: this does not define reactive.

er e

file://E:\\c3.htm

2006-11-12

The Official SAT Online Course

4/22

Choice (B) is incorrect. "Intrusive" means invasive or nosy. If one were to insert this term into the text, the sentence would read "The crafty child tricked his innocent brother, a particularly intrusive and trusting boy, into committing a mischievous prank." The missing term characterizes a brother who is innocent, trusting, and able to be tricked. A brother who is intrusive may or may not be such a person.

Explanation for Incorrect Answer C : Choice (C) is incorrect. "Astute" means knowledgeable and wise. If one were to insert this term into the text, the sentence would read "The crafty child tricked his innocent brother, a particularly astute and trusting boy, into committing a mischievous prank." The missing term characterizes a brother who is innocent, trusting, and able to be tricked. An astute person would not likely be tricked.

Explanation for Incorrect Answer D : Choice (D) is incorrect. "Opportunistic" means taking advantage of opportunities when they arise. If one were to insert this term into the text, the sentence would read "The crafty child tricked his innocent brother, a particularly opportunistic and trusting boy, into committing a mischievous prank." The missing term characterizes a brother who is innocent, trusting, and able to be tricked. Someone who is opportunistic may or may not be such a person.

Explanation for Incorrect Answer E : Choice (E) is incorrect. "Circumspect" means possessing caution or prudence. If one were to insert this term into the text, the sentence would read "The crafty child tricked his innocent brother, a particularly circumspect and trusting boy, into committing a mischievous prank." The missing term characterizes a brother who is innocent, trusting, and able to be tricked. It would be hard to trick a circumspect person.

(A)

compromise

(B)

humility

(C) machinations

(E)

deftness

nR
ANSWERS

Explanation for Correct Answer E : Choice (E) is correct. "Deftness" means skill or dexterity. If one were to insert this term into the text, the sentence would read "Ellen Ochoa's deftness with the apparatus in the space shuttle Discovery was apparent when she adroitly manipulated the shuttle's robot arm." The missing term must have a meaning close to that of "adroit," which means skillful or dexterous. Deft and adroit are synonymous.

Explanation for Incorrect Answer A : Choice (A) is incorrect. "Compromise" is a settlement of mutual concessions. If one were to insert this term into the text, the sentence would read "Ellen Ochoa's compromise with the apparatus in the space shuttle Discovery was apparent when she adroitly manipulated the shuttle's robot arm." The missing term must have a meaning close to that of "adroit," which means skillful or dexterous. A compromise in this context has nothing to do with an adroit action.

Explanation for Incorrect Answer B : Choice (B) is incorrect. "Humility" is a modest attitude. If one were to insert this term into the text, the sentence would read "Ellen Ochoa's humility with the apparatus in the space shuttle Discovery was apparent when she adroitly manipulated the shuttle's robot arm." The missing term must have a meaning close to that of "adroit," which means skillful or dexterous. Humility has nothing to do with performing an adroit action.

Explanation for Incorrect Answer C : Choice (C) is incorrect. "Machinations" are crafty schemes. If one were to insert this term into the text, the sentence would read "Ellen Ochoa's machinations with the

eg
AND EXPLANATIONS

(D) synergy

is te

Ellen Ochoas ------- with the apparatus in the space shuttle Discovery was apparent when she adroitly manipulated the shuttles robot arm.

re

d
file://E:\\c3.htm

2006-11-12

The Official SAT Online Course


apparatus in the space shuttle Discovery was apparent when she adroitly manipulated the shuttle's robot arm." The missing term must have a meaning close to that of "adroit," which means skillful or dexterous. Machinations have nothing to do with an adroit action.

5/22

Explanation for Incorrect Answer D : Choice (D) is incorrect. "Synergy" describes an action or operation that is greater than the sum of its parts. If one were to insert this term into the text, the sentence would read "Ellen Ochoa's synergy with the apparatus in the space shuttle Discovery was apparent when she adroitly manipulated the shuttle's robot arm." The missing term must have a meaning close to that of "adroit," which means skillful or dexterous. Synergy has nothing to do with an adroit action.

Passage 1

When I entered journalism school in the 1920s, I found out that perennial and fundamental laws governing the art of good writing had been discovered. Experts had stubbornly and rigorously analyzed Line readers modest capacity to dedicate their attention to the 5 printed page and had established once and for all, apparently with the mathematical precision of astronomers, the order of readers natural preferences. They found that effective prose was composed of a limited number of very simple and common words grouped in short, crisp 10 sentences. When designed rigorously, such prose could penetrate the opaque barrier of millions of readers indifference, apathy, inattention, and obtuseness.

Passage 2

Beginning writers are often taught that effective prose is crisp and concise and that most readers have 15 no patience with densely complex sentences and obscure vocabulary. While clarity and succinctness are certainly worthy goals, I sometimes worry that our assumption that the reading public can comprehend only such writing might be selling

nR

eg

is

te

re

file://E:\\c3.htm

2006-11-12

The Official SAT Online Course

6/22

20

them short. Assuming that readers are merely able to digest simple words, and that they have no interest in puzzling through more challenging prose, turns that theory into a self-fulfilling prophecy. Its our responsibility as writers to offer the public something beyond workmanlike writing: if we dont, readers will never appreciate 25 writing as an art rather than as a mechanical craft.
6

Both passages address which of the following topics?


(A)

Why people choose writing as a career

(B)

The kind of writing that readers can understand

(C) How readers convey their preferences to writers

(D) The difference between beginning writers and experts

(E)

Why long sentences are easier to read than short ones

Explanation for Correct Answer B : Choice (B) is correct. Each passage specifically addresses readers' abilities to understand writing. Passage 1 addresses journalistic practices of writing short, simple sentences designed to be understood by a large audience. Passage 2 challenges the idea that readers can only handle short, simple sentences.

nR
(A)

Explanation for Incorrect Answer A : Choice (A) is incorrect. Although both passages are about people who write, neither addresses why people choose to write as a career.

Explanation for Incorrect Answer C : Choice (C) is incorrect. Although admitting readers may prefer a certain style of writing, neither passage says anything about how writers know what readers prefer or that readers convey, or communicate, their preferences to writers.

Explanation for Incorrect Answer D : Choice (D) is incorrect. Neither passage discusses different levels of writing experience.

Explanation for Incorrect Answer E : Choice (E) is incorrect. Both passages acknowledge the common belief that short sentences are easier to understand than long ones.

Which statement in Passage 2 most directly contradicts the assertion in Passage 1, lines 8-10 (They found . . . sentences) ?

Beginning writers . . . concise (lines 14-15)

(B)

most readers . . . vocabulary (lines 15-16)

(C) clarity and . . . goals (line 17)

(D) they have . . . prose (lines 21-22)

(E)

Its our . . . writing (lines 23-24)

ANSWERS

eg is te r
AND EXPLANATIONS

ANSWERS

AND EXPLANATIONS

ed

file://E:\\c3.htm

2006-11-12

The Official SAT Online Course

7/22

Explanation for Correct Answer E : Choice (E) is correct. By stating that it is writers' responsibility to offer "challenging prose," Passage 2 contradicts the assertion in Passage 1 that "effective prose" must be very simple.

Explanation for Incorrect Answer A : Choice (A) is incorrect. Both statements say the same thing, that effective prose is simple and straightforward.

Explanation for Incorrect Answer B : Choice (B) is incorrect. The statement in Passage 2 that most readers "have no patience" with complex writing supports Passage 1's description of effective prose; in fact, Passage 1 uses this same argument.

Explanation for Incorrect Answer C : Choice (C) is incorrect. Both passages agree that, as Passage 2 concedes, "clarity and succinctness," or conciseness, are "worthy goals."

Explanation for Incorrect Answer D : Choice (D) is incorrect. The statement in Passage 2 that people have "no interest" in "challenging prose" supports the argument for simple prose presented in Passage 1.

The author of Passage 2 would most likely respond to the view of readers expressed in the last sentence of Passage 1 (lines 11-13) with

(B)

perplexity

(C) disdain

(D) humor

(E)

appreciation

nR eg
ANSWERS

AND EXPLANATIONS

Explanation for Correct Answer A : Choice (A) is correct. The author of Passage 2 argues that readers would like challenging writing if only it were given to them. Consequently, the author of Passage 2 would likely show concern, or unease, with Passage 1's scornful description of readers. The author states that "I sometimes worry" that a negative view of readers is being taught to young writers.

Explanation for Incorrect Answer B : Choice (B) is incorrect. The author of Passage 2 would not likely be perplexed, or confused, by the views expressed at the end of Passage 1, but would simply disagree with them.

Explanation for Incorrect Answer C : Choice (C) is incorrect. Disdain, or scorn, would be inconsistent with the reasonable and concerned tone of Passage 2.

Explanation for Incorrect Answer D : Choice (D) is incorrect. Passage 2 does not use humor in framing an argument that challenges the commonly held negative view of readers; rather, the author is concerned about the negative view of readers that is affecting the way young writers compose their work.

Explanation for Incorrect Answer E : Choice (E) is incorrect. The author of Passage 2 would likely disagree with the opinions in Passage 1, rather than showing appreciation.

is te r
2006-11-12

(A)

concern

file://E:\\c3.htm

ed

The Official SAT Online Course


In comparison to Passage 2, the tone of Passage 1 is more
(A)

8/22

earnest

(B)

inspirational

(C) complacent

(D) defensive

(E)

sarcastic

ANSWERS

AND EXPLANATIONS

Explanation for Correct Answer E : Choice (E) is correct. Passage 1 mocks the "experts" and also its own subject (writing) by using long sentences and uncommon words to explain that good writing should do the opposite. The language is especially sarcastic when the author portrays the "experts'" treatment of writing as a science that has "perennial and fundamental laws" that can be calculated "apparently with the mathematical precision of astronomers." The tone of Passage 2 is concerned, not sarcastic. Passage 2 speaks of the responsibility of writers to raise the reading level of their audience.

The following passage is from a 1994 collection of essays about animals, written by a poet, philosopher, and animal trainer.

The question that comes first to my mind is this: What would it mean to say that an animal has the right to the pursuit of happiness? How would that come about, and in Line relationship to whom? In speaking of animal happiness, we 5 often tend to mean something like creature comforts. The emblems of this are the golden retriever rolling in the grass, the horse with his nose deep in the oats, kitty by the fire. Creature

nR

Explanation for Incorrect Answer D : Choice (D) is incorrect. Neither passage is defensive in tone; Passage 1 is mocking in tone, while Passage 2 makes a reasoned argument.

eg

Explanation for Incorrect Answer C : Choice (C) is incorrect. The tone of ridicule in Passage 1 is far from complacent, or easy-going. Neither of the passages are complacent, or untroubled; Passage 1 is sarcastic, or cutting, and Passage 2 is more earnest and inspirational.

is t

Explanation for Incorrect Answer B : Choice (B) is incorrect. Passage 1 presents negative opinions about readers, which could hardly be called inspirational. Passage 2, on the other hand, does attempt to inspire young writers to believe that readers are better than they think and that they can read more complex prose.

er

ed

Explanation for Incorrect Answer A : Choice (A) is incorrect. The tone of Passage 1 is sarcastic and cutting when it describes the certainty of "experts" on the essence of good writing, which is the opposite of being earnest, or sincere. Passage 2 clearly expresses earnest, or heartfelt, feelings about the practice of writing and the capabilities of readers.

file://E:\\c3.htm

2006-11-12

The Official SAT Online Course

9/22

accomplished dressage horse). This happiness, like the artists, must come from something within the animal, something trainers call talent, and so cannot be imposed on the animal. But at the same time it does not arise in a vacuum; if it had not been a fairly ordinary 25 thing in one part of the world at one point to teach young children to play the harpsichord, it is doubtful that Mozarts music would exist. There are animal versions, if not equivalents, of Mozart, and they cannot make their spontaneous passions into sustained happiness without 30 education, any more than Mozart could have. Aristotle identified happiness with ethics and with work, unlike Thomas Jefferson, who defined happiness as Indolence of Body; Tranquility of Mind, and thus what I

nR

eg

is

te
*

comforts are important to animals: Grub first, then ethics is a motto that would describe many a wise 10 Labrador retriever, and I have a bull terrier named Annie whose continual quest for the perfect pillow inspires her to awesome feats. But there is something more to animals, something more to my Annie, a capacity for satisfactions that come from work in the full sense 15 something approximately like what leads some people to insist that they need a career (though my own temperament is such that I think of a good woodcarver or a dancer or a poet sooner than I think of a business executive when I contemplate the kind of happiness enjoyed by 20 an

re

file://E:\\c3.htm

2006-11-12

The Official SAT Online Course

10/22

35

* Dressage is a complex series of movements signaled to a horse by its rider.


The author presents examples in lines 7-8 in order to

10

(B)

suggest that appearances of happiness are deceptive

(C) evoke images of contentment

nR
(E)

(D) support an apparently implausible argument

arouse nostalgic longings

ANSWERS

Explanation for Correct Answer C : Choice (C) is correct. The examples given in the lines "the golden retriever rolling the grass, the horse with his nose deep in the oats, and kitty by the fire" are all presented as examples of "creature comforts," images that evoke, or bring to mind, notions of happiness or contentment.

Explanation for Incorrect Answer A : Choice (A) is incorrect. Although the examples in lines 7-8 do show animals engaged in a variety of activities, this is not their primary purpose. Their primary purpose is to offer examples of animal activities that humans believe indicate that animals are content.

Explanation for Incorrect Answer B : Choice (B) is incorrect. Although the passage argues that animal happiness differs from human happiness, it does not suggest that the examples presented in lines 78 are deceptive, or misleading.

Explanation for Incorrect Answer D : Choice (D) is incorrect. The passage does not suggest that anyone would be likely to find its arguments "implausible," or hard to believe.

Explanation for Incorrect Answer E :

eg

(A)

illustrate the variety of activities in which animals engage

AND EXPLANATIONS

is t

er e

call creature comforts. Aristotle also excluded as unethical anything that animals and artists do, for reasons that look wholly benighted to me. Nonetheless, his central insights are more helpful than anything else I know in beginning to understand why some horses and dogs can only be described as competent, good at what they do, 40 and therefore happy. Not happy because leading lives of pleasure, but rather happy because leading lives in which the sensation of getting it right, the click, as of the pleasure that comes from solving a puzzle or surmounting something, 45 is a governing principle.

file://E:\\c3.htm

2006-11-12

The Official SAT Online Course

11/22

Choice (E) is incorrect. The author does not mention nostalgic longings, even though the activities described could certainly evoke those feelings in readers.

11

The motto in line 9 indicates that animals


(A)

are much more intelligent than many people believe

(B)

have been forced to develop keen survival skills

(C) desire consistency in their daily lives

(D) enjoy close relationships with human beings

(E)

are concerned primarily with immediate physical gratification

ANSWERS

AND EXPLANATIONS

Explanation for Correct Answer E : Choice (E) is correct. In the context of the passage, the motto, "Grub first, then ethics," means that it is important to satisfy physical needs before one addresses ethical or moral dilemmas. For animals, this means that they tend to be concerned about physical needs first.

Explanation for Incorrect Answer C : Choice (C) is incorrect. Although animals may desire a consistent routine, the motto is about food, not consistency.

12

nR
(A)

Explanation for Incorrect Answer D : Choice (D) is incorrect. The passage does not address the topic of animals' relationships with people.

Which of the following statements is most consistent with the authors discussion of temperament in lines 17-21?

The author believes a poet can be successful in business.

(B)

The author considers artistic pursuits to be the most personally fulfilling of all endeavors.

(C) The author suspects that a busy life can have its own rewards.

(D) The author believes that few people are ever satisfied with the jobs they have chosen.

(E)

The author considers subjectivity and self-knowledge to be critical to human gratification.

ANSWERS

Explanation for Correct Answer B : Choice (B) is correct. "Temperament" means disposition or character. The author feels that the careers "of a good woodcarver or a dancer or a poet" are more consistent with his own temperament than a career in business. This suggests that the author believes artistic careers to be the most "personally fulfilling of all endeavors."

eg i

Explanation for Incorrect Answer B : Choice (B) is incorrect. Although ensuring that one has enough food could be called a "survival skill," the use of the motto, "Grub first, then ethics," in this passage does not suggest that these skills have been forced on animals. Furthermore, the passage never discusses survival skills.

AND EXPLANATIONS

st

er

Explanation for Incorrect Answer A : Choice (A) is incorrect. Although the passage says that the motto, "Grub first, then ethics," "would describe many a wise Labrador retriever" (lines 10-11), there is no suggestion that such an animal would be any wiser than would be expected. The motto simply suggests that animals are concerned with their physical needs before all others.

ed

file://E:\\c3.htm

2006-11-12

The Official SAT Online Course

12/22

Explanation for Incorrect Answer A : Choice (A) is incorrect. Although the author does mention a poet as one of the examples in lines 17-21, the issue of whether or not a poet could have a successful business career is not addressed.

Explanation for Incorrect Answer C : Choice (C) is incorrect. In lines 17-21, the author does not discuss the issue of whether or not "a busy life can have its own rewards." Instead, the author seems concerned with the satisfaction that can be gained through work that is personally fulfilling (such as the work done by a "good woodcarver or a dancer or a poet").

Explanation for Incorrect Answer D : Choice (D) is incorrect. Although the author may hold the view that "few people are ever satisfied with the jobs they have chosen," this view is not addressed in the passage. On the contrary, the passage suggests that work can be an extremely rewarding pursuit.

Explanation for Incorrect Answer E : Choice (E) is incorrect. The author probably believes this to be true, but it is not relevant to the author's discussion of temperament in lines 17-21.

13

The authors discussion of Mozart in lines 25-28 primarily emphasizes the


(A)

role of social circumstances in the emergence of a musical genius

(B)

fact that young children are sometimes pushed to excel

(D) belief that the harpsichord was the ideal musical instrument for Mozarts early talent

Explanation for Correct Answer A : Choice (A) is correct. The statement points out that a social circumstance, namely musical education, enabled Mozart's genius to find expression. The passage also suggests that without this musical education Mozart's natural talent may not have found expression.

14

nR
(A)

Explanation for Incorrect Answer B : Choice (B) is incorrect. Although teaching young children to play the harpsichord might be described as pushing them to excel, the primary focus of the discussion in lines 25-28 is on the role of musical education in the development of musical genius.

Explanation for Incorrect Answer C : Choice (C) is incorrect. The author makes no comment about the relative incidence, or numbers of occurrences, of genius in the past or in the present.

Explanation for Incorrect Answer D : Choice (D) is incorrect. Even though the harpsichord may have been the ideal musical instrument for Mozarts early talent, the emphasis in the passage is not on which instrument he played, but on his having been given a musical education.

Explanation for Incorrect Answer E : Choice (E) is incorrect. Although the author discusses the pleasure that artists derive from achievement earlier in the passage, in this section the emphasis is on the role of musical education in the development of Mozart's talent.

In line 30, passions most nearly means

love affairs

(B)

violent outbursts

eg is

ANSWERS

AND EXPLANATIONS

te

(E)

pleasure that artists derive from achievement

re

(C) observation that genius was more common in the past than it is today

file://E:\\c3.htm

2006-11-12

The Official SAT Online Course

13/22

(C) enthusiasms

(D) prejudices

(E)

sufferings

ANSWERS

AND EXPLANATIONS

Explanation for Correct Answer C : Choice (C) is correct. In the passage, the author describes "animal versions, if not equivalents, of Mozart," arguing that "they cannot make their spontaneous passions into sustained happiness without education, any more than Mozart could have." The author uses the word "passions" in the sense of impulses, talents, or enthusiasms that require training and education if they are to lead to "happiness."

Explanation for Incorrect Answer A : Choice (A) is incorrect. "Passions" does not refer to love or ardent affections in this context, but to strong impulses, or enthusiasms, such as Mozart's feeling for music.

15

Which situation most accurately illustrates the authors definition of a happy animal?

nR
(B)

(A)

A bird finding its one lifetime mate

A dog herding sheep into a pen

(C) A horse being carefully groomed for a show

(D) A monkey escaping from a city zoo

(E)

A cat caring devotedly for her kittens

ANSWERS

Explanation for Correct Answer B : Choice (B) is correct. The passage states, "there is something more" to animals' happiness than mere comfort, namely, the "satisfactions that come from work." A dog herding sheep into a pen is performing the kind of work that the author describes as satisfying to animals.

Explanation for Incorrect Answer A : Choice (A) is incorrect. Although finding a mate could be thought of as contributing to happiness, the kind of happiness the author is discussing involves work.

Explanation for Incorrect Answer C : Choice (C) is incorrect. "A horse being carefully groomed for a show" is a creature receiving attention, not one that is working or performing a challenging task.

Explanation for Incorrect Answer D : Choice (D) is incorrect. Although escaping from a zoo might be pleasurable as well as challenging for a monkey, it is not the kind of "work in the full sense" (line 15) that would give the animal pleasure for a job well done.

eg

AND EXPLANATIONS

is

Explanation for Incorrect Answer E : Choice (E) is incorrect. The context does not mention suffering nor does the passage have anything to do with suffering.

te

re

Explanation for Incorrect Answer D : Choice (D) is incorrect. "Passions" does not refer to prejudices or strong opinions in this context; rather, it refers to positive impulses, such as talents, that have the possibility of being shaped into masterpieces like Mozart's music.

Explanation for Incorrect Answer B : Choice (B) is incorrect. In this context, "passions" refer to a talent, or enthusiasm, that could be cultivated into happiness through education. Consequently, it would be strange to say that animals "cannot make" their violent outbursts "into sustained happiness without education."

file://E:\\c3.htm

2006-11-12

The Official SAT Online Course


Explanation for Incorrect Answer E : Choice (E) is incorrect. Caring for kittens is an action of devotion, but it is not work or a career in the sense discussed in the passage.

14/22

This passage was written in 1996 after the discovery of a meteorite that appeared to contain fossil evidence of microscopic life on Mars.

The rock that sprang to Martian life late last summer did not shock me by offering up apparent fossils of an extinct alien form of life. I had long believed that the universe teems with life elsewhere, and that our failure to find Line it simply results from a lack of exploration. What did 5 amaze me about the potato-size rock that fell from Mars was that it had traveled millions of miles across space to land here, blasted from world to world by a planetary collision of the sort that purportedly killed off our dinosaurs, and had lain waiting for millennia upon an Antarctic ice field, until an 10 observant young woman traveling in an expedition party picked it up, because she figured that it had come from another world. How could she know such a thing? The composition of ALH 84001, as the much scrutinized rock is designated, closely matches the 15 makeup of Martian matter that was analyzed on site in 1976 by miniature chemistry laboratories aboard two Viking Mars landers. As a result of this positive identification, no astronomer seriously doubts the meteorite s Martian origin. Researchers think they have pinpointed 20 its former resting place to just two possible sitesa region called Sinus Sabaeus, fourteen degrees south of the Martian equator, or a crater east of the Hesperia Planitia region.

nR

eg is

te

re

file://E:\\c3.htm

2006-11-12

The Official SAT Online Course

15/22

25

30

40

45

nR

eg

is te

35

The bold precision of this assessment is for me the most stunning surprise dealt by the rock from Mars even more mind-boggling than the suggestive traces of something that might once have lived and died in its microscopic fissures. I cannot resist comparing this new intimacy with our solar system to the shoebox diorama of the planets I designed for my grade-school science fair. I used marbles, jack balls, and Ping-Pong balls, all hanging on strings and painted different colors, all inside a box representing our solar system. This crude assortment of materials allowed a reasonable representation of what was known 40 years ago about the nine planets: Mars was red and had two moons; Jupiter dwarfed the other planets (I should have used a basketball but it wouldnt fit in the box); Saturn had rings. If my school-age daughter were to attempt such a construction today, shed need handfuls of jelly beans and gum balls to model the newly discovered satellites of the giant planets. Shed want rings around Jupiter, Uranus, Neptune, too, not to mention a moon for Pluto. Similarly, our solar system, once considered unique, now stands as merely the first known example of a planetary system in our galaxy. Since October of 1995, astronomers at ground-based observatories in Europe and the United States have announced that theyve found evidence of at least seven alien planets orbiting other stars. As yet, not one of these large planetssome of which are many times the mass of Jupiterhas

re

file://E:\\c3.htm

2006-11-12

The Official SAT Online Course

16/22

50 actually been

seen through a telescope; we know about them indirectly through the gravitational effects they exert on their parent stars. Yet, even though we have no picture of what they look like, enough information has been deduced about their atmospheric conditions to grant the 55 nickname Goldilocks to a planet attending the star 70 Virginis, an appellation suggesting that the cloud-top temperature is just right, as the storybook Goldilocks would say, for the presence of liquid water. Liquid water, not known to exist anywhere in our solar system now except 60 on Earth, is thought crucial to biological life; thus, only a short leap of faith is needed to carry hopeful scientists from the presence of water to the existence of extraterrestrial life. To raise the specter of the Mars rock once again, the primitive life-forms that pressed their 65 memory inside it likewise suggest an era when dry-as-dust Mars was a wet world, where rivers flowed.

16

nR eg
(A)

In lines 5-12, the author suggests that the expeditionists discovery of the meteorite was surprising primarily because it

defied scientists doubts that such an object could reach Earth

(B)

occurred after her party had given up any hope of success

(C) resulted from a seemingly unlikely sequence of events

(D) provided evidence to contradict a long-standing scientific theory

(E)

led to an unprecedented degree of scrutiny

ANSWERS

AND EXPLANATIONS

Explanation for Correct Answer C : Choice (C) is correct. The author writes, "[w]hat did amaze me about the potatosize rock that fell from Mars was that it had traveled millions of miles to land here" (lines 5-7). Further, the author describes the rock's journey as one in which it was "blasted from world to world," and had "lain waiting for milennia upon an Antarctic ice field" (lines 7-10). The author is clearly amazed at the "seemingly unlikely sequence of events" that led to the rock's discovery.

is

te

re

file://E:\\c3.htm

2006-11-12

The Official SAT Online Course

17/22

Explanation for Incorrect Answer A : Choice (A) is incorrect. The passage does not mention whether the scientists had "doubts that such an object could reach earth." The author expresses her awe or wonder over the process by which the meteorite came to earth and was found, but this has little to do with the beliefs of scientists.

Explanation for Incorrect Answer B : Choice (B) is incorrect. The passage does not mention the expedition party's expectations or attitudes prior to finding the rock.

Explanation for Incorrect Answer D : Choice (D) is incorrect. Far from contradicting a "long-standing scientific theory," the "apparent fossils of an extinct alien form of life" (lines 2-3) in the rock tend to confirm the author's belief that life may exist on other planets.

Explanation for Incorrect Answer E : Choice (E) is incorrect. It is not surprising that such a rare find would be studied so closely.

17

In line 15, designated most nearly means


(A)

drawn

(B)

called

(C) stipulated

ANSWERS

AND EXPLANATIONS

18

nR
(A)

Explanation for Incorrect Answer A : Choice (A) is incorrect. The term ALH 84001 is not a picture, drawing, or means of depicting an object. ALH 84001 is a name.

Explanation for Incorrect Answer C : Choice (C) is incorrect. "Stipulated" means required or demanded. There is no mention that the naming of the rock ALH 84001 was stipulated, required, or demanded.

Explanation for Incorrect Answer D : Choice (D) is incorrect. "Selected" means chosen, and its use in this context would imply that there were other rocks to choose from. The author makes the unique nature of this rock clear.

Explanation for Incorrect Answer E : Choice (E) is incorrect. "Allocated" means gave or assigned. Line 15 does not refer to any allocation or assignment; rather, it refers to the name given to the rock.

The author considers the researchers conclusion bold (line 24) primarily because it

daringly offers two conflicting answers to a single question

(B)

firmly supports a young geologists tentative theory

(C) confidently uses a small piece of evidence to build an exact hypothesis

(D) courageously defies a conventional interpretation of Antarctic evidence

(E)

defiantly espouses an unpopular theory about comets in our solar system

ANSWERS

eg

AND EXPLANATIONS

is

Explanation for Correct Answer B : Choice (B) is correct. "Designated" means named. The passage makes it clear that ALH 84001 is the name given to the rock by scientists.

te re

(E)

allocated

d
2006-11-12

(D) selected

file://E:\\c3.htm

The Official SAT Online Course

18/22

Explanation for Correct Answer C : Choice (C) is correct. The author views the scientists' ability to use minute details of chemical composition to match the rock found in Antarctica to rocks from particular regions studied on Mars as "bold," or confident.

Explanation for Incorrect Answer A : Choice (A) is incorrect. The passage does not offer conflicting answers to the question of the rock's origin; rather, the author explains that researchers "think they have pinpointed its [the rock's] former resting place to just two possible sites."

Explanation for Incorrect Answer B : Choice (B) is incorrect. The author does not mention a "young geologist" in the passage.

Explanation for Incorrect Answer D : Choice (D) is incorrect. Researchers were concerned with the rock's place of origin (lines 20-23), not its Antarctic discovery site.

Explanation for Incorrect Answer E : Choice (E) is incorrect. The author does not mention comets or theories about them.

19

The author uses the phrase this new intimacy (line 28) to refer to the
(A)

hands-on quality of the learning experience repre-sented by the shoebox diorama

(C) general acceptance of the theory that biological life once existed on Mars

(D) increased knowledge that scientists have about our solar system

(E)

way that events on one planet affect those on another

ANSWERS

U
20

nR

Explanation for Correct Answer D : Choice (D) is correct. In line 28, "intimacy" means close acquaintance. The author is referring to how much scientists have learned about the solar system in recent decades as compared to what was known 40 years ago (lines 33-35).

Explanation for Incorrect Answer A : Choice (A) is incorrect. The context of the phrase "this new intimacy" refers to how much scientists have learned about our solar system, not their "learning experience."

Explanation for Incorrect Answer B : Choice (B) is incorrect. Although the author does use herself and her daughter (both nonspecialists) as examples in this passage, her use of the phrase "this new intimacy" refers to what astronomers (specialists) know about our solar system.

Explanation for Incorrect Answer C : Choice (C) is incorrect. In the context of the passage, the phrase "this new intimacy" specifically refers to the solar system and does not allude to the possibility of life on Mars.

Explanation for Incorrect Answer E : Choice (E) is incorrect. The author uses the phrase "this new intimacy" to refer to increasing knowledge about our solar system. The author does not discuss whether events happening on one planet influence events on another.

In line 33, crude most nearly means

eg i

AND EXPLANATIONS

st e

(B)

understanding that nonspecialists now have about meteorological phenomena

re d

file://E:\\c3.htm

2006-11-12

The Official SAT Online Course

19/22

(A)

natural and unaltered

(B)

rough and inexpert

(C) obvious

(D) vulgar

(E)

nonspecific

ANSWERS

AND EXPLANATIONS

Explanation for Correct Answer B : Choice (B) is correct. "Crude" means rough or inexpert. The author's diorama and its materials"marbles, jack balls, and Ping-Pong balls" (lines 30-31)could well be described as "crude," or rough and inexpert, especially since the author used everyday materials in an attempt to model the solar system.

Explanation for Incorrect Answer A : Choice (A) is incorrect. The materials mentioned ("marbles, jack balls, and PingPong balls") were manufactured items, not natural ones.

21

nR
(A)

In line 42, the author refers to Plutos moon most likely in order to

illustrate a feature of our solar system discovered since the authors childhood

(B)

cite an object too small in scale to have been included in the authors diorama

(C) draw a parallel between it and our own moon

(D) contrast the scientific curiosity of todays children with that of children years ago

(E)

emphasize the need for a greater commitment to space exploration

ANSWERS

Explanation for Correct Answer A : Choice (A) is correct. The author refers to Pluto's moon in order to compare our current knowledge of our solar system with that of the past. The author's daughter would need to include Pluto's moon if she were to build a diorama, because the moon's existence is now current knowledge.

Explanation for Incorrect Answer B : Choice (B) is incorrect. Pluto's moon was absent from the author's diorama not because it was too small, but because it had not yet been discovered.

Explanation for Incorrect Answer C : Choice (C) is incorrect. The author is comparing past and present knowledge about objects in space, not the objects themselves.

eg

Explanation for Incorrect Answer E : Choice (E) is incorrect. On the contrary, the author's account suggests that the materials used in the diorama were chosen quite specifically to represent objects in the solar system.

AND EXPLANATIONS

is t

er

Explanation for Incorrect Answer D : Choice (D) is incorrect. "Vulgar" means indecent or in poor taste. The passage does not suggest that the materials used in the diorama were either indecent or in poor taste.

ed

Explanation for Incorrect Answer C : Choice (C) is incorrect. "Obvious" means easily perceived or understood. The author's description emphasizes the fact that the materials used in the diorama were rough and unsophisticated, not the idea that they were easily perceived or understood.

file://E:\\c3.htm

2006-11-12

The Official SAT Online Course

20/22

Explanation for Incorrect Answer D : Choice (D) is incorrect. The author is not comparing the children's curiosity. Instead, the author is comparing the knowledge of today's children with that of children years ago.

Explanation for Incorrect Answer E : Choice (E) is incorrect. Although the author might well be in favor of continued space exploration, the idea is not mentioned in the passage.

22

The reasoning process presented in lines 49-53 (As . . . stars) is best described as
(A)

inference based on an untested theory

(B)

extrapolation from similar situations

(C) analysis of a single case by multiple observers

(D) hypothesis confirmed by direct observation

(E)

comparison of theory with physical evidence

ANSWERS

AND EXPLANATIONS

U
23

nR eg
(A)

Explanation for Incorrect Answer A : Choice (A) is incorrect. The author mentions that the basis for knowledge about these planets is the "gravitational effects" these planets "exert on their parent stars" (lines 52-53). Thus the basis of knowledge about these planets is the theory of gravity. This is not an untested theory.

Explanation for Incorrect Answer C : Choice (C) is incorrect. The author does not mention the number of observers involved. Further, it seems unlikely that the number of observers involved would alter any description of the reasoning process.

Explanation for Incorrect Answer D : Choice (D) is incorrect. The passage states that none of the planets have "actually been seen through a telescope" (lines 50-51). Thus none of the planets mentioned in lines 49-53 have been directly observed.

Explanation for Incorrect Answer E : Choice (E) is incorrect. Although the author mentions the discovery of a rock from Mars (lines 5-6), no physical evidence has been collected to support the existence of the planets described in lines 49-53.

In lines 53-59 the author refers to the Goldilocks fairy tale (Yet . . . water) in order to make which point about a particular planet?

The planets environment may be conducive to a result some scientists are eager to find.

(B)

The planets atmosphere was once thought to be too cold to support biological life.

(C) The simple methods astronomers used to discover the features of this planet resemble the explorations of curious children.

(D) Scientists wishful speculations about the existence of this planet deserve little more credence than a fairy tale.

(E)

Only after much trial and error did astronomers determine the precise location of this planet.

is te

re d

Explanation for Correct Answer B : Choice (B) is correct. "Extrapolate" means infer. The passage states, "we know about them [the planets] indirectly through the gravitational effects they exert on their parent stars" (51-53). In other words, the passage describes planets that are known only by their gravitational effects. This knowledge would have to be arrived at by extrapolation, or inference, from observations of known planets and the gravitational effects of their planet stars.

file://E:\\c3.htm

2006-11-12

The Official SAT Online Course

21/22

ANSWERS

AND EXPLANATIONS

Explanation for Correct Answer A : Choice (A) is correct. The planet was nicknamed "Goldilocks" because its temperature was "just right," or perfect, for the presence of liquid water. The possibility of liquid water, "not known to exist anywhere in our solar system now except on Earth, is thought crucial to biological life." Scientists are eager to find a planet with liquid water because it could support life.

Explanation for Incorrect Answer B : Choice (B) is incorrect. The passage does not say what scientists previously thought about the atmospheric temperature of the planet.

Explanation for Incorrect Answer C : Choice (C) is incorrect. In lines 53-59, the author uses the fairy tale to make a point about what Goldilocks found (things that were "just right"), not to make a point about the methods that Goldilocks used.

Which of the following, if true, would the hopeful scientists (line 62) most likely interpret as evidence of the potential for life on Mars?
(A)

nR
(B)

Mars was affected by the same planetary collision that caused the extinction of dinosaurs.

Mars had a very mild atmospheric temperature millions of years ago.

(C) Mars had a wet environment at one time in the past.

(D) The rock that fell from Mars resembled rocks found on the Antarctic ice field.

(E)

The rock that fell from Mars had very few microscopic fissures.

ANSWERS

Explanation for Correct Answer C : Choice (C) is correct. Since water "is thought crucial to biological life" (line 61), evidence that Mars was once wet would be seen as suggesting that life could have existed there.

Explanation for Incorrect Answer A : Choice (A) is incorrect. Although the passage does mention the possibility that Mars may have experienced a "planetary collision of the sort that purportedly killed off our dinosaurs" (lines 8-9), the passage does not suggest that such a planetary collision would have any bearing on Mars's capacity to support life.

Explanation for Incorrect Answer B : Choice (B) is incorrect. A mild atmospheric temperature need not necessarily include water, which is essential to life. Furthermore, there is no mention of the effects of a mild atmospheric temperature on the development of life on Mars.

Explanation for Incorrect Answer D : Choice (D) is incorrect. The passage does not say whether rocks in Antarctica have

eg

24

AND EXPLANATIONS

is

te

Explanation for Incorrect Answer E : Choice (E) is incorrect. Although Goldilocks did find things that were "just right" through trial and error, the author does not mention the use of trial and error in her discussion of this planet's location. Instead, the author simply states that the planet was found by measuring the gravitational effects exerted on its parent star (lines 51-53).

re

Explanation for Incorrect Answer D : Choice (D) is incorrect. Though the planet has not been observed visually, the author does not express any doubt that it exists. Further, the author describes scientists' means of discovering this and other planets: "we know about them indirectly through the gravitational effects they exert on their parent stars" (lines 51-53).

file://E:\\c3.htm

2006-11-12

The Official SAT Online Course


any properties that would suggest the potential for life, or that rocks in the Antarctic ice field bear any resemblance, or likeness, to rocks from Mars.

22/22

Explanation for Incorrect Answer E : Choice (E) is incorrect. Fossils in a rock, which the author describes as "suggestive traces of something that might once have lived and died" (lines 26-27), are mentioned as evidence suggestive of life. Fissures, or cracks, are not.

Back to Score Report

Copyright 2006 The College Board. All rights reserved.

Privacy Policy

Terms of Use

Contact Us

nR

eg is te r
file://E:\\c3.htm

ed
2006-11-12

The Official SAT Online Course

1/12

Help | Profile | My Organizer | My Bookmarks | Logout

Answers and Explanations

Test Sections

Back to Score Report

Section 1

View Answers and Explanations


Online - Practice Test #3

Section 2

Section 3

Section 4

Each month, a telephone service charges a base rate of

and an additional

Section 5

per call for the first


(A)
(B)
(C)
(D)
(E)

calls and

for every call after that. How much

Section 7

does the telephone service charge for a month in which

calls are made?

Section 8

Section 9

Section 10

ANSWERS

AND EXPLANATIONS

Explanation for Correct Answer C : calls will be Choice (C) is correct. The bill for

. This simplifies to

nR eg
cost of

Explanation for Incorrect Answer A :

is
Choice (A) is not correct. This amount,

te
.

plus

re
would only account for the base

d
in additional charges for

individual calls. The first

calls, which would cost

would

cost more than

Explanation for Incorrect Answer B :

Choice (B) is not correct. This amount,

would only account for the base

cost of

plus

in additional charges for

individual calls. The first

calls, which would cost

would

cost more than

Explanation for Incorrect Answer D :

Choice (D) is not correct. This amount,

would be enough for the base cost

of

plus

in additional charges for calls. This

would be enough for an average per-call charge of

which is

larger than the given per-call charges of either

or

Explanation for Incorrect Answer E :

Choice (E) is not correct. This amount,

would be enough for the base cost

of

plus

in additional charges for calls. This

would be enough for an average per-call charge of

which is

larger than the given per-call charges of either

or

file://E:\\c4.htm

2006-11-12

The Official SAT Online Course

2/12

experienced its largest increase in According to the chart above, Company monthly profits between which two consecutive months?
(A)

January and February

(B)

February and March

(C) March and April

(D) April and May

(E)

May and June

Between February and March, profits decreased by about

er e
is

Explanation for Correct Answer C : Choice (C) is correct. Between January and February, profits increased by about

ANSWERS

AND EXPLANATIONS

Between March and April, profits increased by about

Between April and

May, profits decreased by about

Between May and June, profits increased

an increase of

nR
about

Explanation for Incorrect Answer A : Choice (A) is not correct. Between January and February the profits did increase by

U
3

Explanation for Incorrect Answer B : Choice (B) is not correct. Between February and March profits decreased.

Explanation for Incorrect Answer D : Choice (D) is not correct. Between April and May profits decreased.

Explanation for Incorrect Answer E : Choice (E) is not correct. Between May and June, profits increased by about

eg i

but between March and April profits increased by about

but between March and April profits increased by about

st

by

The largest monthly increase occurred between March and April, with

In the figure above, the measure of

the measure of

If the

file://E:\\c4.htm

2006-11-12

The Official SAT Online Course

3/12

measure of

is

the measure of a right angle, what is the measure of

(A)
(B)
(C)
(D)
(E)

ANSWERS

AND EXPLANATIONS

Explanation for Correct Answer A :

Choice (A) is correct. A right angle measures

so

Since

is

the measure of

ed
the measure of

Explanation for Incorrect Answer B :

Choice (B) is not correct. This would be the measure of

if its measure

were

the measure of a right angle, instead of

the measure of

Explanation for Incorrect Answer C :

te r
is

Choice (C) is not correct. This choice is

but the

eg is

problem states that the measure of

the measure of

Explanation for Incorrect Answer D :

Choice (D) is not correct. This is the measure of

not the measure of

nR

Explanation for Incorrect Answer E :

Choice (E) is not correct. The measure of

is less than the measure of

which is less than

so the measure of

cannot be more than

U
4

Each number to the Each square in the grid above is to be filled with either or right of the grid is the sum of the numbers in the row to its left, and each number below the grid is the sum of the numbers in the column above it. For example, there below the third column because the sum of the numbers in that column is is a be? are all entered correctly into the grid, what will row and When the

(A)

file://E:\\c4.htm

2006-11-12

The Official SAT Online Course

4/12

(B)
(C)
(D)
(E)

ANSWERS

AND EXPLANATIONS
is

Explanation for Correct Answer E : Choice (E) is correct. Since the sum of row

there cannot be any

in

row

and it must be filled with

On the other hand, the sum for row

is

so the three empty boxes in that row can be filled in with

The same is true

for the three empty boxes in the second column, which also has a sum of

The

first column, in order to have a sum of

must have zeros except for the

already

in the first position in row column four boxes on row box to have a total sum of

At this point, there are only two empty boxes, the in the column four needs a Row and on row reads (from left to right) In summary, row

te re
row

Explanation for Incorrect Answer A : must have a Choice (A) is not correct. Row

sum in the first column is only

cannot also have a

Explanation for Incorrect Answer B : Choice (B) is not correct. This choice contradicts the given information that the third column is all zeroes.

Explanation for Incorrect Answer C : Choice (C) is not correct. This choice contradicts the given information that the third column is all zeroes.

Explanation for Incorrect Answer D : in the second column of row Choice (D) is not correct. Without a

eg is

d
in the first column and since the

in the first column.

the second

U
If
I.

nR
II.

column would not have a sum of

which of the following can be true?

III.

(A)

None

(B)

I only

(C) II only

(D) I and II only

(E)

I, II, and III

ANSWERS

AND EXPLANATIONS
either

Explanation for Correct Answer E :

Choice (E) is correct. Since

or

or

both. If

then

If

then

It is possible for either

to equal or all be true.

or for both

and

to equal

Therefore, I, II, and III can

file://E:\\c4.htm

2006-11-12

The Official SAT Online Course

5/12

Explanation for Incorrect Answer A :

Choice (A) is not correct. Since

either

or

or both, must

equal

Explanation for Incorrect Answer B : Choice (B) is not correct. It is possible that

but it is also possible that

and

Explanation for Incorrect Answer C : Choice (C) is not correct. It is possible that

but it is also possible that

and

Explanation for Incorrect Answer D : Choice (D) is not correct. It is possible that both

and

are equal to

and so

can be true.

eg

is
Which of the following could be the equation of the graph of function above?

te
shown

nR
(A)
(B)
(C)

(D)
(E)

ANSWERS

AND EXPLANATIONS
-axis at only one point,

Explanation for Correct Answer B : Choice (B) is correct. The graph shown intersects the

This implies that the quadratic function has the form

where

is a constant. Every answer choice in this problem has leading coefficient so the only possible choice that could represent the graph shown is the

equation

Explanation for Incorrect Answer A : Choice (A) is not correct. The graph of this equation intersects the

re
-axis at two

d
points,

and

because

This equation

cannot represent the graph shown.

file://E:\\c4.htm

2006-11-12

The Official SAT Online Course

6/12

Explanation for Incorrect Answer C : Choice (C) is not correct. The graph of this equation does not intersect the

-axis

at the point

because

not

This equation cannot

represent the graph shown.

Explanation for Incorrect Answer D : Choice (D) is not correct. The graph of this equation does not intersect the

-axis

at the point

because

not

This equation cannot

represent the graph shown.

Explanation for Incorrect Answer E : Choice (E) is not correct. The graph of this equation does not intersect the

-axis

at the point

because

not

This equation cannot

represent the graph shown.

er e
is the midpoint of

In

above,

and

(B)

(approximately

(C)

nR eg
(D)

(approximately

(approximately

(E)

(approximately

ANSWERS

AND EXPLANATIONS

Explanation for Correct Answer A : Choice (A) is correct. Since it is given that the measures of two of the three angles

is t
)

(A)

(approximately

d
What is the length of
of triangle

are

triangle

is equilateral. Also, since

is the

midpoint of

and

Since

is

isosceles and the measures of

and

are

Thus,

must

be a

triangle. Based on the properties of

triangles,

Explanation for Incorrect Answer B : Choice (B) is not correct. Since it is given that the measures of two of the three

angles of triangle

are

triangle

is equilateral. Also,

and

Since

is isosceles and the

measures of

and

are

Thus,

must be a

The properties of such a special right angle triangle do not

include sides with lengths

and one that is an integer multiple of

Explanation for Incorrect Answer C : Choice (C) is not correct. An answer of about

is too large. Since the three

line segments

and

form a triangle, the length of

must be less

than the sum of the lengths of

and

Because both

and

have

length

the length of

must be less than

file://E:\\c4.htm

2006-11-12

The Official SAT Online Course

7/12

Explanation for Incorrect Answer D : Choice (D) is not correct. An answer of about

is too large. Since the three line

segments

and

form a triangle, the length of

must be less than

the sum of the lengths of

and

Because both

and

have length

the length of

must be less than

Explanation for Incorrect Answer E : Choice (E) is not correct. An answer of about

is too large. Since the three line

segments

and

form a triangle, the length of

must be less than

the sum of the lengths of

and

Because both

and

have length

the length of

must be less than

If

and
(A)

are positive numbers, what percent of

is

(B)
(C)

(E)

ANSWERS

AND EXPLANATIONS

Explanation for Correct Answer E :

is

te
is

re
is of

d
divide

(D)

Choice (E) is correct. To find what fraction

by

eg

To put this in percent form, multiply by

to get

nR

Explanation for Incorrect Answer A :

Choice (A) is not correct. If the formula

were correct, it would work

for any values of

and

Certainly,

of

However, using

and

in this formula gives

not

Explanation for Incorrect Answer B :

Choice (B) is not correct. If the formula

were correct, it would work for

any values of

and

Certainly,

is

of

However, using

and

in this formula gives

not

Explanation for Incorrect Answer C :

Choice (C) is not correct. If the formula

were correct, it would work

for any values of

and

Certainly,

is

of

However, using

and

in this formula gives

not

Explanation for Incorrect Answer D :

Choice (D) is not correct. If the formula

were correct, it would work

file://E:\\c4.htm

2006-11-12

The Official SAT Online Course

8/12

for any values of

and

Certainly,

is

of

However, using

and

in this formula gives

not

How many

-pound sticks of butter together weigh as much as

pounds of

butter?

Your Response:

Correct Response(s): 100

Explanation:

The correct answer is

Since the weight of each stick is

pound,

sticks

together make a pound. To get

pounds, requires

sticks.

The problem can also be represented algebraically. Let

be the number of sticks

needed, then

therefore,

or

10

If

, what is the value of

Correct Response(s): 3

nR

Explanation:

eg
. The equation

Your Response:

is
can be solved for

The correct answer is

te re
and

using the following steps:

11

d
is

In isosceles triangle

the measure of angle

If another angle of the

triangle measures

where

what is one possible value of

Your Response:

Correct Response(s): 20 or 50

Explanation:

The correct answers are

and

. The sum of the measures of the angles must

be

and the question states that

Since the triangle is isosceles, two

of the angles must have the same measure. Either the angles are

and

or the angles are

and

This means that either

or

In the first case,

and in the

second case

Either answer is correct.

file://E:\\c4.htm

2006-11-12

The Official SAT Online Course

9/12

12

If

is directly proportional to

and if

when

what is the value of

when

Your Response:

Correct Response(s): 8/15, .533

Explanation:

The correct answer is

or

(a truncated form of the repeating decimal). If

is directly proportional to

then

for some constant

Using the value

of

when

gives the equation

Solving for

gives that

Then for

or

. The answer can be

entered into the grid either as a fraction or as a decimal.

13

nR
The Lyndhurst High School twelfth graders are represented in the circle graph in Figure 1. Figure 2 is another way to illustrate the use of computers by these twelfth twelfth graders are represented in both figures, what is the graders. If the same total number of twelfth graders represented by the shaded circle in Figure 2?
Your Response:

Correct Response(s): 180

Explanation:

. The shaded circle in Figure 2 represents all the The correct answer is students who use computers in school, including those who use computers both at students, or of the home and at school. From Figure 1 in the question,

eg

is

te re
file://E:\\c4.htm

d
2006-11-12

The Official SAT Online Course

10/12

students, use computers at school only, and

of the

students, or

students, use computers both at home and at school. The entire shaded circle,

then, represents

students who use computers at school.

14

In the figure above, the lengths and widths of rectangles

and

are whole

numbers. The areas of rectangles What is the area of the entire figure?

and

are

and

respectively.

Your Response:

Correct Response(s): 144

Explanation:

The correct answer is

. Rectangle

factored as

or

Rectangle

re d
has an area of

which can be

and rectangle

share a side. The

length of the shared side is either

or

since the area of rectangle

is

which has

and

as possible factors. If the length of the shared side were

then the dimensions of rectangle

would be

by

and

would be the

te
by

length of the side shared by rectangle

and rectangle

Since the area of

rectangle

is

it cannot have a side of length

From this it follows that the

length of the side shared by rectangle

and rectangle

must be

The length

is

of the side shared by rectangle

and rectangle

is

Since the area of

eg

rectangle

is

the other dimension of rectangle

is

Since

rectangle

has dimensions

rectangle

has dimensions

by

and

rectangle

has dimensions

by

the dimensions of the large rectangle are

nR
by

Therefore, the total area of the large rectangle is

U
15

In the chart above, if the number result of step

chosen in step

is

what number will be the

Your Response:

Correct Response(s): 20

Explanation:

file://E:\\c4.htm

2006-11-12

The Official SAT Online Course

11/12

The correct answer is

. First take the chosen number for step

which is

and use it in step

Here

is not prime, because

So add

to get

Then in step

divide this number by

to get

The

result of step

is the greatest integer less than or equal to

which is

16

In an art class, there were just enough staplers, rulers and glue bottles so that every students had to share a ruler, and every students had to share a stapler, every students had to share a glue bottle. If the sum of the number of staplers, rulers, how many students were in the class? and glue bottles used by the class was

Your Response:

Correct Response(s): 60

Explanation:

The correct answer is

. Let

represent the total number of students in the art

class. Since every

students had to share a stapler, there were exactly

staplers. Since every

students had to share a ruler, there were exactly

rulers. Since every

students had to share a glue bottle, there were exactly

glue bottles. From the given information that the sum of the number of staplers, it follows that rulers, and glue bottles used by the class was

er e
and
in terms of

which simplifies to

Therefore,

17

eg

is t
what is the greatest
. Since

If

and

are integers such that

nR

possible value of

Your Response:

Correct Response(s): 608

Explanation:

d
and since

and further to

The correct answer is

and

have no

common factors other than

it must be the case that

is a multiple of

and

is a multiple of

Solving for

yields

Since

or

or

The largest

multiple of

less than

is

So

and

and

which is less than

18

How many positive integers less than times an even integer?

are multiples of

and are equal to

Your Response:

file://E:\\c4.htm

2006-11-12

The Official SAT Online Course


Correct Response(s): 33

12/12

Explanation:

The correct answer is

. Since all of these integers are equal to

times an even

integer, they are all multiples of

and

It is given that these integers are also

they must and Since these integers must be multiples of multiples of numbers, which is all be multiples of the least common multiple of these Integers that fulfill all The integers must also be less than

these conditions are

and so on, up to

which is the greatest such number since

is not

less than

So there are

integers that meet these conditions.

Back to Score Report

Copyright 2006 The College Board. All rights reserved.

Privacy Policy

Terms of Use

Contact Us

nR

eg i

st

er e
file://E:\\c4.htm

d
2006-11-12

The Official SAT Online Course

1/21

Help | Profile | My Organizer | My Bookmarks | Logout

Answers and Explanations

Test Sections

Back to Score Report

Section 1

View Answers and Explanations


Online - Practice Test #3

Section 2

Section 3

Section 4

Section 5

Industrial growth that was being stifled by the countrys dictatorship, but now they are developing their full economic potential.
(A)

Section 7

Section 8

Industrial growth that was being stifled by the countrys dictatorship, but now they are developing their full economic potential.

Section 9

(B)

The dictatorship had stifled industrial growth, but the country is now developing their full economic potential.

Section 10

(C) Industrial growth was stifled by the countrys dictatorship, and so now they are developing their full economic potential.

(D) Though the dictatorship had stifled industrial growth, the country is now developing its full economic potential.

(E)

Now developing their full economic potential, the countrys dictatorship had stifled industrial growth.

nR eg
(A)

Explanation for Incorrect Answer A : Choice (A) is a sentence fragment. Although the clause after the coordinating conjunction "but" is complete, the phrase before the conjunction has no independent verb and therefore does not state a complete thought.

Explanation for Incorrect Answer B : Choice (B) has an error in pronoun agreement. The plural pronoun "their" does not agree with the singular noun to which it presumably refers, "country."

Explanation for Incorrect Answer C : Choice (C) uses vague pronouns. The sentence contains no nouns to which the plural pronouns "they" and "their" can refer.

Explanation for Incorrect Answer E : Choice (E) exhibits improper modification. The introductory phrase, "Now developing their full economic potential," cannot logically modify the noun that follows, "dictatorship."

Looking down through the boats glass bottom, a school of yellow fish was seen swimming along with the turtles.

a school of yellow fish was seen

(B)

a school of yellow fish were seen

(C) we saw a school of yellow fish

(D) we seen a school of yellow fish

(E)

yellow fish in schools are seen

ANSWERS

AND EXPLANATIONS

is

te

Explanation for Correct Answer D : Choice (D) is correct. It avoids the error of the original by properly combining a dependent clause (before the comma) with an independent clause (after the comma) to state a complete thought. The logical relationship of the two parts of the sentence is introduced by the subordinating conjunction "though."

re

ANSWERS

AND EXPLANATIONS

d
2006-11-12

file://E:\\c5.htm

The Official SAT Online Course

2/21

Explanation for Correct Answer C : Choice (C) is correct. It avoids the error of the original by placing the pronoun "we" immediately after the introductory phrase ("Looking down through the boats glass bottom") that modifies it.

Explanation for Incorrect Answer A : Choice (A) exhibits improper modification. The introductory phrase, "Looking down through the boats glass bottom," cannot logically modify the noun phrase that immediately follows, "a school of yellow fish."

Explanation for Incorrect Answer B : Choice (B) has an error in subject-verb agreement. Although the plural verb "were" agrees with the plural intervening noun "fish," it does not agree with the singular subject "school."

Explanation for Incorrect Answer D : Choice (D) uses an incorrect form of the irregular verb. The context requires not the past participle, "seen," but the past tense, "saw."

Explanation for Incorrect Answer E : Choice (E) displays improper modification. The introductory phrase, "Looking down through the boats glass bottom," does not modify the noun that immediately follows, "fish."

(A)

waves and of transmitting those waves through space; after this those waves must be transformed

(B)

waves, transmitting the waves through space, and transforming them

(D) waves and of transmitting them through space; after this the waves have to be translated

nR
ANSWERS

(E)

waves, of the transmitting of those waves through space and of translating same

Explanation for Correct Answer B : Choice (B) is correct. It avoids the error of the original by using three parallel verbal phrases ("transforming sounds," "transmitting the waves," and "transforming them") to describe the three functions of a radio system.

Explanation for Incorrect Answer A : Choice (A) fails to maintain parallelism. It uses parallel verbal phrases ("transforming sounds" and "transmitting those waves") to describe the first two functions of a radio system but shifts (after the semicolon) to an independent clause to describe the third function.

Explanation for Incorrect Answer C : Choice (C) violates parallelism. The noun "translation" is not parallel with the earlier verbal forms "transforming" and "transmitting."

Explanation for Incorrect Answer D : Choice (D) displays a flaw in parallelism. The independent clause used (after the semicolon) to describe the third function of a radio system is not parallel with the verbal phrases ("transforming sounds" and "transmitting them") that describe the first two functions.

Explanation for Incorrect Answer E : Choice (E) uses an inappropriate idiom. As a substitute for the noun "waves," the pronoun "same" is less effective than the pronoun "them."

eg

(C) waves, of transmitting them through space, and then the translation of them

AND EXPLANATIONS

is te

A radio system consists of a means of transforming sounds into electromagnetic waves and of transmitting those waves through space; after this those waves must be transformed back into sounds.

re

d
2006-11-12

file://E:\\c5.htm

The Official SAT Online Course

3/21

It underlies the poem that human beings are free to choose and may be blamed for their choices.
(A)

It underlies the poem

(B)

In the poem, they assumed

(C) In the poem, a basic assumption which is made is

(D) It is an assumption that underlies the poem

(E)

The basic assumption of the poem is

ANSWERS

AND EXPLANATIONS

Explanation for Correct Answer E : Choice (E) is correct. It avoids the error of the original by using a specific noun, "assumption," instead of the vague pronoun "it."

Explanation for Incorrect Answer A : Choice (A) uses a vague pronoun. Since the pronoun "it" apparently refers to the entire idea stated by the long dependent clause (introduced by "that"), the meaning is unclear.

nR
(A)

The modern city may not have new citadels or cathedrals, but there is a great many new office buildings and freeways.

but there is a great many new office buildings and freeways

(B)

but it does have a great many new office buildings and freeways

(C) but a great many new office buildings and freeways

(D) although many new office buildings and freeways are there

(E)

although a great many new office buildings and freeways are seen

ANSWERS

Explanation for Correct Answer B : Choice (B) is correct. It avoids the error of the original by using both a singular subject, "it," and a singular verb, "does have," to form a clause that is parallel to the earlier clause, "The modern city may not have."

Explanation for Incorrect Answer A : Choice (A) involves subject-verb disagreement. In the inverted construction introduced by "there," the singular verb "is" does not agree with its plural compound subject, "buildings and freeways."

Explanation for Incorrect Answer C : Choice (C) fails to maintain parallelism. The phrase without any verb ("but a great many new office buildings and freeways") is not parallel with the independent clause before the comma.

eg

Explanation for Incorrect Answer D : Choice (D) contains an unnecessary word. With a minor change in word order ("An assumption that underlies the poem is"), the vague pronoun "it" can be eliminated.

AND EXPLANATIONS

is te

Explanation for Incorrect Answer C : Choice (C) exhibits wordiness. The clause "which is made" can be deleted without any loss of meaning.

re

Explanation for Incorrect Answer B : Choice (B) uses a vague pronoun. The sentence contains no noun prior to the pronoun "they" to which that pronoun can refer.

file://E:\\c5.htm

2006-11-12

The Official SAT Online Course

4/21

Explanation for Incorrect Answer D : Choice (D) violates parallelism. The phrase "are there" (with "office buildings and freeways" as its subject) is not parallel with the earlier verb phrase "may not have" (with "citadels or cathedrals" as its direct object).

Explanation for Incorrect Answer E : Choice (E) displays flawed parallelism. The verb phrase "are seen" (with "office buildings and freeways" as its subject) is not parallel with the earlier verb phrase "may not have" (with "citadels or cathedrals" as its direct object).

The remains of the Apatosaurus provide evidence of there being giants existing on Earth during the late Jurassic period.
(A)

of there being giants existing

(B)

of there having been giants existing

(C) of there existing giants

(D) that giants have existed

(E)

that giants existed

ANSWERS

AND EXPLANATIONS

nR
(A)

Explanation for Incorrect Answer C : Choice (C) contains an unnecessary word. The awkward phrase, "of there existing giants," can be simplified and shortened to "of giants existing."

Explanation for Incorrect Answer D : Choice (D) uses an improper verb tense. The present perfect tense, "have existed," incorrectly suggests that the existence of giants continues up to the present time.

Farming in that area of the country is inefficient because of their farm machinery shortage and that they lack the knowledge to operate it.

because of their farm machinery shortage and that they lack the

(B)

in that they have a shortage of farm machinery and a lack of

(C) because of the shortage of farm machinery and their lacking the

(D) in that there is both a shortage of farm machinery as well as a lack of

(E)

because of the shortage of farm machinery and the lack of

ANSWERS

Explanation for Correct Answer E : Choice (E) is correct. It avoids the error of the original by omitting the vague and unnecessary pronouns "their" and "they."

Explanation for Incorrect Answer A :

eg

Explanation for Incorrect Answer B : Choice (B) exhibits wordiness. The idea expressed with six words ("of there having been giants existing") can be stated more smoothly and efficiently with only three ("that giants existed").

AND EXPLANATIONS

is t

Explanation for Incorrect Answer A : Choice (A) displays wordiness. Five words ("of there being giants existing") can be reduced to three ("that giants existed").

er e

Explanation for Correct Answer E : Choice (E) is correct. It avoids the error of the original by using a straightforward clause ("that giants existed") and by eliminating unnecessary words.

file://E:\\c5.htm

2006-11-12

The Official SAT Online Course

5/21

Choice (A) includes vague pronouns. The sentence contains no plural noun to which the plural pronouns "their" and "they" can refer.

Explanation for Incorrect Answer B : Choice (B) uses a vague pronoun. The meaning of the plural pronoun "they" remains uncertain because the sentence has no plural noun.

Explanation for Incorrect Answer C : Choice (C) contains a vague pronoun. Since the sentence has no plural noun, the use of the plural pronoun "their" is incorrect.

Explanation for Incorrect Answer D : Choice (D) uses an inappropriate idiom. Since the adjective "both" introduces two parallel nouns ("shortage" and "lack"), the proper conjunction to link them is "and," not "as well as."

The most popular painting in the exhibit of works by local artists was done by a seventy-year-old woman, who painted an exquisite self-portrait of herself.
(A)

was done by a seventy-year-old woman, who painted an exquisite selfportrait of herself (B) was an exquisite self-portrait of a seventy-year-old woman, who painted herself (C) was an exquisite self-portrait of a seventy-year-old woman

(E)

was by a seventy-year-old woman, being an exquisite self-portrait

Explanation for Correct Answer C : Choice (C) is correct. It avoids the wordiness of the original by improving word order (placing "an exquisite self-portrait" immediately after the linking verb "was") and by eliminating the unnecessary words "done," "who painted," and "of herself."

nR
(A)

Explanation for Incorrect Answer A : Choice (A) uses excess words. With a minor change in word order (placing "an exquisite self-portrait" immediately after the linking verb "was"), the words "done," "who painted," and "of herself" become unnecessary.

Explanation for Incorrect Answer B : Choice (B) exhibits wordiness. The final clause, "who painted herself," repeats the idea contained in "self-portrait" and is therefore unnecessary.

Explanation for Incorrect Answer D : Choice (D) displays wordiness. If the phrase "[an] exquisite self-portrait" is placed immediately after the linking verb "was," the words "done" and "and it is her own" are not needed.

Explanation for Incorrect Answer E : Choice (E) has an excess word. The word "being" is not needed if the noun phrase "an exquisite self-portrait" is placed before the prepositional phrase "by a seventyyear-old woman."

The tiles are sorted not only by their surface appearance but also according to their hardness and their capacity of conducting heat.

according to their hardness and their capacity of conducting heat

(B)

according to their hardness and of their heat-conducting capacity

(C) by their hardness and if they have the capacity for heat conduction

(D) by their hardness and their capacity for conducting heat

(E)

by their hardness and capacity in heat conduction

eg i

st e

ANSWERS

AND EXPLANATIONS

re
2006-11-12

file://E:\\c5.htm

(D) was done by a seventy-year-old woman, and it is her own exquisite selfportrait

The Official SAT Online Course

6/21

ANSWERS

AND EXPLANATIONS

Explanation for Correct Answer D : Choice (D) is correct. It avoids the error of the original by using the same preposition, "by," to introduce parallel phrases describing the two criteria used in sorting tiles.

Explanation for Incorrect Answer A : Choice (A) fails to maintain parallelism. The preposition "according to" is not parallel with the earlier preposition "by" even though both introduce phrases describing criteria for sorting tiles.

Explanation for Incorrect Answer B : Choice (B) displays wordiness. The preposition "of" is unnecessary.

Explanation for Incorrect Answer C : Choice (C) contains excess words. The phrase "if they have the" can be reduced to one word"their."

Explanation for Incorrect Answer E : Choice (E) uses an inappropriate idiom. "In" is not the correct preposition to link the noun "capacity" with the later phrase "heat reduction."

10

Radio frequencies have to be allocated to users so that one transmission will not interfere with another.
(A)

one transmission will not interfere with another

(B)

each transmission cant interfere with anothers

(C) transmitting them will not interfere with one another

(D) no transmission is interfered with by another

11

nR eg
ANSWERS
(A)

(E)

no one transmission would have interference with the other

AND EXPLANATIONS

Explanation for Correct Answer A : Choice (A) is correct. It uses the appropriate noun, "transmission," and avoids excess words in describing an event.

Explanation for Incorrect Answer B : Choice (B) uses an incorrect pronoun form. The form "another's" refers illogically to something possessed by another transmission rather than to the transmission itself.

Explanation for Incorrect Answer C : Choice (C) contains a vague and ambiguous pronoun. The pronoun "them" (apparently referring to "transmissions") cannot logically refer to either of the nouns in the sentence, "frequencies" or "users."

Explanation for Incorrect Answer D : Choice (D) displays wordiness. Four words ("is interfered with by") can be reduced to two ("interferes with").

Explanation for Incorrect Answer E : Choice (E) uses excess words. The idea expressed with four words ("would have interference with") can be stated better with only two ("interferes with").

The Pony Express was an ingenious system for carrying mail; it was in existence only briefly, however, before the telegraph system made it obsolete.

mail; it was in existence only briefly, however,

is te re

d
file://E:\\c5.htm

2006-11-12

The Official SAT Online Course

7/21

(B)

mail, for it was in existence only briefly, however,

(C) mail; however, existing only briefly

(D) mail, having existed only briefly

(E)

mail, but was existing only briefly

ANSWERS

AND EXPLANATIONS

Explanation for Correct Answer A : Choice (A) is correct. It uses two appropriately worded independent clauses that are linked correctly by a semicolon.

Explanation for Incorrect Answer B : Choice (B) uses an inappropriate idiom. The conjunction "for" incorrectly suggests that the clause to follow will present a reason for the condition described in the preceding clause.

Explanation for Incorrect Answer C : Choice (C) exhibits improper coordination. It incorrectly uses a semicolon to link parts of unequal grammatical rank (a complete thought before the semicolon and a phrase after it).

12

At first we panicked when we discovered we had missed our flight, but then we took a

nR
ANSWERS

bus to another airport, where there are several planes leaving for Denver that

evening. No error

Corrected Sentence: At first we panicked when we discovered we had missed our flight, but then we took a bus to another airport, where there were several planes leaving for Denver that evening.

Explanation for Correct Answer C : The error in this sentence occurs at (C), where the tense sequence is incorrect. The present tense of the verb, "are," is inconsistent with the earlier past-tense verbs, "panicked," "discovered," and "took."

Explanation for Incorrect Answer A : There is no error at (A). The past perfect tense, "had missed," is appropriate to describe an action that was completed before the time of another action in the past (the discovery).

Explanation for Incorrect Answer B : There is no error at (B). The past-tense verb "took" is consistent with other verbs in

eg is

Explanation for Incorrect Answer E : Choice (E) uses an inappropriate verb tense. The past progressive tense "was existing" suggests a continuing state of being that is inconsistent with the completed state of being indicated by use of the simple past "was" in the first clause.

AND EXPLANATIONS

te

re

Explanation for Incorrect Answer D : Choice (D) displays inappropriate modification and word order. If placed at the beginning of the sentence, the phrase introduced by "having existed" might modify "The Pony Express," but it does not modify the noun immediately before it, "mail."

file://E:\\c5.htm

2006-11-12

The Official SAT Online Course

8/21

past tense ("panicked" and "discovered").

Explanation for Incorrect Answer D : There is no error at (D). The participle "leaving" functions correctly to introduce an adjective phrase describing the noun "planes."

Explanation for Incorrect Answer E : There is an error in the sentence.

13

Only after the floodwaters had

rose

two feet

was

the mayor willing

to order the

evacuation of

some homes.

No error

ANSWERS

AND EXPLANATIONS

Corrected Sentence: Only after the floodwaters had risen two feet was the mayor willing to order the evacuation of some homes.

14

nR
ANSWERS

Explanation for Incorrect Answer D : There is no error at (D). The noun "evacuation" serves correctly as the direct object of the infinitive "to order," and the preposition "of" introduces a phrase that modifies "evacuation."

Explanation for Incorrect Answer E : There is an error in the sentence.

Confucianism is more a code of ethics than like a religion; it presents no deities

but fosters instead a respect for ones ancestors and for an orderly society. No error

Corrected Sentence: Confucianism is more a code of ethics than a religion; it presents no deities but fosters instead a respect for ones ancestors and for an orderly society.

Explanation for Correct Answer A : The error in this sentence occurs at (A), where there is wordiness. After the phrase "more . . . than," the word "like" is unnecessary and spoils the parallelism of the two ideas being compared ("a religion" and "a code") in the opening clause.

eg

Explanation for Incorrect Answer C : There is no error at (C). The infinitive "to order" functions properly to introduce the adjective phrase that completes the sentence.

AND EXPLANATIONS

is t

Explanation for Incorrect Answer B : There is no error at (B). In this inverted construction, the singular verb "was" agrees with its singular subject, "mayor," and the past tense correctly indicates a time after another action in the past (the rising of the floodwaters).

er

ed

Explanation for Correct Answer A : The error in this sentence occurs at (A), where the verb form is incorrect. After the helping verb "had," the correct form of this irregular verb is not the past, "rose," but the past participle, "risen."

file://E:\\c5.htm

2006-11-12

The Official SAT Online Course

9/21

Explanation for Incorrect Answer B : There is no error at (B). The conjunction "but" is appropriate to indicate a contrast, and the singular verb "fosters" agrees with its singular subject, "it."

Explanation for Incorrect Answer C : There is no error at (C). The adverb "instead" functions correctly after the verb "fosters" to indicate an alternative goal.

Explanation for Incorrect Answer D : There is no error at (D). The adjective "orderly" appropriately describes the noun "society."

Explanation for Incorrect Answer E : There is an error in the sentence.

15

Just as parents vary in their readiness to have their children leave home for college,
. young people vary in his or her readiness to leave No error

ANSWERS

AND EXPLANATIONS

Corrected Sentence: Just as parents vary in their readiness to have their children leave home for college, young people vary in their readiness to leave.

16

nR
ANSWERS

Explanation for Incorrect Answer A : There is no error at (A). The phrase "Just as" is an appropriate idiom to introduce a comparison between parents and young people.

Explanation for Incorrect Answer B : There is no error at (B). The verb "vary" is plural and agrees with its plural subject, "people."

Explanation for Incorrect Answer D : There is no error at (D). The infinitive "to leave" functions correctly as an adjective to modify the noun immediately before it, "readiness."

Explanation for Incorrect Answer E : There is an error in the sentence.

Local party organizations have discovered that voter turnout is diminished

considerably whenever the media projected election results early in the day. No error

Corrected Sentence: Local party organizations have discovered that voter turnout is diminished considerably whenever the media project election results early in the day.

Explanation for Correct Answer C : The error in this sentence occurs at (C), where the tense sequence is incorrect. The

eg

AND EXPLANATIONS

is t

Explanation for Correct Answer C : The error in this sentence occurs at (C), where the number of the pronouns is incorrect. The singular pronouns "his" and "her" do not agree with the plural noun to which they refer, "people."

er e

d
2006-11-12

file://E:\\c5.htm

The Official SAT Online Course

10/21

past tense of the verb, "projected," is inconsistent with the present tense of the earlier verb, "is diminished."

Explanation for Incorrect Answer A : There is no error at (A). The adverb "considerably" correctly modifies the verb immediately before it.

Explanation for Incorrect Answer B : There is no error at (B). The subordinating conjunction "whenever" properly introduces the dependent adverbial clause that completes the sentence.

Explanation for Incorrect Answer D : There is no error at (D). The prepositional phrase "in the day" functions correctly as an adverb telling when.

Explanation for Incorrect Answer E : There is an error in the sentence.

17

Absent from the speech were any mention of the students and laboratory technicians

ANSWERS

AND EXPLANATIONS

Corrected Sentence: Absent from the speech was any mention of the students and laboratory technicians upon whose contributions the chemist had depended heavily.

Explanation for Correct Answer B : The error in this sentence occurs at (B), where the verb does not agree with its subject. In this inverted construction, the plural verb "were" does not agree with the singular subject "mention."

18

nR

Explanation for Incorrect Answer A : There is no error at (A). The predicate adjective "absent" functions correctly to describe the noun "mention," and the preposition "from" introduces an adverb phrase that modifies "absent."

Explanation for Incorrect Answer C : There is no error at (C). The relative pronoun "whose" serves appropriately to link the preposition "upon" with its object, "contributions."

Explanation for Incorrect Answer D : There is no error at (D). The adverb "heavily" is used properly to modify the verb "had depended."

Explanation for Incorrect Answer E : There is an error in the sentence.

Pauls letter to myself about the missing money was not intended to be read by

any other member of the organization. No error

eg i

st er

ed

. upon whose contributions the chemist had depended heavily No error

file://E:\\c5.htm

2006-11-12

The Official SAT Online Course

11/21

ANSWERS

AND EXPLANATIONS

Corrected Sentence: Pauls letter to me about the missing money was not intended to be read by any other member of the organization.

Explanation for Correct Answer A : The error in this sentence occurs at (A), where the pronoun is incorrect. The reflexive form, "myself," is not appropriate, since the same person did not both send and receive the letter.

Explanation for Incorrect Answer B : There is no error at (B). The singular helping verb "was" agrees with its singular subject, "letter," and the past participle form "intended" is correct after the helping verb.

Explanation for Incorrect Answer C : There is no error at (C). The infinitive phrase "to be read" functions appropriately as an adverb telling how the letter was intended.

Explanation for Incorrect Answer D : There is no error at (D). The adverb "any" serves correctly to modify the adjective "other," and "other" properly modifies the noun "member."

Explanation for Incorrect Answer E : There is an error in the sentence.

19

, After the prince characterized modern architecture as ugly he has been severely

criticized for having been so outspoken in public. No error

ANSWERS

20

nR

Corrected Sentence: After the prince characterized modern architecture as ugly, he was severely criticized for having been so outspoken in public.

Explanation for Correct Answer B : The error in this sentence occurs at (B), where the tense sequence is inappropriate. The use of "After" with the past tense ("characterized") in the introductory clause requires the use of the past tense ("was . . . criticized") rather than the present perfect tense ("has been . . . criticized") in the main clause.

Explanation for Incorrect Answer A : There is no error at (A). The word "as" correctly precedes the adjective "ugly" after the verb "characterized."

Explanation for Incorrect Answer C : There is no error at (C). The adverb "severely" is appropriate to tell how the prince was criticized.

Explanation for Incorrect Answer D : There is no error at (D). The adjective "outspoken" is used correctly after the verbal "having been," and the adverb "so" properly modifies "outspoken."

Explanation for Incorrect Answer E : There is an error in the sentence.

eg

AND EXPLANATIONS

is t

er

ed

file://E:\\c5.htm

2006-11-12

The Official SAT Online Course

12/21

No matter

how

cautious

snowmobiles are driven, they

of damaging are the capable

land over which they travel.

No error

ANSWERS

AND EXPLANATIONS

Corrected Sentence: No matter how cautiously snowmobiles are driven, they are capable of damaging the land over which they travel.

Explanation for Correct Answer B : The error in this sentence occurs at (B), where an adjective is used incorrectly. An adverb ("cautiously") rather than the adjective ("cautious") is needed to modify the verb "are driven."

Explanation for Incorrect Answer A : There is no error at (A). The phrase "No matter" is an appropriate idiom to link the introductory dependent clause to the sentence's main clause.

21

nR
ANSWERS

The black squirrels drew a crowd of students, for it had never been seen on the

campus before. No error

Corrected Sentence: The black squirrels drew a crowd of students, for they had never been seen on the campus before.

Explanation for Correct Answer C : The error in this sentence occurs at (C), where the number of the pronoun is incorrect. The singular pronoun "it" does not agree with the plural noun to which it refers, "squirrels."

Explanation for Incorrect Answer A : There is no error at (A). The past-tense verb "drew" is correct to indicate a time after an earlier time in the past (when the squirrels were not seen).

Explanation for Incorrect Answer B : There is no error at (B). The conjunction "for" is used appropriately to join a clause describing an effect with a later clause stating a cause.

Explanation for Incorrect Answer D : There is no error at (D). After the helping verb "had," the verb forms "been" and "seen" are correct to form passive voice and past perfect tense.

eg is

Explanation for Incorrect Answer E : There is an error in the sentence.

AND EXPLANATIONS

te

Explanation for Incorrect Answer D : There is no error at (D). The preposition "of" is appropriate to begin the phrase that explains what snowmobiles can do, and the gerund "damaging" serves appropriately as the object of the preposition.

re

Explanation for Incorrect Answer C : There is no error at (C). The plural verb "are" agrees with its plural subject, "they," and its present tense is consistent with that of the earlier verb, "are driven."

file://E:\\c5.htm

2006-11-12

The Official SAT Online Course

13/21

Explanation for Incorrect Answer E : There is an error in the sentence.

22

A majority of the

students who attended

the job fair expressed interest in

becoming a doctor or lawyer No error .

ANSWERS

AND EXPLANATIONS

Corrected Sentence: A majority of the students who attended the job fair expressed interest in becoming doctors or lawyers.

Explanation for Correct Answer D : The error in this sentence occurs at (D), where there is noun-noun disagreement. The singular nouns "doctor" and "lawyer" do not agree with the earlier plural noun, "students."

Explanation for Incorrect Answer B : There is no error at (B). The verb in past tense, "attended," is consistent with the later past-tense verb, "expressed."

Explanation for Incorrect Answer C : There is no error at (C). The gerund "becoming" serves appropriately as the object of the preposition "in."

Explanation for Incorrect Answer E : There is an error in the sentence.

23

nR
ANSWERS

Never before had a group of artists been so isolated from society and from official

patronage as was the so-called Impressionists. No error

Corrected Sentence: Never before had a group of artists been so isolated from society and from official patronage as were the so-called Impressionists.

Explanation for Correct Answer D : The error in this sentence occurs at (D), where there is subject-verb disagreement. In this inverted construction, the singular verb "was" does not agree with its plural subject, "Impressionists."

Explanation for Incorrect Answer A : There is no error at (A). The helping verb "had" is used correctly to produce past perfect tense and thus indicate a time prior to the time of the Impressionists.

Explanation for Incorrect Answer B : There is no error at (B). After the helping verb "had been," the past participle form,

eg is te

AND EXPLANATIONS

re

Explanation for Incorrect Answer A : There is no error at (A). The noun "students" is used properly as the object of the preposition "of," and the relative pronoun "who" functions correctly to introduce an adjective clause describing "students."

file://E:\\c5.htm

2006-11-12

The Official SAT Online Course

14/21

"isolated," is correct, and "so" is an appropriate adverb to modify the verb.

Explanation for Incorrect Answer C : There is no error at (C). The conjunction "and" is appropriate to join two parallel prepositional phrases, and the repetition of the preposition "from" makes the parallelism more emphatic.

Explanation for Incorrect Answer E : There is an error in the sentence.

24

The flowers that Jane and Jonathan

ordered

to be sent

less to their mother fresh were

and much more expensive than

Carr's Flower Shop .

No error

ANSWERS

AND EXPLANATIONS

Corrected Sentence: The flowers that Jane and Jonathan ordered to be sent to their mother were less fresh and much more expensive than those from Carr's Flower Shop.

Explanation for Incorrect Answer B : There is no error at (B). The prepositional phrase "to their mother" functions correctly as an adverb telling where.

25

nR
ANSWERS

Explanation for Incorrect Answer C : There is no error at (C). The adverb "less" correctly modifies the adjective "fresh."

Explanation for Incorrect Answer E : There is an error in the sentence.

A possible first step in developing a nonsexist vocabulary with which to analyze the

works of the nineteenth-century writer Elizabeth Gaskell would be to stop referring to

her as Mrs. Gaskell. No error

Corrected Sentence: A possible first step in developing a nonsexist vocabulary with which to analyze the works of the nineteenth-century writer Elizabeth Gaskell would be to stop referring to her as Mrs. Gaskell.

Explanation for Correct Answer E : There is no error in this sentence.

eg

AND EXPLANATIONS

is

Explanation for Incorrect Answer A : There is no error at (A). The past-tense verb "ordered" is consistent with the later verb in past tense, "were."

te

re

Explanation for Correct Answer D : The error in this sentence occurs at (D), where the comparison is illogical. The flowers ordered are compared to a flower shop rather than to flowers from that shop.

file://E:\\c5.htm

2006-11-12

The Official SAT Online Course

15/21

Explanation for Incorrect Answer A : There is no error at (A). The preposition "in" and its object, the gerund "developing," correctly begin an adjective phrase that describes the noun "step."

Explanation for Incorrect Answer B : There is no error at (B). A phrase describing the noun "vocabulary" is properly introduced by the preposition "with" and its object, the pronoun "which."

Explanation for Incorrect Answer C : There is no error at (C). The preposition "of" is appropriate to introduce a phrase that describes the noun "works."

Explanation for Incorrect Answer D : There is no error at (D). The infinitive "to stop" functions correctly after the linking verb "would be" to name a possible first step.

26

After 140 years under the sea the remains of the Monitor, an ironclad warship that ,
was sunk during the Civil War, is being gradually brought to the surface. No error

27

nR
ANSWERS

Explanation for Incorrect Answer A : There is no error at (A). The preposition "after" is appropriate to begin a phrase that indicates a period of time.

Explanation for Incorrect Answer B : There is no error at (B). The preposition "under" properly introduces the prepositional phrase "under the sea."

Explanation for Incorrect Answer D : There is no error at (D). After the helping verb "is being," the correct verb form is the past participle, "brought," and the preposition "to" properly introduces an adverb phrase telling where.

Explanation for Incorrect Answer E : There is an error in the sentence.

I have gone to only one football game after I graduated from high school. No error

Corrected Sentence: I have gone to only one football game since I graduated from high school.

Explanation for Correct Answer C : The error in this sentence occurs at (C), where the idiom is inappropriate. In

eg is

Explanation for Correct Answer C : The error in this sentence occurs at (C), where the verb does not agree with its subject. Although the singular verb "is" agrees with the interrupting noun "warship," it does not agree with the plural subject "remains."

AND EXPLANATIONS

te

Corrected Sentence: After 140 years under the sea, the remains of the Monitor, an ironclad warship that was sunk during the Civil War, are being gradually brought to the surface.

re

ANSWERS

AND EXPLANATIONS

d
2006-11-12

file://E:\\c5.htm

The Official SAT Online Course

16/21

context, the conjunction "after" is less idiomatic than the conjunction "since."

Explanation for Incorrect Answer A : There is no error at (A). The preposition "to" correctly introduces an adverbial phrase identifying where the speaker went.

Explanation for Incorrect Answer B : There is no error at (B). The adverb "only" correctly modifies the adjective "one," and "one" is used appropriately to describe the noun "game."

Explanation for Incorrect Answer D : There is no error at (D). The past tense of the verb, "graduated," is correct to describe a completed action.

Explanation for Incorrect Answer E : There is an error in the sentence.

28

The radio station received the most number of calls from listeners on the evening it

aired a discussion of the music of Aretha Franklin. No error

ANSWERS

AND EXPLANATIONS

Corrected Sentence: The radio station received the largest number of calls from listeners on the evening it aired a discussion of the music of Aretha Franklin.

29

nR
ANSWERS

Explanation for Incorrect Answer B : There is no error at (B). The prepositional phrase "on the evening" functions correctly as an adverb telling when.

Explanation for Incorrect Answer C : There is no error at (C). The singular pronoun "it" agrees with the singular noun to which it refers, "station."

Explanation for Incorrect Answer D : There is no error at (D). The noun "music" serves appropriately as the object of the preposition "of," and the subsequent preposition "of" introduces a phrase that specifies the music that was discussed.

Explanation for Incorrect Answer E : There is an error in the sentence.

When the village elders present recommendations, there is hardly ever any opposition

against their proposals. No error

Corrected Sentence: When the village elders present recommendations, there is

eg

Explanation for Correct Answer A : The error in this sentence occurs at (A), where the idiom is inappropriate. Before "number" the adjective "most" is less idiomatic than "largest."

AND EXPLANATIONS

is

te

re
2006-11-12

file://E:\\c5.htm

The Official SAT Online Course

17/21

hardly ever any opposition to their proposals.

Explanation for Correct Answer D : The error in this sentence occurs at (D), where the idiom is not suitable. "Against" is not the appropriate preposition to introduce a phrase that follows the noun "opposition."

Explanation for Incorrect Answer A : There is no error at (A). The plural verb "present" agrees with its plural subject, "elders," and the present tense is consistent with that of the later verb "is."

Explanation for Incorrect Answer B : There is no error at (B). In this inverted construction, the singular verb "is" agrees with its singular subject, "opposition."

Explanation for Incorrect Answer C : There is no error at (C). By telling when, the adverb "ever" correctly modifies the verb "is," and another adverb, "hardly," appropriately modifies and limits "ever."

Explanation for Incorrect Answer E : There is an error in the sentence.

30

nR eg
(A)

(6) Also, who decided that the person who does your taxes is more important than the person who makes sure that your house is warm or that your car runs ? (7) I know firsthand how frustrating it can be. (8) They think of you only in terms of your job. (9) I used to clean houses in the summer because the money was good; but yet all the people whose houses I cleaned seemed to assume that because I was vacuuming their carpets I did not deserve their respect. (10) One woman came into the bathroom while I was scrubbing the tub. (11) She kept asking me if I had any questions. (12) Did she want me to ask whether to scrub the tub counter-clockwise instead of clockwise ? (13) Her attitude made me angry! (14) Once I read that the jobs people consider important have changed. (15) Carpenters used to be much more admired than doctors. (16) My point is, then, that who I want to be is much more important than what I want to be!

Of the following, which is the best way to phrase sentence 1 (reproduced below) ?

People today have placed emphasis on the kinds of work that others do, it is wrong.

(As it is now)

(B)

People today place too much emphasis on the kinds of work that others do.

(C) What kinds of work others do is being placed too much emphasis on by people today.

(D) The wrong kind of emphasis had been placed on the kinds of work others do today.

(E)

The wrong emphasis is being placed today on people and what kind of work they do.

ANSWERS

AND EXPLANATIONS

Explanation for Correct Answer B : Choice (B) is correct. The phrase "too much" smoothly embeds a negative judgment within the first clause, thereby making the second clause unnecessary.

Explanation for Incorrect Answer A :

is te

re

(1) People today have placed emphasis on the kinds of work that others do, it is wrong. (2) Suppose a woman says she is a doctor. (3) Immediately everyone assumes that she is a wonderful person, as if doctors were incapable of doing wrong. (4) However, if you say youre a carpenter or mechanic, some people think that youre not as smart as a doctor or a lawyer. (5) Cant someone just want to do this because he or she loves the work ?

file://E:\\c5.htm

2006-11-12

The Official SAT Online Course

18/21

Choice (A) is unsatisfactory because it uses a comma improperly to join two independent clauses.

Explanation for Incorrect Answer C : Choice (C) is unsatisfactory because the word order is garbled. The preposition "on" (one of several misplaced words) is far removed from its object, "kinds."

Explanation for Incorrect Answer D : Choice (D) is unsatisfactory because the past perfect tense ("had been placed") is inappropriate to describe an action occurring today.

Explanation for Incorrect Answer E : Choice (E) is unsatisfactory because of wordiness. The sentence does not need both the noun "people" and the pronoun "they."

31

In context, which of the following is the best way to revise and combine the underlined portions of sentences 2 and 3 (reproduced below) ?

Suppose a woman says she is a doctor. Immediately everyone assumes that she is a wonderful person, as if doctors were incapable of doing wrong.
(A)

Suppose a woman says she is a doctor, but immediately

(B)

If a woman says she is a doctor, for instance, immediately

(D) Immediately, if they say, for example, she is a doctor,

(E)

Therefore, a woman is maybe saying she is a doctor; immediately

ANSWERS

AND EXPLANATIONS

32

nR
(A)

Explanation for Incorrect Answer A : Choice (A) is unsatisfactory because the connecting word "but" usually introduces a contrast rather than a result.

Explanation for Incorrect Answer C : Choice (C) is unsatisfactory because the transition word "however" inappropriately suggests that a contrasting idea will follow.

Explanation for Incorrect Answer D : Choice (D) is unsatisfactory because it uses a vague pronoun, "they."

Explanation for Incorrect Answer E : Choice (E) is unsatisfactory because the introductory word "therefore" is inappropriate. It incorrectly suggests that this sentence is drawing a conclusion based on evidence presented earlier.

In context, the phrase do this in sentence 5 would best be replaced by

hold this particular opinion

(B)

resist temptation

(C) ask someone for assistance

(D) become a carpenter or a mechanic

(E)

aspire to learn medicine

eg

Explanation for Correct Answer B : Choice (B) is correct. In the combined sentence, the dependent clause (appropriately introduced by "if") states a possible condition, and the main clause then describes a likely result. "For instance" indicates that the situation illustrates the statement in sentence 1.

is

te

re

(C) When a woman says she is a doctor, however, immediately

file://E:\\c5.htm

2006-11-12

The Official SAT Online Course

19/21

ANSWERS

AND EXPLANATIONS

Explanation for Correct Answer D : Choice (D) is correct. In context, the verb "become" is more precise than "do," and the nouns "carpenter" and "mechanic" are much more specific than the pronoun "this."

Explanation for Incorrect Answer A : Choice (A) is unsatisfactory because its implication is illogical. Holding a negative opinion would not be the likely result of loving one's work.

Explanation for Incorrect Answer B : Choice (B) is unsatisfactory because it introduces an irrelevant idea. No other sentence in the passage even mentions temptation.

Explanation for Incorrect Answer C : Choice (C) is unsatisfactory because the relationship between the two clauses becomes illogical. Loving one's work would not necessarily produce a request for assistance.

33

Which of the following is the best way to revise and combine the underlined portions of sentences 7 and 8 (reproduced below) ?

(A)

be; they--people, that is--think of you

(B)

be when they are thinking of one

(C) be how people think of you

nR
(E)

(D) be when people think of you

be; having people think of you

ANSWERS

Explanation for Correct Answer D : Choice (D) is correct. The subordinating conjunction "when" provides an appropriate link between the two clauses, and the noun "people" replaces the vague pronoun "they."

Explanation for Incorrect Answer A : Choice (A) is unsatisfactory because of wordiness. The pronoun "they" and the phrase "that is" are not needed.

Explanation for Incorrect Answer B : Choice (B) is unsatisfactory because it retains the vague pronoun "they" and introduces another inappropriate pronoun, "one" (that is inconsistent with the later pronoun "your").

Explanation for Incorrect Answer C : Choice (C) is unsatisfactory because "how" is not an acceptable transition word to link the first clause (ending with "be") with the second clause.

Explanation for Incorrect Answer E : Choice (E) is unsatisfactory because it uses improper coordination. The semicolon incorrectly links unequal parts (an independent clause before the semicolon and a phrase after it).

eg is

I know firsthand how frustrating it can be. They think of you only in terms of your job.

AND EXPLANATIONS

te r

ed

Explanation for Incorrect Answer E : Choice (E) is unsatisfactory because it shifts emphasis away from the value of work done by carpenters and mechanics. Instead of continuing the idea in sentence 4, this choice introduces a new thought.

file://E:\\c5.htm

2006-11-12

The Official SAT Online Course

20/21

34

In context, the phrase but yet in sentence 9 would best be replaced by


(A)

incidentally,

(B)

however,

(C) in fact,

(D) in addition,

(E)

for example,

ANSWERS

AND EXPLANATIONS

Explanation for Correct Answer B : Choice (B) is correct. The word "however" properly indicates a contrast between a positive aspect of the job (good pay) and a negative aspect (lack of respect).

Explanation for Incorrect Answer A : Choice (A) is unsatisfactory because "incidentally" does not indicate contrast. Instead, it suggests that the information to follow is of minor importance.

Explanation for Incorrect Answer D : Choice (D) is unsatisfactory because the phrase "in addition" fails to introduce a contrast. It actually implies that the second clause will continue or reinforce the idea presented earlier.

35

nR eg
(A)

Explanation for Incorrect Answer E : Choice (E) is unsatisfactory because the phrase "for example" does not imply contrast. It suggests instead that specific details to follow will support an earlier generalization.

The best place to begin a new paragraph in sentences 6-16 would be with sentence

10

(B)

11

(C) 12

(D) 14

(E)

15

ANSWERS

AND EXPLANATIONS

Explanation for Correct Answer D : Choice (D) is correct. The account of the writer's own experience as a house cleaner ends with sentence 13, and sentence 14 appropriately begins a new paragraph about historical changes in attitudes toward different kinds of work.

Explanation for Incorrect Answer A : Choice (A) is unsatisfactory because such a division would improperly separate a specific example (starting in sentence 10) from the general statement it illustrates (in sentence 9).

Explanation for Incorrect Answer B :

is te

re

Explanation for Incorrect Answer C : Choice (C) is unsatisfactory because the phrase "in fact" does not prepare for a contrast. It implies only that the second clause will correct or clarify an earlier misconception.

file://E:\\c5.htm

2006-11-12

The Official SAT Online Course


Choice (B) is unsatisfactory because this break would awkwardly interrupt the narrative beginning in sentence 10 and continuing through sentence 13.

21/21

Explanation for Incorrect Answer C : Choice (C) is unsatisfactory because such an interruption in the narrative (running from sentence 10 through sentence 13) would be inappropriate.

Explanation for Incorrect Answer E : Choice (E) is unsatisfactory because the resulting separation of a specific example (in sentence 15) from the generalization that it supports (in sentence 14) would be improper.

Back to Score Report

Copyright 2006 The College Board. All rights reserved.

Privacy Policy

Terms of Use

Contact Us

nR

eg

is t

er
file://E:\\c5.htm

ed
2006-11-12

The Official SAT Online Course

1/22

Help | Profile | My Organizer | My Bookmarks | Logout

Answers and Explanations

Test Sections

Back to Score Report

Section 1

View Answers and Explanations


Online - Practice Test #3

Section 2

Section 3

Section 4

Section 5

Section 7

Much of our knowledge of dinosaurs comes from excavated bones, which, in ------other clues such as fossilized tracks and eggs, help us to ------- the evolution of these creatures.
(A)

Section 8

convergence with. . supplant

Section 9

(B)

divergence from. . decode

Section 10

(C) dependence on. . belie

(D) opposition to. . amplify

(E)

conjunction with. . trace

Explanation for Incorrect Answer A : Choice (A) is incorrect. "Convergence with" means come together with. "Supplant" means replace. If one were to insert these terms into the text, the sentence would read "Much of our knowledge of dinosaurs comes from excavated bones, which, in convergence with other clues such as fossilized tracks and eggs, help us to supplant the evolution of these creatures." The first missing term suggests that knowledge and clues come together. The second missing term indicates what the knowledge and clues enable people to do. Supplant does not make sense in the sentence because evolution cannot be replaced.

nR

Explanation for Incorrect Answer B : Choice (B) is incorrect. "Divergence from" means departing from. "Decode" means to put a coded message into plain language. If one were to insert these terms into the text, the sentence would read "Much of our knowledge of dinosaurs comes from excavated bones, which, in divergence from other clues such as fossilized tracks and eggs, help us to decode the evolution of these creatures." The first missing term suggests that knowledge and clues come together. The second missing term indicates what the knowledge and clues enable people to do. Knowledge and clues would enable people to decode how dinosaurs evolved However, "divergence from" does not suggest that the knowledge and clues are coming together.

Explanation for Incorrect Answer C : Choice (C) is incorrect. "Dependence on" means leaning on. "Belie" means contradict or fail to confirm. If one were to insert these terms into the text, the sentence would read "Much of our knowledge of dinosaurs comes from excavated bones, which, in dependence on other clues such as fossilized tracks and eggs, help us to belie the evolution of these creatures." The first missing term suggests that knowledge and clues come together. The second missing term indicates what the knowledge and clues enable people to do. Bones themselves would not depend on clues. Additionally, while knowledge and clues might belie, or contradict, evolutionary theories, they do not belie evolution itself.

Explanation for Incorrect Answer D : Choice (D) is incorrect. "Opposition to" means against. "Amplify" means make

eg

is te

Explanation for Correct Answer E : Choice (E) is correct. "Conjunction with" means together with. "Trace" means track or follow. If one were to insert these terms into the text, the sentence would read "Much of our knowledge of dinosaurs comes from excavated bones, which, in conjunction with other clues such as fossilized tracks and eggs, help us to trace the evolution of these creatures." The first missing term suggests that knowledge and clues come together. The second missing term indicates what the knowledge and clues enable people to do. It makes sense to say that knowledge and clues work together to enable people to trace the evolution of dinosaurs.

re

d
2006-11-12

ANSWERS

AND EXPLANATIONS

file://E:\\c6.htm

The Official SAT Online Course

2/22

larger or greater. If one were to insert these terms into the text, the sentence would read "Much of our knowledge of dinosaurs comes from excavated bones, which, in opposition to other clues such as fossilized tracks and eggs, help us to amplify the evolution of these creatures." The first missing term suggests that knowledge and clues come together. The second missing term indicates what the knowledge and clues enable people to do. People cannot amplify the evolution of creatures.

Vernal pools are among the most ------- of ponds: they form as a result of snowmelt and a high water table in winter, and then they ------- by late summer.
(A)

transitory . . expand

(B)

anachronistic . . overflow

(C) immutable . . drain

(D) itinerant . . teem

(E)

ephemeral . . evaporate

ANSWERS

AND EXPLANATIONS

nR

Explanation for Incorrect Answer A : Choice (A) is incorrect. "Transitory" means temporary or brief. "Expand" means increase in size. If one were to insert these terms into the text, the sentence would read "Vernal pools are among the most transitory of ponds: they form as a result of snowmelt and a high water table in winter, and then they expand by late summer." "Vernal pools" are spring pools. The first missing term is a characteristic of a vernal pool. What follows the colon is an explanation of the first missing term. A transitory pond would disappear after a brief time. However, ponds that expand are not temporary.

Explanation for Incorrect Answer B : Choice (B) is incorrect. "Anachronistic" means from the wrong time period. "Overflow" means flow over the limits of something. If one were to insert these terms into the text, the sentence would read "Vernal pools are among the most anachronistic of ponds: they form as a result of snowmelt and a high water table in winter, and then they overflow by late summer." "Vernal pools" are spring pools. The first missing term is a characteristic of a vernal pool. What follows the colon is an explanation of the first missing term. A pond cannot be anachronistic.

Explanation for Incorrect Answer C : Choice (C) is incorrect. "Immutable" means unchangeable. "Drain" means flow away. If one were to insert these terms into the text, the sentence would read "Vernal pools are among the most immutable of ponds: they form as a result of snowmelt and a high water table in winter, and then they drain by late summer." "Vernal pools" are spring pools. The first missing term is a characteristic of a vernal pool. What follows the colon is an explanation of the first missing term. A pond that is immutable would not fill in winter and drain in summer.

Explanation for Incorrect Answer D : Choice (D) is incorrect. "Itinerant" means roaming or wandering from place to place. "Teem" means become filled to overflowing. If one were to insert these terms into the text, the sentence would read "Vernal pools are among the most itinerant of ponds: they form as a result of snowmelt and a high water table in winter, and then they teem by late summer." "Vernal pools" are spring pools. The first missing term is a characteristic of a vernal pool. What follows the colon is an explanation of the first missing term. Itinerant is not described by the concept of teeming. As well, a pond stays in the same place; it cannot be itinerant.

eg

is

te

re d

Explanation for Correct Answer E : Choice (E) is correct. "Ephemeral" means brief. "Evaporate" means disappear by turning into vapor. If one were to insert these terms into the text, the sentence would read "Vernal pools are among the most ephemeral of ponds: they form as a result of snowmelt and a high water table in winter, and then they evaporate by late summer." "Vernal pools" are spring pools. The first missing term is a characteristic of a vernal pool. What follows the colon is an explanation of the first missing term. Evaporation shows that the ponds disappear after a period of time. This corresponds with the use of ephemeral to describe a vernal pool.

file://E:\\c6.htm

2006-11-12

The Official SAT Online Course

3/22

The ------- experiences of Madonna Swan, the 1983 North American Indian Woman of the Year, cannot be fully appreciated if they are ------- in a tidy summary.
(A)

varied . . interposed

(B)

diverse . . condensed

(C) profound . . magnified

(D) transformative . . embellished

(E)

impressive . . immersed

ANSWERS

AND EXPLANATIONS

Explanation for Correct Answer B : Choice (B) is correct. "Diverse" means composed of different elements, and "condensed" means made compact. If one were to insert these terms into the text, the sentence would read "The diverse experiences of Madonna Swan, the 1983 North American Indian Woman of the Year, cannot be fully appreciated if they are condensed in a tidy summary." Experiences combining different elements cannot be easily compacted in a tidy summary.

Explanation for Incorrect Answer C : Choice (C) is incorrect. "Profound" means deep. "Magnified" means enlarged. If one were to insert these terms into the text, the sentence would read "The profound experiences of Madonna Swan, the 1983 North American Indian Woman of the Year, cannot be fully appreciated if they are magnified in a tidy summary." While deeply meaningful experiences may be summarized, a summary is by definition a brief version of something. It makes no sense to say that something could be enlarged in a summary.

nR
(A)

Explanation for Incorrect Answer D : Choice (D) is incorrect. "Transformative" means causing a change. "Embellished" means enhanced. If one were to insert these terms into the text, the sentence would read "The transformative experiences of Madonna Swan, the 1983 North American Indian Woman of the Year, cannot be fully appreciated if they are embellished in a tidy summary." A summary presents main points in a general way; it does not "embellish," or enhance, those points.

Explanation for Incorrect Answer E : Choice (E) is incorrect. "Impressive" means inspiring awe or admiration, and "immersed" means submerged. If one were to insert these terms into the text, the sentence would read "The impressive experiences of Madonna Swan, the 1983 North American Indian Woman of the Year, cannot be fully appreciated if they are immersed in a tidy summary." Although impressive experiences might be mentioned in a summary, it would be strange to describe such experiences as being submerged.

The representative was a traditionalist, reluctant to support any legislation inconsistent with the nations most ------- principles.

orthodox

(B)

impassioned

(C) precarious

(D) impressionable

(E)

indeterminate

eg is te

Explanation for Incorrect Answer A : Choice (A) is incorrect. "Varied" means mixed, and "interposed" means inserted. If one were to insert these terms into the text, the sentence would read "The varied experiences of Madonna Swan, the 1983 North American Indian Woman of the Year, cannot be fully appreciated if they are interposed in a tidy summary." While varied experiences may, with difficulty, be summarized, it does not make sense to speak of experiences being inserted into a summary. That would suggest that the summary of experiences already exists.

re

file://E:\\c6.htm

2006-11-12

The Official SAT Online Course

4/22

ANSWERS

AND EXPLANATIONS

Explanation for Correct Answer A : Choice (A) is correct. "Orthodox" means conforming to traditional beliefs. If one were to insert this term into the text, the sentence would read "The representative was a traditionalist, reluctant to support any legislation inconsistent with the nation's most orthodox principles." The missing term describes the kind of legislative principles that a "traditionalist," a person who upholds traditional beliefs, would support. Orthodox principles are exactly the type of principles a traditionalist would tend to support.

Explanation for Incorrect Answer B : Choice (B) is incorrect. "Impassioned" means passionate. If one were to insert this term into the text, the sentence would read "The representative was a traditionalist, reluctant to support any legislation inconsistent with the nation's most impassioned principles." The missing term describes the kind of legislative principles that a "traditionalist," a person who upholds traditional beliefs, would support. It is odd to say that principles themselves could be impassioned. This word is usually used to describe people, not things such as principles. In any case, impassioned principles would not necessarily be in line with tradition.

Explanation for Incorrect Answer E : Choice (E) is incorrect. "Indeterminate" means vague. If one were to insert this term into the text, the sentence would read "The representative was a traditionalist, reluctant to support any legislation inconsistent with the nation's most indeterminate principles." The missing term describes the kind of legislative principles that a "traditionalist," a person who upholds traditional beliefs, would support. Although principles could be described as indeterminate, a traditionalist may or may not be likely to support them.

nR
(A)

The author constructed a scenario in which playful, creative children are rewarded for their ------- and strict, dour adults are punished for their -------.

spontaneity . . rigidity

(B)

digressions . . mirth

(C) solemnity . . malice

(D) inflexibility . . rigor

(E)

improvisations . . buoyancy

ANSWERS

Explanation for Correct Answer A : Choice (A) is correct. "Spontaneity" is uncontrolled and impulsive behavior. "Rigidity" means stiffness. If one were to insert these terms into the text, the sentence would read "The author constructed a scenario in which playful, creative children are rewarded for their spontaneity and strict, dour adults are punished for their rigidity." The first missing term should be associated with creativity or playfulness, and the second should be related to "strict" and "dour," which mean limiting and stern. Spontaneity is often associated with creativity, and someone who is strict or unyielding could aptly be described as exhibiting rigidity.

eg

Explanation for Incorrect Answer D : Choice (D) is incorrect. "Impressionable" means capable of being easily impressed or swayed. If one were to insert this term into the text, the sentence would read "The representative was a traditionalist, reluctant to support any legislation inconsistent with the nation's most impressionable principles." The missing term describes the kind of legislative principles that a "traditionalist," a person who upholds traditional beliefs, would support. Impressionable is a word that applies to people, not to things such as principles.

AND EXPLANATIONS

is

te

re

Explanation for Incorrect Answer C : Choice (C) is incorrect. "Precarious" means on edge or unsafe. If one were to insert this term into the text, the sentence would read "The representative was a traditionalist, reluctant to support any legislation inconsistent with the nation's most precarious principles." The missing term describes the kind of legislative principles that a "traditionalist," a person who upholds traditional beliefs, would support. A traditionalist may or may not support principles that are precarious.

file://E:\\c6.htm

2006-11-12

The Official SAT Online Course

5/22

Explanation for Incorrect Answer B : Choice (B) is incorrect. "Digressions" are wanderings. "Mirth" is gladness. If one were to insert these terms into the text, the sentence would read "The author constructed a scenario in which playful, creative children are rewarded for their digressions and strict, dour adults are punished for their mirth." The first missing term should be associated with creativity or playfulness, and the second should be related to "strict" and "dour," which mean limiting and stern. Mirth does not work in the sentence because it is not a quality of strict, dour people.

Explanation for Incorrect Answer C : Choice (C) is incorrect. "Solemnity" means gloomy seriousness. "Malice" means hatred. If one were to insert these terms into the text, the sentence would read "The author constructed a scenario in which playful, creative children are rewarded for their solemnity and strict, dour adults are punished for their malice." The first missing term should be associated with creativity or playfulness, and the second should be related to "strict" and "dour," which mean limiting and stern. Solemnity is the opposite of playfulness, and thus could not be the answer.

nR
(A)

Although usually warm and ------- in greeting friends, Lauren was too reserved ever to be truly -------.

joyous . . conventional

(B)

cordial . . effusive

(C) restrained . . gracious

(D) dismissive . . ebullient

(E)

genial . . antisocial

ANSWERS

Explanation for Correct Answer B : Choice (B) is correct. "Cordial" means warm and friendly. "Effusive" means unrestrained in emotional expression. If one were to insert these terms into the text, the sentence would read "Although usually warm and cordial in greeting friends, Lauren was too reserved ever to be truly effusive." The first missing term is similar in meaning to "warm," which, in this instance, means affectionate. The second term should have a meaning that is the opposite of "reserved," which means restrained or shy. "Cordial" means socially warm and "effusive" is the opposite of reserved.

Explanation for Incorrect Answer A : Choice (A) is incorrect. "Joyous" means happy. "Conventional" means lacking originality or individuality. If one were to insert these terms into the text, the sentence would read "Although usually warm and joyous in greeting friends, Lauren was too reserved ever to be truly conventional." The first missing term is similar in

eg
AND EXPLANATIONS

is te

Explanation for Incorrect Answer E : Choice (E) is incorrect. "Improvisations" are things created without planning. "Buoyancy" means the state of being light or cheerful. If one were to insert these terms into the text, the sentence would read "The author constructed a scenario in which playful, creative children are rewarded for their improvisations and strict, dour adults are punished for their buoyancy." The first missing term should be associated with creativity or playfulness, and the second should be related "strict" and "dour," which mean limiting and stern. Although improvisations works in the sentence, the second word does not. Buoyancy is not associated with strict, dour people.

re

Explanation for Incorrect Answer D : Choice (D) is incorrect. "Inflexibility" means the state of being rigid or unyielding. "Rigor" means strictness. If one were to insert these terms into the text, the sentence would read "The author constructed a scenario in which playful, creative children are rewarded for their inflexibility and strict, dour adults are punished for their rigor." The first missing term should be associated with creativity or playfulness, and the second should be related to "strict" and "dour," which mean limiting and stern. Playful, creative children would most likely not be inflexible.

file://E:\\c6.htm

2006-11-12

The Official SAT Online Course

6/22

meaning to "warm," which, in this instance, means affectionate. The second word should have a meaning that is the opposite of "reserved," which means restrained or shy. "Conventional" is not the opposite of reserved.

Explanation for Incorrect Answer C : Choice (C) is incorrect. "Restrained" means kept under control. "Gracious" means friendly. If one were to insert these terms into the text, the sentence would read "Although usually warm and restrained in greeting friends, Lauren was too reserved ever to be truly gracious." The first missing term is similar in meaning to "warm," which, in this instance, means affectionate. The second word should have a meaning that is the opposite of "reserved," which means restrained or shy. The word "restrained" does not suggest affection.

Explanation for Incorrect Answer D : Choice (D) is incorrect. "Dismissive" describes an attitude of offhand rejection. "Ebullient" means full of high spirits. If one were to insert these terms into the text, the sentence would read "Although usually warm and dismissive in greeting friends, Lauren was too reserved ever to be truly ebullient." The first missing term is similar in meaning to "warm," which, in this instance, means affectionate. The second word should have a meaning that is the opposite of "reserved," which means restrained or shy. Lauren would not be both warm and dismissive when she greets friends, since the these are two opposite attitudes.

Legal scholars argue that when justice is interpreted too broadly, the concept becomes -------, easily changed and controlled by outside forces.
(A)

malleable

(C) coherent

(D) felicitous

nR
(E)

prosaic

ANSWERS

Explanation for Correct Answer A : Choice (A) is correct. "Malleable" means capable of being changed or controlled. If one were to insert this term into the text, the sentence would read "Legal scholars argue that when 'justice' is interpreted too broadly, the concept becomes malleable, easily changed and controlled by outside forces." "Malleable" properly describes a "justice" that can be altered by outside forces.

Explanation for Incorrect Answer B : Choice (B) is incorrect. "Influential" means powerful or having an effect on others. If one were to insert this term into the text, the sentence would read "Legal scholars argue that when 'justice' is interpreted too broadly, the concept becomes influential, easily changed and controlled by outside forces." An "influential" concept is one that changes outside forces, not one that is changed by outside forces.

Explanation for Incorrect Answer C : Choice (C) is incorrect. "Coherent" means well ordered or clear. If one were to insert this term into the text, the sentence would read "Legal scholars argue that when 'justice' is interpreted too broadly, the concept becomes coherent, easily changed and controlled by outside forces." It makes no sense to imply that a concept can be easily changed or controlled because it is clear.

Explanation for Incorrect Answer D : Choice (D) is incorrect. "Felicitous" means well suited or particularly appropriate to a situation. If one were to insert this term into the text, the sentence would read

eg
AND EXPLANATIONS

(B)

influential

is te re d

Explanation for Incorrect Answer E : Choice (E) is incorrect. "Genial" means cheerful. "Antisocial" means unsociable. If one were to insert these terms into the text, the sentence would read "Although usually warm and cordial in greeting friends, Lauren was too reserved ever to be truly effusive." The first missing term is similar in meaning to "warm," which, in this instance, means affectionate. The second word should have a meaning that is the opposite of "reserved," which means restrained or shy. One would not say that a person is too reserved to be antisocial. An extremely reserved person might be considered antisocial.

file://E:\\c6.htm

2006-11-12

The Official SAT Online Course

7/22

"Legal scholars argue that when 'justice' is interpreted too broadly, the concept becomes felicitous, easily changed and controlled by outside forces." If something is well suited, it makes no sense to describe it as easily changed or controlled.

Explanation for Incorrect Answer E : Choice (E) is incorrect. "Prosaic" means ordinary. If one were to insert this term into the text, the sentence would read "Legal scholars argue that when 'justice' is interpreted too broadly, the concept becomes prosaic, easily changed and controlled by outside forces." An ordinary concept might be a concept that is easily changed or controlled, but it is not necessarily so.

The instructors voice was so ------- that most students preferred taking a test over listening to its grating sound.
(A)

receptive

(B)

cajoling

(C) melodious

(D) muted

(E)

strident

ANSWERS

AND EXPLANATIONS

nR

Explanation for Incorrect Answer A : Choice (A) is incorrect. "Receptive" means open and responsive to ideas or suggestions. If one were to insert this term into the text, the sentence would read "The instructor's voice was so receptive that most students preferred taking a test over listening to its grating sound." The missing term describes a voice that has a "grating sound." A grating sound has nothing to do with being receptive. Furthermore, no voice could be described as receptive.

Explanation for Incorrect Answer B : Choice (B) is incorrect. "Cajoling" means persuading or coaxing. If one were to insert this term into the text, the sentence would read "The instructor's voice was so cajoling that most students preferred taking a test over listening to its grating sound." The missing term describes a voice that has a "grating sound." A cajoling sound is not synonymous with a grating sound. A person who is cajoling would most likely use a soft voice, not a grating one.

Explanation for Incorrect Answer C : Choice (C) is incorrect. "Melodious" means sweet-sounding or full or melody. If one were to insert this term into the text, the sentence would read "The instructor's voice was so melodious that most students preferred taking a test over listening to its grating sound." The missing term describes a voice that has a "grating sound." A melodious voice is the opposite of a grating one.

Explanation for Incorrect Answer D : Choice (D) is incorrect. "Muted" means muffled or softened. If one were to insert this term into the text, the sentence would read "The instructor's voice was so muted that most students preferred taking a test over listening to its grating sound." The missing term describes a voice that has a "grating sound." A muted voice is the opposite of a grating voice.

The big doors of the hotel are padlocked. So far nobody

has smashed their glass panels. Nobody could


file://E:\\c6.htm

eg i

st e

re

Explanation for Correct Answer E : Choice (E) is correct. "Strident" means loud, harsh and discordant. If one were to insert this term into the text, the sentence would read "The instructor's voice was so strident that most students preferred taking a test over listening to its grating sound." The missing term describes a voice that has a "grating sound." A strident sound is synonymous with a grating sound.

2006-11-12

The Official SAT Online Course

8/22

The passage is characterized by all of the following EXCEPT

(B)

auditory descriptions

(C) contrast

(E)

hypothetical musings

ANSWERS

10

nR
(A)

Explanation for Correct Answer D : Choice (D) is correct. The passage does not appeal to reason but to emotion and to the senses. It describes the feelings and impressions associated with the physical appearance of the hotel.

Explanation for Incorrect Answer A : Choice (A) is incorrect. The passage does use visual imagery, such as the descriptions of the padlocked doors and the reflections in their glass panels.

Explanation for Incorrect Answer B : Choice (B) is incorrect. The sounds of the shutter hinge and the wind are auditory descriptions, or descriptions of sound, that are compared to music.

Explanation for Incorrect Answer C : Choice (C) is incorrect. The lonely, vast land outside the hotel is contrasted with the happy crowd of "best friends" that the hotel "seems to promise you."

Explanation for Incorrect Answer E : Choice (E) is incorrect. Through hypothetical musings, or imaginative reflections, the author describes the empty, padlocked hotel as if it were the scene of a party.

Lines 10-12 (piano keys . . . rooms) convey a feeling of

lightheartedness

(B)

bewilderment

eg is

(D) an appeal to reason

AND EXPLANATIONS

te r
2006-11-12

(A)

visual imagery

file://E:\\c6.htm

ed

stand to do it because the panels mirror your own face as well as the view behind your back: acres of chive grass Line edging the sparkly beach, a movie-screen sky, and an 5 ocean that wants you more than anything. No matter the outside loneliness, if you look inside, the hotel seems to promise you ecstasy and the company of all your best friends. And music. The shift of a shutter hinge sounds like the cough of a trumpet; piano keys waver a quarter note above the 10 wind so you might miss the hurt jamming those halls and closed-up rooms.

The Official SAT Online Course

9/22

(C) melancholy

(D) nostalgia

(E)

detachment

ANSWERS

AND EXPLANATIONS

Explanation for Correct Answer C : Choice (C) is correct. These lines convey a feeling of melancholy, or sadness, because of the "hurt" that haunts the imaginary cheerfulness of the inside of the hotel.

Explanation for Incorrect Answer A : Choice (A) is incorrect. Earlier in the passage, the hotel's inside was imagined as lighthearted and happy, but this is contradicted by the "hurt jamming," or pain pressing, inside the "halls" and "closed-up rooms" of the hotel.

Explanation for Incorrect Answer B : Choice (B) is incorrect. Bewilderment, or confusion, is not conveyed in the description of the hotel's sadness. Rather, there is a degree of clarity expressed in the recognition of the sadness that fills the hotel.

Explanation for Incorrect Answer E : Choice (E) is incorrect. There is no feeling of detachment, or lack of involvement, in the emotionally charged description of the "hurt" the author imagines "jamming those halls and closed-up rooms."

Diffusion theory is an umbrella idea encompassing various alternative theories of Americas discovery by explorers from other parts of the world. Columbus (and Leif Ericsson and Zheng He) had a lot more Line competitors than most people think: Prince Madoc of Wales, 5 the Zeni brothers of Venice, Corte Real of Portugal, Polands Jan of Kolno. The fact is, crossing the Atlantic was probably not as big a deal as Columbus-centric historians thought. Diffusionists may not be able to pinpoint who beat Columbus to the punch, yet theyre sure 10 someone did. They may well be right, but if you scrutinize any specific claim, it melts away. This is probably why diffusionists

nR

eg is

te re

Explanation for Incorrect Answer D : Choice (D) is incorrect. Even though nostalgia, or a longing for the past, is present elsewhere in the passage, there is no longing for the hotel's "hurt" and sadness expressed in lines 10-12.

file://E:\\c6.htm

2006-11-12

The Official SAT Online Course

10/22

emphasize quantity over quality.


11

In line 1, umbrella is used to convey which of the following qualities?


(A)

Comprehensiveness

(B)

Impenetrability

(C) Utility

(D) Ordinariness

(E)

Foresight

ANSWERS

AND EXPLANATIONS

Explanation for Correct Answer A : Choice (A) is correct. "Comprehensiveness" is the quality of being all-inclusive and wide-ranging, covering many ideasjust as an umbrella might cover, or stretch, over several objects. In this context, "umbrella" refers to "diffusion theory," which covers or includes within itself "various alternative theories" (line 2) about the discovery of America.

12

nR
(A)

Explanation for Incorrect Answer E : Choice (E) is incorrect. The passage explains that "diffusion theory" includes many alternative theories. The notion of foresight is not discussed in the passage.

The strategy employed by the diffusionists in making their claim is most evident in which assertion?

Since nothing can be proven with absolute certainty, we ultimately rely on faith.

(B)

Before the Wright brothers success, experts thought that craft that were heavier than air could not fly.

(C) So many UFO sightings have been reported that at least one of them must be authentic.

(D) Penicillin, like many other discoveries, was stumbled on by accident.

(E)

Although folk medicine was at first derided by the medical establishment, people still relied on it.

ANSWERS

Explanation for Correct Answer C : Choice (C) is correct. This assertion uses the great quantity of UFO sightings as evidence that at least one of them must be authentic. The quantity of the sightings, or the number of times people claim to see UFOs, is emphasized over the quality of the sightings (a high-quality sighting is one that could be verified by independent experts). This is the same strategy that diffusionists use; diffusionists also "emphasize quantity over quality" (line 13).

eg

Explanation for Incorrect Answer D : Choice (D) is incorrect. Even though an umbrella is an ordinary object, the passage does not refer to the ordinariness of the "diffusion theory," but to its comprehensiveness.

AND EXPLANATIONS

is

Explanation for Incorrect Answer C : Choice (C) is incorrect. The utility, or usefulness, of an umbrella is not the quality featured in the passage. The passage uses the term "umbrella" to refer to the fact that the "diffusion theory" comprises "various alternative theories" (line 2).

te

re

Explanation for Incorrect Answer B : Choice (B) is incorrect. "Impenetrable" refers to something that cannot be penetrated. It makes sense that an umbrella would be described as "impenetrable" by rain, but the theories mentioned in the passage have been penetrated, or shown to be flawed, be researchers: "if you scrutinize any specific claim, it melts away" (lines 11-12). Thus they cannot be described as "impenetrable."

file://E:\\c6.htm

2006-11-12

The Official SAT Online Course

11/22

Explanation for Incorrect Answer A : Choice (A) is incorrect. Although diffusionists cannot at this point say which explorer beat Columbus, diffusionists do not conclude that proof is impossible or irrelevant.

Explanation for Incorrect Answer B : Choice (B) is incorrect. The Wright Brothers decisively disproved a theory, whereas diffusion theory has to do with ideas that have not been either proven or disproven.

Explanation for Incorrect Answer D : Choice (D) is incorrect. Diffusionists may embrace theories that claim America was discovered by accident, but this is not mentioned in the passage.

Explanation for Incorrect Answer E : Choice (E) is incorrect. The passage does not describe diffusionists as being concerned with rivalries between new technologies and old ones.

Is a persons gender an important influence on how he or she behaves with others? Contemporary sociologists and other scholars have argued this question fiercely. The following pair of passages presents two contrasting voices from that debate.

Passage 1

The desire to affirm that women and men are completely equal has made some scholars reluctant to show ways in which they are different, because differences between two groups of people have so often Line been used to justify unequal treatment and 5 opportunity. Much as I understand and am in sympathy with those who wish there were no differences between women and menonly reparable social injusticemy research on styles of conversation tells me that, at least in this area, it simply isnt so. I believe that there are 10 gender differences in ways of speaking, and we need to identify and understand them. Without such understanding, we are doomed to blame others or ourselvesor our own relationshipsfor the otherwise mystifying and damaging effects of our contrasting conversational 15 styles. It is clear to me that recognizing gender differences in conversational styles would free individuals

nR eg

is t

er

ed

file://E:\\c6.htm

2006-11-12

The Official SAT Online Course

12/22

20

25

30

35

40

from the burden of an inappropriate sense of being at fault for chronic disagreements. Many women and men feel dissatisfied with their close relationshipswith spouses, siblings, parentsand become even more frustrated when they try to talk things out. Taking a sociolinguistic approach to such troubling encounters makes it possible to explain these dissatisfactions without accusing anyone of being wrong and without blamingor discarding the relationship. The sociolinguistic approach I take in my work is based on my belief that many frictions arise because, here in the United States, boys and girls grow up in what are essentially different cultures, so that talk between women and men is actually cross-cultural communication. For little boys, talk is primarily a means of making statements of achievement through games like bragging contests. This may also be done by exhibiting knowledge or skill and by holding center stage through such verbal performance as storytelling, joking, or imparting information. Little girls appear to be eager to share and compare interests and ideas. Emphasis is placed on displaying similarities and matching experiences. For them, the language of conversation is primarily a language of rapport: a way of establishing connection and negotiating relationships. So this view of childrens behavior predicts that more women than men will be comfortable speaking one-on-one, to individuals.

nR

eg is t

er

ed

file://E:\\c6.htm

2006-11-12

The Official SAT Online Course

13/22

And even when addressing an audience, women may be more concerned than men with establishing 45 rapport.
Passage 2

50

60

65

nR eg

is

55

Gender stereotypes should concern us for several reasons. First, they may dictate what we notice and bias our perceptions in the direction of expectation. Some researchers attempt to elucidate gender differences in order to help women and men understand and respond to one another better. In the process, however, their work encourages people to notice and attend to differences rather than similarities, to perceive men and women in accordance with stereotypes that may not accurately depict their behavior or intentions. Second, gender stereotypes may not only describe behavior but also prescribe it, dictating how men and women should behave. People begin to act in ways that support other peoples gender-role expectations of them. It is time to rethink our understanding of gender, to move away from the notion that men and women have two contrasting styles of interaction that were acquired in childhood. We need to move from a conceptualization of gender as an attribute or style of behavior to an understanding of gender as something people do in social interaction. As a noted scholar proposes, None of us is feminine or is masculine or fails to be either of those. In particular contexts people do feminine, in others, they do masculine. People display contradictory

te r

ed

file://E:\\c6.htm

2006-11-12

The Official SAT Online Course

14/22

70

75

85

90

13

nR
(A)

The two passages differ most on which topic?

Whether boys and girls communicate in gender specific patterns

(B)

Whether important social behavior is learned in childhood

(C) Whether adult conversational styles can be studied systematically

(D) Whether gender plays a role in determining a childs playtime activities

eg

is te

re

80

behaviors as they encounter different social norms and pressures. Some researchers view male-female conversations as cross-cultural communication. The two-cultures approach postulates that difficulties in communication between men and women arise because of a clash of conversational styles. But this approach has a number of limitations. First, the coherence of male and female subcultures in childhood has been exaggerated. We arrive at a contrasting picture of the cultures of boys and girls only by singling out those children who fit common gender stereotypes and marginalizing others. We fail to notice the children who do not fit those stereotypesfor example, boys who excel at caring for younger siblings or girls who enjoy building things in shop class. Second, although children may choose samesex playmates as preferred partners, they interact daily inside and outside school with the opposite sex. Children have countless experiences communicating with people of both sexes: they do not learn to communicate in gendersegregated worlds. They learn to display different styles of interaction in different contexts: they do not learn a single gender-related style. The same child may display dominance and give orders to a younger playmate but show deference and follow orders from an older friend.

file://E:\\c6.htm

2006-11-12

The Official SAT Online Course

15/22

(E)

Whether society concerns itself with the concept of gender roles

ANSWERS

AND EXPLANATIONS

Explanation for Correct Answer A : Choice (A) is correct. The passages differ substantially on the question of whether gender determines how individuals communicate with each other. In Passage 1, the author states, "I believe that there are gender differences in ways of speaking, and we need to identify and understand them" (lines 10-12). The author of Passage 2 argues that "this approach has a number of limitations" (line 75), and stresses that individuals need to move toward "an understanding of gender as something people do in social interaction" (lines 64-66). Passage 2 sees gender as a fluid component of social interactions that anyone can adopt depending on the situation, whereas Passage 1 sees gender as a fixed determinant of communication styles.

Explanation for Incorrect Answer B : Choice (B) is incorrect. Both passages give examples of social behaviors in childhood, and both assume that these behaviors are at least partly learned.

Explanation for Incorrect Answer C : Choice (C) is incorrect. Passage 1 implies that a study of conversational styles between adults is possible and desirable, but Passage 2 does not address this topic.

14

nR eg
(A)

The primary purpose of Passage 1 is to

present a historical overview of a controversy

(B)

acknowledge previous errors in thinking

(C) urge changes in organized activities provided for children

(D) assert the value of a particular approach to an issue

(E)

downplay the significance of a recent discovery

ANSWERS

AND EXPLANATIONS

Explanation for Correct Answer D : Choice (D) is correct. Passage 1's argument is that gender-specific communication differences account for misunderstandings between men and women. The author asserts the value of the sociolinguistic approach to the issue of gender differences, arguing that it is an approach that should improve understanding between men and women.

Explanation for Incorrect Answer A : Choice (A) is incorrect. Passage 1 does not present the history of the controversy over gender differences, but rather explains how a sociolinguistic approach to the problem will help people deal with these differences.

Explanation for Incorrect Answer B : Choice (B) is incorrect. Passage 1 does acknowledge that some scholars are "reluctant," or unwilling, to show ways in which men and women are different, but the passage is not primarily concerned with the errors of others.

Explanation for Incorrect Answer C :

is

Explanation for Incorrect Answer E : Choice (E) is incorrect. The authors both agree that society is very interested in gender roles.

te r

Explanation for Incorrect Answer D : Choice (D) is incorrect. Passage 1 discusses the role of gender in shaping how children approach activities such as conversation; Passage 2 does not deny that gender is a factor in children's lives, but suggests that the situation is more complex than sociolinguists claim. The passages do not primarily address children's playtime activities.

ed

file://E:\\c6.htm

2006-11-12

The Official SAT Online Course

16/22

Choice (C) is incorrect. Passage 1 does not suggest changes in children's activities, but rather describes how the different cultures of boys and girls lead them to communicate differently.

Explanation for Incorrect Answer E : Choice (E) is incorrect. Passage 1 does not discuss any recent discovery.

15

Passage 1 makes which suggestion about the work of some scholars (line 2)?
(A)

It will become widely accepted in the scientific community.

(B)

It is well intentioned but misguided.

(C) It attempts to be objective but does not succeed.

(D) It puts forth a convincing theory.

(E)

It could be used to excuse injustice in a society.

ANSWERS

AND EXPLANATIONS

16

nR
(A)

Explanation for Incorrect Answer C : Choice (C) is incorrect. Passage 1 does not suggest that the scholars are objective, or unbiased, about gender differences; rather, it implies that their "desire to affirm that women and men are completely equal" (lines 1-2) has affected their observations of people's actual behavior.

Explanation for Incorrect Answer D : Choice (D) is incorrect. The author of Passage 1 firmly disagrees with the theory that there is no difference between women and men.

Explanation for Incorrect Answer E : Choice (E) is incorrect. Passage 1 suggests the opposite: that the scholars' work attempts to cure social injustice by denying differences between men and women. There is no sense that the scholars' work might be used to excuse injustice.

Passage 1 argues that recognizing gender differences (line 16) would most likely

cause people to exaggerate their similarities when communicating with one another

(B)

lead to further dissatisfaction in conversations among friends and relatives

(C) promote the equal treatment of distinct social groups

(D) relieve individuals of much of the blame for problems in relationships

(E)

affect the way that future research on gender is conducted

ANSWERS

Explanation for Correct Answer D : Choice (D) is correct. Passage 1's approach to gender difference is concerned with men's and women's close relationships with spouses, siblings and friends. According to the passage, "recognizing gender differences in conversational styles would free individuals from the burden of an inappropriate sense of being at fault" (lines 1618). In other words, this recognition would keep people from blaming themselves

eg

AND EXPLANATIONS

is

Explanation for Incorrect Answer A : Choice (A) is incorrect. Far from predicting that the scholars' work will eventually be accepted, Passage 1 discounts the belief that there are no differences between women and men by saying, "it simply isn't so" (line 10).

te

re
file://E:\\c6.htm

Explanation for Correct Answer B : Choice (B) is correct. Passage 1 explains that some scholars have been "reluctant to show ways in which [men and women] are different, because differences between two groups have so often been used to 'justify' unequal treatment" (lines 2-5). The author of Passage 1 appreciates the good intentions of those scholars, but ultimately disagrees with them. The author clearly believes that acknowledging gender differences will help people relate to members of the opposite sex.

2006-11-12

The Official SAT Online Course

17/22

(or others) for gender-based misunderstandings.

Explanation for Incorrect Answer A : Choice (A) is incorrect. Recognizing gender differences would not cause people to exaggerate their similarities. It would make men and women aware of their differences, which would lead to improved understanding between people.

Explanation for Incorrect Answer B : Choice (B) is incorrect. According to Passage 1, recognizing gender differences would decrease dissatisfaction, not increase it.

Explanation for Incorrect Answer C : Choice (C) is incorrect. While the passage does not seek to justify unequal treatment and opportunity for men and women, neither is it concerned with promoting equal treatment. Rather, it argues for understanding gender-specific communication patterns in order to improve relationships.

Explanation for Incorrect Answer E : Choice (E) is incorrect. The passage is not concerned with how research on gender should be conducted, but with improving the way men and women relate to each other in conversation.

17

In lines 36-41 (Little girls . . . relationships), the author of Passage 1 assumes that for girls, a primary function of communication is to
(A)

foster a sense of intimacy between speaker and listener

(C) convey information previously unknown by the listener

(E)

create an objective atmosphere for personal discussions

ANSWERS

18

nR

Explanation for Correct Answer A : Choice (A) is correct. Passage 1 describes how girls seek to foster intimacy, or closeness, with others through communication. Little girls are said to emphasize "displaying similarities and matching experiences" (lines 38-39) in their conversations. The passage points out that for girls, the purpose of language is "establishing connection and negotiating relationships" (lines 40-41).

Explanation for Incorrect Answer B : Choice (B) is incorrect. The passage does not mention the idea that girls try to establish conversational rules; instead, it argues that they use communication as a way of establishing relationships through sharing interests and ideas.

Explanation for Incorrect Answer C : Choice (C) is incorrect. Lines 36-41 state that instead of seeking to impart information, girls focus on "displaying similarities and matching experiences" (lines 38-39) in their conversations.

Explanation for Incorrect Answer D : Choice (D) is incorrect. Nostalgic feelings, or feelings of longing for the past, are not mentioned in the description of girls' communication patterns.

Explanation for Incorrect Answer E : Choice (E) is incorrect. According to the passage, girls are interested in establishing rapport, or creating agreement. The passage does not describe either girls or boys as seeking to be objective, or unbiased, in their conversations.

The author of Passage 2 would most likely challenge the claim made in lines 27-31 of Passage 1 (The sociolinguistic . . . communication) by arguing that

eg

AND EXPLANATIONS

is t

(D) promote nostalgic feelings about past friendships

er

(B)

establish a set of conversational rules shared by speaker and listener

ed

file://E:\\c6.htm

2006-11-12

The Official SAT Online Course

18/22

(A)

children do not grow up in single-gender cultures

(B)

children may become skilled at deceiving adults

(C) gender differences are impossible to assess scientifically

(D) there is less conflict between men and women than sociologists assume

(E)

childrens behaviors have changed dramatically in recent years

ANSWERS

AND EXPLANATIONS

Explanation for Correct Answer A : Choice (A) is correct. The claim made in lines 27-31 of Passage 1 is that "boys and girls grow up in what are essentially different cultures," so any challenge must address that claim directly. The author of Passage 2 states that children have "countless experiences communicating with people of both sexes" (line 86). The author of Passage 2 would thus argue that children do not grow up in a singlegender culture.

Explanation for Incorrect Answer B : Choice (B) is incorrect. The author of Passage 2 does not discount sociolinguistic research by suggesting that children deceive adults, but by suggesting that researchers deceive themselves.

Explanation for Incorrect Answer E : Choice (E) is incorrect. There is no mention in either passage of how the behavior of children has changed in recent years. Neither passage is concerned with changes over time.

19

nR
(A)

The sentence in lines 47-48 in Passage 2 (First . . . expectation) primarily emphasizes which damaging effect of gender stereotypes?

They may offend the person being stereotyped.

(B)

They may distort our observations of people we meet.

(C) They have been used to justify gender inequality.

(D) They commonly cause miscommunication between men and women.

(E)

They reflect negatively on those who believe in them.

ANSWERS

Explanation for Correct Answer B : Choice (B) is correct. According to the author, stereotypes "may dictate what we notice and bias our perceptions in the direction of expectation" (lines 47-48). In other words, when we meet people, gender stereotypes may cause us to look for the qualities we expect those people to have, and to focus on those qualities instead of others. These stereotypes make it difficult to observe people with a fair, unbiased eye; they distort, or skew, our view of people.

Explanation for Incorrect Answer A : Choice (A) is incorrect. The passage does not mention the feelings of people being stereotyped.

eg

AND EXPLANATIONS

is t

Explanation for Incorrect Answer D : Choice (D) is incorrect. The claim made in lines 27-31 of Passage 1 is that "boys and girls grow up in what are essentially different cultures." The author of Passage 2 would not be likely to address this claim by arguing that men and women experience less conflict than some scholars assume.

er

Explanation for Incorrect Answer C : Choice (C) is incorrect. Passage 2 argues that gender differences have not been studied scientifically, or objectively, by researchers. However, the author of Passage 2 does not claim that this task is impossible.

ed

file://E:\\c6.htm

2006-11-12

The Official SAT Online Course

19/22

Explanation for Incorrect Answer C : Choice (C) is incorrect. Passage 2 does not mention the use of stereotypes to justify unequal treatment of men and women.

Explanation for Incorrect Answer D : Choice (D) is incorrect. The author of Passage 2 may believe that miscommunication between men and women can be caused by stereotyping; however, lines 47-48 refer to the ways gender stereotypes distort people's views of one another.

Explanation for Incorrect Answer E : Choice (E) is incorrect. Passage 2 states that gender stereotypes influence our perceptions about others. It does not discuss what these stereotypes reveal about those who believe in them.

20

The assumptions underlying the research work described in lines 48-51 of Passage 2 are most similar to the assumptions held by the
(A)

scholars in line 2

(B)

women and men in line 19

(C) noted scholar in line 66

(D) author of Passage 1

(E)

author of Passage 2

ANSWERS

AND EXPLANATIONS

21

nR
(A)

Explanation for Incorrect Answer A : Choice (A) is incorrect. The "scholars" in line 2 are reluctant, or unwilling, to show ways in which men and women are different. The research work described in lines 48-51 assumes that there are gender differences and asks that they be acknowledged.

Explanation for Incorrect Answer B : Choice (B) is incorrect. The dissatisfied women and men mentioned in line 19 most likely do not assume that gender differences account for their misunderstandings. The "researchers" in Passage 2 are trying to help such people.

Explanation for Incorrect Answer C : Choice (C) is incorrect. The "noted scholar" mentioned in line 66 does not assume that gender differences determine the way a person acts. This scholar's idea is very different from the claim of the researchers mentioned in lines 48-51: that acknowledging gender differences will help men and women communicate.

Explanation for Incorrect Answer E : Choice (E) is incorrect. The author of Passage 2 does not assume that acknowledging gender differences will help men and women communicate. This author is concerned about the harm done by gender stereotypes.

The quotation in lines 66-69 (None . . . masculine) primarily serves to

introduce a personal experience

(B)

provide a typical example

(C) elaborate on an idea

(D) signal a change in topic

eg

is t

Explanation for Correct Answer D : Choice (D) is correct. The assumptions of the researchers mentioned in lines 48-51 are very similar to those of the author of Passage 1. Both want "to elucidate gender differences in order to help women and men understand and respond to each other better." As the author of Passage 1 states, "it is clear to me that recognizing gender differences in conversational styles would free individuals from the burden of an inappropriate sense of being at fault" (lines 16-18).

er

ed

file://E:\\c6.htm

2006-11-12

The Official SAT Online Course

20/22

(E)

offer recent research data

ANSWERS

AND EXPLANATIONS

Explanation for Correct Answer C : Choice (C) is correct. The sentence preceding the quote argues that we need to think of gender "as something people do in social interaction" (lines 65-66). The quotation, by pointing out that "in particular contexts people do feminine, in others, they do masculine," elaborates on the idea that people's behavior is determined by situation rather than by gender.

Explanation for Incorrect Answer A : Choice (A) is incorrect. The quotation is not about someone's personal experience; it is a statement about people's experiences in general.

Explanation for Incorrect Answer B : Choice (B) is incorrect. The quotation does not provide any kind of example; it is a generalization, or broad idea.

Explanation for Incorrect Answer D : Choice (D) is incorrect. Instead of signaling a new topic, this quotation extends the topic: the idea that gender is "something people do" that varies with the situation.

Explanation for Incorrect Answer E : Choice (E) is incorrect. Even though this is a quotation by a "noted scholar," it does not mention any recent research data.

Passage 2 suggests that some scholars construct a contrasting picture of the cultures of boys and girls (lines 77-78) by studying children whose

(B)

demand for approval from adults is particularly strong

(C) rebellion against authority results in creative behavior

nR
(E)

(D) personalities are highly idiosyncratic

actions correspond to a narrow preconception of behavior

ANSWERS

Explanation for Correct Answer E : Choice (E) is correct. According to Passage 2, in the attempt to highlight the differences between boys and girls, some scholars are "singling out those children who fit common gender stereotypes and marginalizing others" (lines 78-80). By studying children who tend to behave according to stereotypes, or "narrow preconceptions of behavior," those scholars can construct a picture that highlights the differences in the cultures of boys and girls.

Explanation for Incorrect Answer A : Choice (A) is incorrect. Passage 2 does not say anything about children who are eager to interact with strangers.

Explanation for Incorrect Answer B : Choice (B) is incorrect. Passage 2 does not suggest that scholars study children who are in strong need of approval from adults.

Explanation for Incorrect Answer C : Choice (C) is incorrect. The scholars discussed in Passage 2 study children's behavior and attitudes toward each other, not toward adults or figures of authority. Furthermore, there is no mention of rebellion or creative behavior in Passage 2.

Explanation for Incorrect Answer D : Choice (D) is incorrect. Passage 2 does not suggest that children in the studies

eg i

(A)

readiness to interact with strangers is apparent

AND EXPLANATIONS

st

22

er
2006-11-12

file://E:\\c6.htm

ed

The Official SAT Online Course

21/22

have highly idiosyncratic personalities, or that they have very particular individual characteristics. On the contrary, the author states that the studies focus on children who fit common gender stereotypes while ignoring children who do not act in expected ways.

23

The author of Passage 2 implies that the boys mentioned in line 81 and the child mentioned in line 90 resemble one another in that they
(A)

have not learned to imitate adult behaviors

(B)

refuse to get along with their peers

(C) do not conform to traditional gender stereotypes

(D) openly mock adult expectations about their behavior

(E)

communicate primarily with children of their own gender

ANSWERS

AND EXPLANATIONS

Explanation for Incorrect Answer B : Choice (B) is incorrect. The passage doesn't say whether these children get along with their peers. The author's main point is that they do not act according to gender stereotypes.

24

nR
(A)

Explanation for Incorrect Answer D : Choice (D) is incorrect. There is nothing to suggest that these children "mock," or make fun of, adults' expectations of them, or that they are paying attention to adults at all.

Explanation for Incorrect Answer E : Choice (E) is incorrect. The passage suggests that these children, like most children, actually "have countless experiences communicating with people of both sexes" (lines 86-87).

Which of the following best characterizes the ideas about gender communication styles as they are presented in the two passages?

Passage 1 argues that styles are based on competition, while Passage 2 suggests that they are a form of cooperation.

(B)

Passage 1 argues that styles are a burden, while Passage 2 implies that they can help facilitate relationships between men and women.

(C) Passage 1 claims that styles are semantic, while Passage 2 suggests that they are whimsical.

(D) Passage 1 suggests that styles are constant, while Passage 2 argues that they are fluid.

(E)

Passage 1 states that styles are random, while Passage 2 indicates that their patterns become obvious upon closer scrutiny.

ANSWERS

Explanation for Correct Answer D : Choice (D) is correct. According to Passage 1, communication styles are determined by gender and are thus constant and predictable. The author states: "I believe that there are gender differences in ways of speaking, and we need to identify and understand them" (lines 10-12). Passage 2, on the other hand, states that gender

eg is

AND EXPLANATIONS

te

Explanation for Incorrect Answer A : Choice (A) is incorrect. Caring for younger siblings and displaying dominance or deference might be adult behaviors that these children are imitating.

re

Explanation for Correct Answer C : Choice (C) is correct. The author cites these children as examples of children who do not "conform to," or fit in with, traditional gender stereotypes. The "boys who excel at caring for younger siblings" are demonstrating a quality more associated with girls, the author implies. Likewise, the child who displays dominance with a younger playmate and shows deference to an older friend is behaving according to the situation, not according to gender.

file://E:\\c6.htm

2006-11-12

The Official SAT Online Course


communication styles are fluid and change according to the individual and the situation: "people display contradictory behaviors as they encounter different social norms and pressures" (lines 69-70).

22/22

Explanation for Incorrect Answer A : Choice (A) is incorrect. In Passage 1, the author mentions that boys tend to make competitive statements of achievement. However, Passage 1 does not argue that all styles are based on competition, and Passage 2 does not suggest that all conversational styles are a form of cooperation.

Explanation for Incorrect Answer B : Choice (B) is incorrect. Passage 1 does not argue that styles are a burden, but rather that understanding them can help improve understanding between men and women. Passage 2 argues against the existence of gender specific styles altogether; it points out that thinking in terms of "differences rather than similarities" is not helpful.

Explanation for Incorrect Answer C : Choice (C) is incorrect. Passage 1 does claim that styles are semantic, in that they have to do with meaning in language. However, Passage 2 argues that people act and speak according to what they have learned to do in different situations. This has nothing to with conversational styles being whimsical.

is

te
Back to Score Report

re

Explanation for Incorrect Answer E : Choice (E) is incorrect. Passage 1 does not state that conversational styles are random, or a result of chance, but that they are determined by gender. Furthermore, Passage 2 argues that closer scrutiny will show that styles of behavior in general are less dependent on gender than is commonly supposed.

nR

eg

Copyright 2006 The College Board. All rights reserved.

Privacy Policy

Terms of Use

Contact Us

file://E:\\c6.htm

2006-11-12

The Official SAT Online Course

1/16

Help | Profile | My Organizer | My Bookmarks | Logout

Answers and Explanations

Test Sections

Back to Score Report

Section 1

View Answers and Explanations


Online - Practice Test #3

Section 2

Section 3

Section 4

If
(A)
(B)

then

Section 5

Section 7

Section 8

Section 9

(C)

Section 10

(D)

(E)

Explanation for Correct Answer B : Choice (B) is correct. If

ed
then

ANSWERS

AND EXPLANATIONS

and

Explanation for Incorrect Answer A : were Choice (A) is not correct. If

eg i
were

st
then

er
would be equal to

instead of the given value of

nR

Explanation for Incorrect Answer C :

Choice (C) is not correct. If

then

would be equal to

instead of the given value of

Explanation for Incorrect Answer D :

Choice (D) is not correct. If

were

then

would be equal to

instead of the given value of

Explanation for Incorrect Answer E : then were Choice (E) is not correct. If

would be equal to

instead of the given value of

In the triangle above,

file://E:\\c7.htm

2006-11-12

The Official SAT Online Course

2/16

(A)
(B)
(C)
(D)
(E)

ANSWERS

AND EXPLANATIONS
So

Explanation for Correct Answer D : Choice (D) is correct. The sum of the interior angles of a triangle is

Solving for

gives

Explanation for Incorrect Answer A : then the sum of the angles in the given were Choice (A) is not correct. If

triangle would be

instead of

Explanation for Incorrect Answer B : then the sum of the angles in the given were Choice (B) is not correct. If

triangle would be

instead of

triangle would be

triangle would be

nR
(A)

are lawn mowers produced by a lawn mower factory, exactly For every defective. At this rate, how many lawn mowers were produced during a period in lawn mowers were defective? which exactly

(B)

(C)

(D)
(E)

ANSWERS

Explanation for Correct Answer D : Choice (D) is correct. The rate of defective mower production is

eg

AND EXPLANATIONS

is te
which reduces to

Explanation for Incorrect Answer E : then the sum of the angles in the given were Choice (E) is not correct. If

re
instead of

Explanation for Incorrect Answer C : then the sum of the angles in the given were Choice (C) is not correct. If

instead of

If the rate continues then

where x is the

number of lawn mowers produced when exactly

of them are defective. Solving

for

gives

Explanation for Incorrect Answer A : Choice (A) is not correct. Since the rate of defective mower production was

defective mowers out of

the production of only

lawn mowers would

result in fewer than

defective mowers.

Explanation for Incorrect Answer B : Choice (B) is not correct. This choice is the number of lawn mowers produced in a were defective. The question asked for the number of period in which exactly

file://E:\\c7.htm

2006-11-12

The Official SAT Online Course

3/16

lawn mowers produced in a period in which exactly

were defective.

Explanation for Incorrect Answer C : lawn mowers were produced, then the number Choice (C) is not correct. If

of defective mowers would be

The question asked for the

number of lawn mowers produced in a period in which exactly

were defective.

Explanation for Incorrect Answer E : defective mowers per Choice (E) is not correct. There were

, so

lawn mowers would include

defective ones. The question asked for the number

of lawn mowers produced in a period in which exactly

were defective.

If
(A)
(B)

what is the value of

(C)

(D)

(E)

ANSWERS

AND EXPLANATIONS
is the same as

Explanation for Correct Answer B :

re d
So,

Explanation for Incorrect Answer A : Choice (A) is not correct. If the value of

eg is te
Thus,

Choice (B) is correct.

and

were equal to

then

which is not equal to the given value of

nR

Explanation for Incorrect Answer C : Choice (C) is not correct. If the value of

were equal to

then

which is not equal to the given value of

Explanation for Incorrect Answer D : Choice (D) is not correct. If the value of

were equal to

then

which is not equal to the given value of

Explanation for Incorrect Answer E : Choice (E) is not correct. If the value of

were equal to

then

which is not equal to the given value of

about point The figure above will be rotated of the following represents the rotated figure?

in the direction indicated. Which

file://E:\\c7.htm

2006-11-12

The Official SAT Online Course

4/16

(A)

(B)

(C)

(D)

(E)

ANSWERS

AND EXPLANATIONS

Explanation for Correct Answer A :

d
of

direction indicated.

nR

Explanation for Incorrect Answer C : Choice (C) is not correct. This figure would result from rotating the original figure in the opposite direction from that indicated in the question. about

Explanation for Incorrect Answer D : Choice (D) is not correct. This figure would result from rotating the original figure in the direction indicated. about more than

Explanation for Incorrect Answer E : Choice (E) is not correct. This figure would result from rotating the original figure in the direction indicated. about less than

eg

about

in the direction indicated.

is
what is
is

Explanation for Incorrect Answer B : Choice (B) is not correct. This figure would result from rotating the original figure

If

is
(A)

percent of

(B)

(C)

(D)

(E)

ANSWERS

AND EXPLANATIONS

Explanation for Correct Answer B :

te
or

re
percent of
of

Choice (A) is correct. This is the result of rotating the figure

about

in the

Choice (B) is correct. Since

it follows that

percent is

Thus,

percent of

is

which is

file://E:\\c7.htm

2006-11-12

The Official SAT Online Course

5/16

Explanation for Incorrect Answer A : is the value of Choice (A) is not correct. Though percent of

the question asks for

Explanation for Incorrect Answer C : is Choice (C) is not correct. Though

percent of

and

is the value

of

the question asked for

percent of

not of

Explanation for Incorrect Answer D :

Choice (D) is not correct. Though

percent is

the question asked for

the value of

percent of 10.

Explanation for Incorrect Answer E :

Choice (E) is not correct. Since

is

or

of

it follows that

percent is

However,

is not

of

nR
(B)
(C)
(D)
(E)

(A)

ANSWERS

Explanation for Correct Answer E : to Choice (E) is correct. There are four ways to get from miles, then miles, then miles, starting out traveling

eg i

In the diagram of roads above, the numbers represent road distances in miles, and the arrows show the only directions in which travel is permitted on the roads. If the miles, and if the length of the shortest is to length of the longest route from miles, then is to route from

AND EXPLANATIONS
The longest way is miles. The shortest

st er
miles, then

ed
way is

miles, starting out traveling

miles, and then

miles.

So the difference between the longest route and the shortest route is

Explanation for Incorrect Answer A : Choice (A) is not correct. This is the length of the longest route, but the question asked for the difference between the longest route and the shortest route.

Explanation for Incorrect Answer B : but the question Choice (B) is not correct. There is a route that has length asked for the difference between the longest route and the shortest route.

Explanation for Incorrect Answer C : Choice (C) is not correct. This is the length of the shortest route, but the question asked for the difference between the longest route and the shortest route.

file://E:\\c7.htm

2006-11-12

The Official SAT Online Course

6/16

Explanation for Incorrect Answer D : Choice (D) is not correct. The length of the longest route is

and the length of the

shortest route is

so the difference is

not

(not shown) is perpendicular to In the figure above, line Which of the following points lies on line
(A)
(B)

and bisects

(D)
(E)

ANSWERS

AND EXPLANATIONS

Explanation for Correct Answer D :

is te r
or

Choice (D) is correct. The midpoint of segment

ed
is

(C)

The slope of segment

eg

is

so a perpendicular line

will go through the point

and will have

a slope

Using the point-slope form of the equation of a line, line

is described

by the equation

The only answer choice with the

nR

coordinate equal to the

coordinate is

Explanation for Incorrect Answer A :

Choice (A) is not correct. The coordinates

define point

If

was on

line

, then

would not bisect

since it would go through one of the

endpoints of

Explanation for Incorrect Answer B :

Choice (B) is not correct. The midpoint of

is the point

The line

containing

and the point

would bisect

but would not be

perpendicular to

Explanation for Incorrect Answer C :

Choice (C) is not correct. The midpoint of

is the point

The line

containing

and the point

would bisect

but would not be

perpendicular to

Explanation for Incorrect Answer E :

Choice (E) is not correct. The midpoint of

is the point

The line

containing

and the point

would bisect

but would not be

perpendicular to

file://E:\\c7.htm

2006-11-12

The Official SAT Online Course

7/16

If
(A)
(B)
(C)
(D)
(E)

and

what is the value of

ANSWERS

AND EXPLANATIONS
can be expressed as

Explanation for Correct Answer A :

Choice (A) is correct. The number

so the equation

can be rewritten as

Thus,

The numeric value of

can

now be substituted into the second equation, which yields

Solving for

it

follows that

Explanation for Incorrect Answer B :

st er e
were equal to

d
then

Choice (B) is not correct. If

would be

and

Explanation for Incorrect Answer C :

Choice (C) is not correct. If

were equal to

then

would be

and

Explanation for Incorrect Answer D :

eg i

nR

Choice (D) is not correct. If

were equal to

then

would be

and

Explanation for Incorrect Answer E :

Choice (E) is not correct. If

were equal to

then

would be

and

10

What is the radius of a circle whose circumference is


(A)
(B)
(C)
(D)
(E)

ANSWERS

AND EXPLANATIONS
where

Explanation for Correct Answer A : Choice (A) is correct. The formula for the circumference of a circle is

is the radius of the circle. In this case,

which simplifies to

file://E:\\c7.htm

2006-11-12

The Official SAT Online Course

8/16

Explanation for Incorrect Answer B : Choice (B) is not correct. If the radius of the circle were

then the circumference

would be

instead of

Explanation for Incorrect Answer C : Choice (C) is not correct. If the radius of the circle were

then the circumference

would be

instead of

Explanation for Incorrect Answer D : Choice (D) is not correct. If the radius of the circle were

then the circumference

would be

instead of

Explanation for Incorrect Answer E : Choice (E) is not correct. If the radius of the circle were

then the circumference

would be

instead of

11

How many of the prime factors of


(A)

are greater than

(B)

Two

(D) Four

ANSWERS

Explanation for Correct Answer B :

eg is

(E)

Five

AND EXPLANATIONS
Of these

Choice (B) is correct. The prime factorization of thirty is

nR

three prime factors, only two are greater than

Explanation for Incorrect Answer A : are prime factors of and Choice (A) is not correct. Both Hence, more than one prime factor is greater than than

te
that are greater

(C) Three

Explanation for Incorrect Answer C : does have three prime factors, the question Choice (C) is not correct. While asks how many of its prime factors are greater than

Explanation for Incorrect Answer D : only has three prime factors, so it Choice (D) is not correct. The number cannot have four prime factors that are greater than two.

Explanation for Incorrect Answer E : only has three prime factors, so it cannot Choice (E) is not correct. The number have five prime factors that are greater than two.

12

re
numbers above is

One

d
what is

If the average (arithmetic mean) of the of

in terms

file://E:\\c7.htm

2006-11-12

The Official SAT Online Course

9/16

(A)

(B)
(C)

(D)
(E)

ANSWERS

AND EXPLANATIONS
it

Explanation for Correct Answer E : Choice (E) is correct. Since the arithmetic mean of the three numbers is

follows that

This simplifies to

or

Explanation for Incorrect Answer A : were equal to Choice (A) is not correct. If

then the average of the three

numbers would be

However, the average of

and

is

Explanation for Incorrect Answer B :

re

d
were equal to

Choice (B) is not correct. If

then the average of

and

te is
is

would be

However, the average of

and

is

eg

Explanation for Incorrect Answer C : were equal to Choice (C) is not correct. If

then the average of the three

numbers would be

However, the average of

nR

and

Explanation for Incorrect Answer D :

Choice (D) is not correct. If

were equal to

then the average of

and

would be

However, the average of

and

is

13

and the difference between any In the increasing sequence above, the first term is What is the value of the fourth term in the sequence? two consecutive terms is

(A)
(B)

(C)

(D)
(E)

ANSWERS

AND EXPLANATIONS

file://E:\\c7.htm

2006-11-12

The Official SAT Online Course

10/16

Explanation for Correct Answer C : Choice (C) is correct. If the difference between any two consecutive terms is

then the fourth term is equal to the third term plus

or

Also,

the first term subtracted from the second term must be equal to

So

which simplifies to

Thus, the fourth term is

Explanation for Incorrect Answer A : Choice (A) is not correct. This choice is the value of equal to the fourth term.

the first term, which is not

Explanation for Incorrect Answer B : Choice (B) is not correct. This choice is the value of the third term, which is not equal to the fourth term.

Explanation for Incorrect Answer D : Choice (D) is not correct. If the value of the fourth term were

then the value of

the third term would be

If

then

making the second term

However, this would make the difference between the

second and third consecutive terms

instead of

the third term would be

If

would be

This would mean the difference between the

second and third consecutive terms was

14

is te r
If
then

ed
then

Explanation for Incorrect Answer E : Choice (E) is not correct. If the value of the fourth term were

then the value of

and the second term

instead of

Let the function f be defined by

what is the value of

(B)

(C)

nR
(D)

(E)

ANSWERS

Explanation for Correct Answer E :

eg
AND EXPLANATIONS

(A)

Choice (E) is correct. If

or

Solving

for

gives

Explanation for Incorrect Answer A : were equal to Choice (A) is not correct. If

then

Explanation for Incorrect Answer B : were equal to Choice (B) is not correct. If

then

Explanation for Incorrect Answer C :

Choice (C) is not correct. If

were equal to

then

file://E:\\c7.htm

2006-11-12

The Official SAT Online Course

11/16

Explanation for Incorrect Answer D : were equal to Choice (D) is not correct. If

then

15

walls, including a In a windowless, cube-shaped storage room, the ceiling and door, are completely painted. The floor is not painted. If the painted area is equal to square meters, what is the volume of the room, in cubic meters?
(A)

(B)

(C)

(D)
(E)

ANSWERS

AND EXPLANATIONS

Explanation for Correct Answer C : Choice (C) is correct. The area of five of the six square surfaces inside the room is represents the length in meters of each edge of the square meters. If

Solving for

gives

the storage room is

Explanation for Incorrect Answer A : Choice (A) is not correct. If the volume of the room were

eg is

te r
cubic meters.

the length of any of its edges raised to the third power, it follows that the volume of

the length of each edge of the room would be

painted surfaces would be

nR

square meters given in the problem.

Explanation for Incorrect Answer B : Choice (B) is not correct. If the volume of the room were

the length of each edge of the room would be

painted surfaces would be

square meters given in the problem.

Explanation for Incorrect Answer D : Choice (D) is not correct. If the volume of the room were

ed

square surfaces, then the area of each of the surfaces is

and

meters. Since the volume of a cube is

cubic meters, then

meters and the area of the five

square meters instead of the

cubic meters, then

meters and the area of the five

square meters instead of the

cubic meters, then

the length of each edge of the room would be

meters and the area of the

five painted surfaces would be

square meters instead of

the

square meters given in the problem.

Explanation for Incorrect Answer E : Choice (E) is not correct. If the volume of the room were

cubic meters, then

the length of each edge of the room would be

meters and the area of the

five painted surfaces would be

square meters instead of

the

square meters given in the problem.

16

file://E:\\c7.htm

2006-11-12

The Official SAT Online Course

12/16

According to the formula

if the temperature in degrees Fahrenheit

increases by

by how much does the temperature in degrees Celsius

increase?
(A)

(B)

(C)

(D)
(E)

ANSWERS

AND EXPLANATIONS

Explanation for Correct Answer B : in the formula given, gives that Choice (B) is correct. Solving for

Every

-degree change on the Fahrenheit scale results in a

degree change on the Celsius scale. Therefore, a change of temperature of

degrees Fahrenheit corresponds to a change of temperature of

degrees Celsius.

change on the Celsius scale results in a

is

If the temperature in Celsius increased by

te

Explanation for Incorrect Answer A : Choice (A) is not correct. From the formula given in the problem, every

re

d
-degree

-degree change on the Fahrenheit scale.

the increase in the Fahrenheit

temperature would have been

degrees, not

degrees.

Explanation for Incorrect Answer C : Choice (C) is not correct. From the formula given in the problem, every

eg

-degree

nR

change on the Celsius scale results in a

-degree change on the Fahrenheit scale.

If the temperature in Celsius increased by

the increase in the Fahrenheit

temperature would have been

degrees, not

degrees.

Explanation for Incorrect Answer D : Choice (D) is not correct. From the formula given in the problem, every

-degree

change on the Celsius scale results in a

-degree change on the Fahrenheit scale.

If the temperature in Celsius increased by

the increase in the Fahrenheit

temperature would have been

degrees, not

degrees.

Explanation for Incorrect Answer E : Choice (E) is not correct. From the formula given in the problem, every

-degree

change on the Celsius scale results in a

-degree change on the Fahrenheit scale.

If the temperature in Celsius increased by

the increase in the Fahrenheit

temperature would have been

degrees, not

degrees.

17

For all values of

let

be defined by

Which of the following is

file://E:\\c7.htm

2006-11-12

The Official SAT Online Course

13/16

equal to
(A)

(B)

(C)

(D)

(E)

ANSWERS

AND EXPLANATIONS
it follows that

Explanation for Correct Answer D :

Choice (D) is correct. Since

Explanation for Incorrect Answer A :

re d

Choice (A) is not correct. Since the definition for

holds for all values of

it

have to be equal to

and

eg

Explanation for Incorrect Answer B :

is te
and

must hold for

so if

were equal to

then

would

However, from the original definition,

Choice (B) is not correct. Since the definition for

holds for all values of

it

must hold for

so if

were equal to

then

nR

would have to be equal to

However, from the original definition,

and

Explanation for Incorrect Answer C :

Choice (C) is not correct. Since the definition for

holds for all values of

it

must hold for

so if

were equal to

then

would have to be equal to

However, from the original definition,

and

Explanation for Incorrect Answer E :

Choice (E) is not correct. Since the definition for

holds for all values of

it

must hold for

so if

were equal to

then

would have to be equal to

However, from the original

definition,

18

If three different circles are drawn on a piece of paper, at most how many points can be common to all three?
(A)

None

(B)

One

(C) Two

(D) Three

file://E:\\c7.htm

2006-11-12

The Official SAT Online Course

14/16

(E)

Six

ANSWERS

AND EXPLANATIONS

Explanation for Correct Answer C : Choice (C) is correct. Two different circles can cross in at most two places. Since two circles can only meet at two distinct points, a third circle can coincide with the other two, at most, at those two places. An illustration of this is shown in the figure.

Explanation for Incorrect Answer A : Choice (A) is not correct. There can be at least one in common because three circles can all share the same point.

19

nR
(A)
(B)

A club is buying boxes of candy bars to sell for a fund-raiser. If each box contains bars each day, how many boxes are needed to candy bars, and each member sells days? members to sell for supply enough candy bars for

(C)

(D)

(E)

ANSWERS

Explanation for Correct Answer E : Choice (E) is correct. Since each member sells

eg

AND EXPLANATIONS
candy bars a day, over the

is

Explanation for Incorrect Answer E : Choice (E) is not correct. Though the maximum number of points of intersection common to two of the three circles could be six, all three circles would not have all six points in common. See the explanation for the correct answer.

te r
boxes.

Explanation for Incorrect Answer D : Choice (D) is not correct. Though there are three circles, the circles can have at most two points in common. See the explanation for the correct answer.

ed

Explanation for Incorrect Answer B : Choice (B) is not correct. Though three circles can cross at only one point, the question asks for the most points they can have in common. See the explanation for the correct answer.

day period, each member will sell

candy bars. Since there are

members, a

total of

candy bars is needed. Candy bars come in boxes with

bars in

each box, so the group needs

Explanation for Incorrect Answer A :

Choice (A) is not correct.

boxes would contain

candy

file://E:\\c7.htm

2006-11-12

The Official SAT Online Course

15/16

bars. However, over the

-day period, each of the

members will sell

candy

bars so only

candy bars are needed.

Explanation for Incorrect Answer B :

Choice (B) is not correct.

boxes would contain only

candy bars. However,

over the

-day period, each of the

members will sell

candy bars so

candy bars are needed.

Explanation for Incorrect Answer C :

Choice (C) is not correct.

boxes would contain only

candy bars.

However, over the

-day period, each of the

members will sell

candy bars

so

candy bars are needed.

Explanation for Incorrect Answer D :

Choice (D) is not correct.

boxes would contain

candy bars. However,

over the

-day period, each of the

members will sell

candy bars so

candy bars are needed.

eg i
A point in square

st er
In the figure above, the coordinates of

are

is to be chosen at random. If the probability

nR

that the point will be in the shaded triangle is

(A)

(B)

(C)

(D)
(E)

ANSWERS

AND EXPLANATIONS

Explanation for Correct Answer C : Choice (C) is correct. The probability that a point chosen at random from the square is in the shaded triangle is equal to the ratio of the area of the triangle to If A is the area of the square. The area of the square is

the area of the triangle, then

so

ed
are

20

and the coordinates of

what is the value of

The base of the triangle is

and the height of the triangle is

Using the

formula for the area of a triangle, it follows that

so

file://E:\\c7.htm

2006-11-12

The Official SAT Online Course

16/16

Explanation for Incorrect Answer A :

Choice (A) is not correct. The base of the triangle is

and the height of the triangle is

If

were

then the area of the

triangle

would be

and this area should represent

of

the area of the square. However, the square

has area

and

is not

of

Explanation for Incorrect Answer B :

Choice (B) is not correct. The base of the triangle is

and the height of the triangle is

If

were

then the area of the

triangle

would be

and this area should represent

of the area of the square. However, the square

has area

and

is

not

of

Explanation for Incorrect Answer D :

Choice (D) is not correct. If the value of

were equal to

then the

coordinate of point

would be

However, the

point

is to the right of the origin,

so the

-coordinate of point

can not be

Explanation for Incorrect Answer E :

Choice (E) is not correct. The base of the triangle is

er
If

and the height of the triangle is

ed
were

negative.

then the area of the triangle

would be

and this area should represent

of the area of

is t

the square. However, the square

has area

and

is not

of

nR

eg
Back to Score Report

Copyright 2006 The College Board. All rights reserved.

Privacy Policy

Terms of Use

Contact Us

file://E:\\c7.htm

2006-11-12

The Official SAT Online Course

1/16

Help | Profile | My Organizer | My Bookmarks | Logout

Answers and Explanations

Test Sections

Back to Score Report

Section 1

View Answers and Explanations


Online - Practice Test #3

Section 2

Section 3

Section 4

Section 5

Originally ------- mainly by young, urban audiences, rap music was ultimately ------by its appreciative listeners of all ages across the country.
(A)

Section 7

admired . . embraced

Section 8

(B)

performed . . condemned

Section 9

Section 10

(C) derided . . ignored

(D) appropriated . . relinquished

(E)

applauded . . instigated

Explanation for Incorrect Answer B : Choice (B) is incorrect. "Performed" means acted out or achieved. "Condemned" means disapproved of. If one were to insert these terms into the text, the sentence would read "Originally performed mainly by young, urban audiences, rap music was ultimately condemned by its appreciative listeners of all ages across the country." The phrase "appreciative listeners of all ages" indicates that rap music's fan base expanded from what it was "originally." The correct response is the only one in which both missing terms have a positive meaning, indicating that the appreciation began with one group and expanded to include a much larger group. Audiences do not typically perform music, and listeners would not be described as appreciative of something they are condemning.

nR

Explanation for Incorrect Answer C : Choice (C) is incorrect. "Derided" means ridiculed. "Ignored" means disregarded. If one were to insert these terms into the text, the sentence would read "Originally derided mainly by young, urban audiences, rap music was ultimately ignored by its appreciative listeners of all ages across the country." The phrase "appreciative listeners of all ages" indicates that rap music's fan base expanded from what it was "originally." The correct response is the only one in which both missing terms have a positive meaning, indicating that the appreciation began with one group and expanded to include a much larger group. Both "derided" and "ignored" have a negative, not a positive, connotation. Music would not be both appreciated and ignored by the same group of listeners.

Explanation for Incorrect Answer D : Choice (D) is incorrect. "Appropriated" means something is taken and used. "Relinquished" means abandoned. If one were to insert these terms into the text, the sentence would read "Originally appropriated mainly by young, urban audiences, rap music was ultimately relinquished by its appreciative listeners of all ages across the country." The phrase "appreciative listeners of all ages" indicates that rap music's fan base expanded from what it was "originally." The correct response is the only one in which both missing terms have a positive meaning, indicating that the appreciation began with one group and expanded to include a much larger group. Listeners would not relinquish the music they appreciate.

eg

is

Explanation for Correct Answer A : Choice (A) is correct. "Admired" means thought highly of. "Embraced" means held closely. If one were to insert these terms into the text, the sentence would read "Originally admired mainly by young, urban audiences, rap music was ultimately embraced by its appreciative listeners of all ages across the country." The phrase "appreciative listeners of all ages" indicates that rap music's fan base expanded from what it was "originally." The correct response is the only one in which both missing terms have a positive meaning, indicating that the appreciation began with one group and expanded to include a much larger group. Both "admired" and "embraced" are positive terms.

te

re d

ANSWERS

AND EXPLANATIONS

file://E:\\c8.htm

2006-11-12

The Official SAT Online Course

2/16

Explanation for Incorrect Answer E : Choice (E) is incorrect. "Applauded" means approved of. "Instigated" means provoked. If one were to insert these terms into the text, the sentence would read "Originally applauded mainly by young, urban audiences, rap music was ultimately instigated by its appreciative listeners of all ages across the country." The phrase "appreciative listeners of all ages" indicates that rap music's fan base expanded from what it was "originally." The correct response is the only one in which both missing terms have a positive meaning, indicating that the appreciation began with one group and expanded to include a much larger group. Listeners would not have instigated rap music.

It was out of ------- that Professor Green, the author of several highly respected books in his field, described himself to his colleagues as -------.
(A)

embarrassment . . a paragon

(B)

magnanimity . . an avenger

(C) insolence . . a pedant

(D) egotism . . an apprentice

(E)

modesty . . a dilettante

ANSWERS

AND EXPLANATIONS

Explanation for Incorrect Answer A : Choice (A) is incorrect. "Embarrassment" means self-consciousness. "A paragon" is a model of excellence. If one were to insert these terms into the text, the sentence would read "It was out of embarrassment that Professor Green, the author of several highly respected books in his field, described himself to his colleagues as a paragon." The correct response for the second missing term will be the result of Professor Green's feelings as described in the first missing term. Professor Green would not describe himself as a paragon, proclaiming his excellence, as a result of his embarrassment.

nR

Explanation for Incorrect Answer B : Choice (B) is incorrect. "Magnanimity" means forgiving. "An avenger" is one who seeks revenge. If one were to insert these terms into the text, the sentence would read "It was out of magnanimity that Professor Green, the author of several highly respected books in his field, described himself to his colleagues as an avenger." The correct response for the second missing term will be the result of Professor Green's feelings as described in the first missing term. Professor Green cannot be both an avenger and magnanimous.

Explanation for Incorrect Answer C : Choice (C) is incorrect. "Insolence" means disrespect. "A pedant" is someone overly concerned with the formal rules of his field of knowledge. If one were to insert these terms into the text, the sentence would read "It was out of insolence that Professor Green, the author of several highly respected books in his field, described himself to his colleagues as a pedant." The correct response for the second missing term will be the result of Professor Green's feelings as described in the first missing term. Professor Green would be unlikely to describe himself as a pedant, or someone overly concerned with the formal rules of his field of knowledge. If he described himself as a pedant, however, it would not be as a result of insolence.

Explanation for Incorrect Answer D : Choice (D) is incorrect. "Egotism" means conceit. "An apprentice" is a beginning learner. If one were to insert these terms into the text, the sentence would read "It was out of egotism that Professor Green, the author of several highly respected books in his field, described himself to his colleagues as an apprentice." The correct response for the second missing term will be the result of Professor Green's feelings

eg

is

te

Explanation for Correct Answer E : Choice (E) is correct. "Modesty" means humility. "A dilettante" is someone who merely dabbles in a subject. If one were to insert these terms into the text, the sentence would read "It was out of modesty that Professor Green, the author of several highly respected books in his field, described himself to his colleagues as a dilettante." The correct response for the second missing term will be the result of Professor Green's feelings as described in the first missing term. Because Professor Green is highly respected in his field, his description of himself as a dilettante is simply modest; he describes himself at less than his true value.

re

file://E:\\c8.htm

2006-11-12

The Official SAT Online Course

3/16

as described in the first missing term. If Professor Green wrote several highly respected books and was egotistical, he would hardly describe himself as an apprentice, or beginning learner.

Historian Carlo Botta often contradicted himself, as when he first championed and then ------- the ideals of the French Revolution.
(A)

invoked

(B)

investigated

(C) conceived

(D) coveted

(E)

denounced

ANSWERS

AND EXPLANATIONS

Explanation for Correct Answer E : Choice (E) is correct. "Denounced" means spoke ill of. If one were to insert this term into the text, the sentence would read "Historian Carlo Botta often contradicted himself, as when he first championed and then denounced the ideals of the French Revolution." The word "contradicted" indicates that the correct response will mean the opposite of championing, or supporting, ideals. Denouncing ideals is the opposite of championing them. Thus, to do both would be to contradict oneself.

nR
(A)

Explanation for Incorrect Answer B : Choice (B) is incorrect. "Investigated" means researched. If one were to insert this term into the text, the sentence would read "Historian Carlo Botta often contradicted himself, as when he first championed and then investigated the ideals of the French Revolution." The word "contradicted" indicates that the correct response will mean the opposite of championing, or supporting, ideals. While it might have been better for Botta to investigate the ideals before he championed them, his investigation doesn't mean he is contradicting himself.

Explanation for Incorrect Answer C : Choice (C) is incorrect. "Conceived" means thought up. If one were to insert this term into the text, the sentence would read "Historian Carlo Botta often contradicted himself, as when he first championed and then conceived the ideals of the French Revolution." The word "contradicted" indicates that the correct response will mean the opposite of championing, or supporting, ideals. Botta could not have conceived the ideals of the French Revolution after having championed them. He would have conceived them and later championed them.

Explanation for Incorrect Answer D : Choice (D) is incorrect. "Coveted" means strongly desired. If one were to insert this term into the text, the sentence would read "Historian Carlo Botta often contradicted himself, as when he first championed and then coveted the ideals of the French Revolution." The word "contradicted" indicates that the correct response will mean the opposite of championing, or supporting, ideals. Coveting ideals may be odd; however, coveting and championing ideals is not a contradiction.

Luisa worked with extreme precision, ------- that served her well in her law career.

a meticulousness

(B)

an effrontery

(C) an inhibition

(D) a litigiousness

eg is

Explanation for Incorrect Answer A : Choice (A) is incorrect. "Invoked" means to call upon. If one were to insert this term into the text, the sentence would read "Historian Carlo Botta often contradicted himself, as when he first championed and then invoked the ideals of the French Revolution." The word "contradicted" indicates that the correct response will mean the opposite of championing, or supporting, ideals. For Botta to invoke the Revolution's ideals after having championed them would not be a contradiction.

te r

ed

file://E:\\c8.htm

2006-11-12

The Official SAT Online Course

4/16

(E)

impetuousness

ANSWERS

AND EXPLANATIONS

Explanation for Correct Answer A : Choice (A) is correct. "Meticulousness" means excessive care or attention to detail. If one were to insert this term into the text, the sentence would read "Luisa worked with extreme precision, a meticulousness that served her well in her law career." A person who works with extreme precision would be properly described as demonstrating "meticulousness."

Explanation for Incorrect Answer B : Choice (B) is incorrect. "Effrontery" means offensive boldness. If one were to insert this term into the text, the sentence would read "Luisa worked with extreme precision, an effrontery that served her well in her law career." Working with precision is very different from the type of shameless or offensive behavior that is termed "effrontery."

nR
(A)

In 1916 Yellowstone National Park had only 25 bison, but the population has since ------- to more than 2,000.

dispersed

(B)

mediated

(C) attenuated

(D) burgeoned

(E)

reconciled

ANSWERS

Explanation for Correct Answer D : Choice (D) is correct. "To burgeon" means to grow and flourish. If one were to insert this term into the text, the sentence would read "In 1916 Yellowstone National Park had only 25 bison, but the population has since burgeoned to more than 2,000." Since the park began with 25 bison and now has more than 2,000, the correct answer is the word that most closely means "increased." "Burgeoned" and "increased" are similar in meaning.

Explanation for Incorrect Answer A : Choice (A) is incorrect. "To disperse" means to spread out. If one were to insert this term into the text, the sentence would read "In 1916 Yellowstone National Park had only 25 bison, but the population has since dispersed to more than 2,000." Since the park began with 25 bison and now has more than 2,000, the correct answer is

eg

Explanation for Incorrect Answer E : Choice (E) is incorrect. "Impetuousness" means impulsiveness. If one were to insert this term into the text, the sentence would read "Luisa worked with extreme precision, an impetuousness that served her well in her law career." Work that is done on impulse is probably not performed with extreme precision.

AND EXPLANATIONS

is t

Explanation for Incorrect Answer D : Choice (D) is incorrect. "Litigiousness" means a tendency to engage in lawsuits. If one were to insert this term into the text, the sentence would read "Luisa worked with extreme precision, a litigiousness that served her well in her law career." Although "litigious" is a word that would describe a lawyer, it does not mean extreme precision.

er e

Explanation for Incorrect Answer C : Choice (C ) is incorrect. "Inhibition" means something that holds a person back. If one were to insert this term into the text, the sentence would read "Luisa worked with extreme precision, an inhibition that served her well in her law career." As the sentence makes clear, working precisely allowed Luisa to succeed; it did not hold her back.

file://E:\\c8.htm

2006-11-12

The Official SAT Online Course

5/16

the word that most closely means "increased." However, "dispersed" and "increased" are not similar in meaning.

Explanation for Incorrect Answer B : Choice (B) is incorrect. "To mediate" means to settle a dispute between conflicting parties. If one were to insert this term into the text, the sentence would read "In 1916 Yellowstone National Park had only 25 bison, but the population has since mediated to more than 2,000." Since the park began with 25 bison and now has more than 2,000, the correct answer is the word that most closely means "increased." However, "mediated" and "increased" are not similar in meaning. Mediation does not describe the bison's increase in population.

Explanation for Incorrect Answer C : Choice (C) is incorrect. "To attenuate" means to lessen in amount or force. If one were to insert this term into the text, the sentence would read "In 1916 Yellowstone National Park had only 25 bison, but the population has since attenuated to more than 2,000." Since the park began with 25 bison and now has more than 2,000, the correct answer is the word that most closely means "increased." However, "attenuated" and "increased" are not similar in meaning. What happened to the bison population is the opposite of attenuate.

Explanation for Incorrect Answer E : Choice (E) is incorrect. "To reconcile" means to adapt or make peace. If one were to insert this term into the text, the sentence would read "In 1916 Yellowstone National Park had only 25 bison, but the population has since reconciled to more than 2,000." Since the park began with 25 bison and now has more than 2,000, the correct answer is the word that most closely means "increased." However, "reconciled" and "increased" are not similar in meaning.

(A)

mollified

(B)

lionized

(C) accosted

(D) galvanized

(E)

vilified

nR

ANSWERS

Explanation for Correct Answer B : Choice (B) is correct. "Lionized" means celebrated. If one were to insert this term into the text, the sentence would read "Though surgeon and researcher Charles Drew never enjoyed celebrity, he truly deserves to be lionized for his lifes achievements." This sentence makes sense because it states that Drew deserves to be celebrated for his many achievements.

Explanation for Incorrect Answer A : Choice (A) is incorrect. "Mollified" means soothed. If one were to insert this term into the text, the sentence would read "Though surgeon and researcher Charles Drew never enjoyed celebrity, he truly deserves to be mollified for his lifes achievements." There is no indication in this sentence that Drew is angry or upset, thus it is unlikely that he would deserve or need to be soothed.

Explanation for Incorrect Answer C : Choice (C) is incorrect. "Accosted" means approached aggressively. If one were to insert this term into the text, the sentence would read "Though surgeon and researcher Charles Drew never enjoyed celebrity, he truly deserves to be accosted for his lifes achievements." This sentence is illogical because it does not explain why Drew, a man of many accomplishments, would deserve to be approached aggressively.

Explanation for Incorrect Answer D : Choice (D) is incorrect. "Galvanized" means spurred into action. If one were to insert this term into the text, the sentence would read "Though surgeon and researcher Charles Drew never enjoyed celebrity, he truly deserves to be galvanized for his lifes achievements." It is unclear why a man of many achievements would deserve or need to be spurred into action.

eg is te r
AND EXPLANATIONS

Though surgeon and researcher Charles Drew never enjoyed celebrity, he truly deserves to be ------- for his lifes achievements.

ed

file://E:\\c8.htm

2006-11-12

The Official SAT Online Course


Explanation for Incorrect Answer E : Choice (E) is incorrect. "Vilified" means to be criticized harshly. If one were to insert this term into the text, the sentence would read "Though surgeon and researcher Charles Drew never enjoyed celebrity, he truly deserves to be vilifed for his lifes achievements." If Drew did anything to deserve harsh criticism, this sentence does not mention it.

6/16

The passage below is from a 1991 autobiography that focuses on an African American womans adolescent experiences at a prestigious boarding school. The passage describes one part of a meeting of parents, admissions officers, and prospective students. The story the mother recounts at this meeting took place in 1965.

My mother began to tell a story about a science award I had won in third grade. She started with the winning the long, white staircase in the auditorium, and how the announcer called my name twice because we Line were way at the back and it took me so long to get down 5 those steps. Mamas eyes glowed. She was a born raconteur, able to increase the intensity of her own presence and fill the room. She was also a woman who seldom found new audiences for her anecdotes, so she made herself happy, she insisted, with us children, her mother, her 10 sisters, her grandparentsan entire clan of storytellers competing for a turn on the family stage. This time all eyes were on my mother. Her body, brown and plump and smooth, was shot through with energy. This time the story had a purpose. She told them how my science experiment 15 almost did not get considered in the citywide competition. My thirdgrade teacher, angry that Id forgotten to bring a large box for displaying and storing the experiment, made me pack it up to take home. (Our teacher had told us that the boxes were needed to carry the experiments from our 20 class to the

nR

exhibition room, and shed emphasized that


file://E:\\c8.htm

eg

is t

er

ed

2006-11-12

The Official SAT Online Course

7/16

25

30

40

45

nR

eg

35

she would not be responsible for finding thirty boxes on the day of the fair. Without a box, the experiment would have to go home. Other kids, White kids, had forgotten boxes during the week. Theyd brought boxes the next day. I asked for the same dispensation, but was denied. The next day was the fair, she said. That was different.) I came out of school carrying the pieces of an experiment my father had picked out for me from a textbook. This was a simple buoyancy experiment where I weighed each object in the air and then in water, to prove they weighed less in water. I had with me the scale, a brick, a piece of wood, a bucket, and a carefully lettered poster. Well, my mother marched me and my armload of buoyant materials right back into school and caught the teacher before she left. The box was the only problem? Just the box? Nothing wrong with the experiment? An excited eight year old had forgotten a lousy, stinking box that you can get from the supermarket and for that, she was out of the running? The teacher said I had to learn to follow directions. My mother argued that I had followed directions by doing the experiment by myself, which was more than you could say for third graders whod brought dry-cell batteries that lit light bulbs and papiermch volcanoes that belched colored lava. Dont you ever put me in a position like that again, Mama said when we were out of earshot of the classroom. You never know who is just waiting for an

is

te re

file://E:\\c8.htm

2006-11-12

The Official SAT Online Course

8/16

50

55

65

70

75

nR

eg is

te r

60

excuse to shut us out. We got the box; my experiment went into the fair; I won the prize at school. I won third prize for my age group in the city. When Mama finished her story, my ears began to burn. I could not help but believe that they would see through this transparent plug, and before I had even laid hands on an application. Theyd think we were forward and pushy. I forgot, for the moment, how relieved Id felt when Mama had stood in front of that teacher defending me with a blinding sense of purpose, letting the teacher know that I was not as small and Black and alone as I seemed, that I came from somewhere, and where I came from, shed better believe, somebody was home. The other mothers nodded approvingly. My father gave me a wide, clever-girl smile. The officials from the school looked at me deadpan. They seemed amused by my embarrassment. The story was an answer (part rebuke and part condolence) to the school stories that the admissions people told, where no parents figured at all. It was a message about her maternal concerns, and a way to prove that racism was not some vanquished enemy, but a real, live person, up in your face, ready, for no apparent reason, to mess with your kid. When I was in third grade, Mama could do her maternal duty and face down a White teacher who would have deprived me of an award. Who at this

ed

file://E:\\c8.htm

2006-11-12

The Official SAT Online Course

9/16

new school would stand up for her child in her stead?


7

In line 11, competing portrays the members of the authors family as


(A)

vying for the mothers attention

(B)

feeling eager to tell their own stories

(C) taking issue with each other over household duties

(D) selectively sharing information about their experiences

(E)

comparing educational accomplishments

ANSWERS

AND EXPLANATIONS

Explanation for Correct Answer B : Choice (B) is correct. "Eager" means enthusiastic or impatient. The author describes her family as "an entire clan of storytellers competing for a turn on the family stage" (lines 11-12). This sentence indicates that the entire family was "eager" for a chance to tell their stories on the "family stage."

Explanation for Incorrect Answer D : Choice (D) is incorrect. The passage does not indicate that family members were selective about sharing information, only that they competed eagerly to be heard.

nR
(A)

Explanation for Incorrect Answer E : Choice (E) is incorrect. The passage refers to the family members' desire to have their stories heard, regardless of subject. Further, the only educational accomplishments mentioned are those of the daughter.

The third paragraph (lines 15-27) presents the authors third-grade teacher as being primarily

critical of the authors grandiose ambitions

(B)

disillusioned about her students lack of interest in science

(C) concerned that children would never develop a sense of decorum

(D) arbitrary in implementing rules affecting the class

(E)

bitter and outspoken about injustices in her school district

ANSWERS

Explanation for Correct Answer D : Choice (D) is correct. The author writes, "Other kids, White kids, had forgotten boxes during the week. They'd brought boxes the next day. I asked for the same dispensation, but was denied" (lines 24-26). According to the passage, the author's third grade teacher arbitrarily made allowances for some students (White students in particular) and not for others.

Explanation for Incorrect Answer A : Choice (A) is incorrect. The third paragraph presents the teacher as critical of the author for not having followed directions, not for having overly grand ambitions.

eg

AND EXPLANATIONS

is t

er

Explanation for Incorrect Answer C : Choice (C) is incorrect. There is no mention of household duties or the family's feelings about them in the passage.

ed

Explanation for Incorrect Answer A : Choice (A) is incorrect. The sentence clearly states that the mother was just one of "an entire clan of storytellers" (line 11). There is no indication that anyone was trying to get her attention.

file://E:\\c8.htm

2006-11-12

The Official SAT Online Course

10/16

Explanation for Incorrect Answer B : Choice (B) is incorrect. The paragraph doesn't indicate how the teacher feels about her students' interest in science; it only indicates to what degree and for which students she is willing to bend her rules.

Explanation for Incorrect Answer C : Choice (C) is incorrect. The teacher does not indicate in the third paragraph or anywhere else in the passage that she is concerned about the students' decorum, or manners.

Explanation for Incorrect Answer E : Choice (E) is incorrect. Nowhere in the passage does the teacher mention her feelings about injustice or about the school district. On the contrary, the author suggests that the teacher herself is guilty of injustice.

In line 26, dispensation refers to permission for the author to


(A)

have an additional day to complete the experiment

(B)

bring a container for her experiment the next day

(C) ask her father to help her design the display

(D) leave school early to look for a box

(E)

discuss her experiment with the other children

ANSWERS

AND EXPLANATIONS

Explanation for Incorrect Answer A : Choice (A) is incorrect. The author has already completed the experiment and does not need "an additional day to complete the experiment."

10

nR
(A)

Explanation for Incorrect Answer C : Choice (C) is incorrect. The passage indicates that the dispensation the author is requesting is to bring a box the following day, not permission to ask her father for help.

Explanation for Incorrect Answer D : Choice (D) is incorrect. The author does not ask to leave school early to look for a box, but to bring a box the next day.

Explanation for Incorrect Answer E : Choice (E) is incorrect. The passage indicates that the author wants a "dispensation" to bring a box the following day, not to discuss the experiment with others.

The child most likely intended to use the bucket (line 33) as

an object to counterbalance the weight of another object

(B)

a means of steadying the scale in the experiment

(C) a container to carry the other materials for the project

(D) a receptacle for the water used in the experiment

(E)

a way of transporting liquid from place to place

ANSWERS

Explanation for Correct Answer D : Choice (D) is correct. The author describes the experiment as a "simple buoyancy

eg is

AND EXPLANATIONS

te

Explanation for Correct Answer B : Choice (B) is correct. "Dispensation" means favor or courtesy. The sentences immediately preceding line 26 indicate that the author wished to receive the same "dispensation," the same courtesy, given the "White" children, namely, being allowed to bring a box the next day.

re

d
2006-11-12

file://E:\\c8.htm

The Official SAT Online Course

11/16

experiment" which involves weighing "each object in the air and then in water, to prove they weighed less in water" (lines 30-32). Since the experiment calls for weighing objects in water, the bucket is most likely to be used to hold the water that is needed.

Explanation for Incorrect Answer A : Choice (A) is incorrect. The experiment involves weighing particular objects in air and in water (lines 30-32). The author does not mention the use of objects to counterbalance the weight of other objects.

Explanation for Incorrect Answer B : Choice (B) is incorrect. The author does not mention the need to steady the scale. Further, the piece of wood or the brick from the list would be more appropriate objects to use to steady a scale.

Explanation for Incorrect Answer C : Choice (C) is incorrect. The author's statement that "I came out of school carrying the pieces of an experiment" (lines 28-29) suggests that the author does not have a suitable container to carry the materials for the project. Because one of the main points of the story is that the author does not have a suitable container for her materials, it is clear that the bucket is not being used for this purpose.

11

Lines 34-36 (Well . . . left) portray the mothers attitude of

(B)

weary disappointment

(C) protective generosity

(D) overwhelming dismay

(E)

righteous indignation

nR

ANSWERS

Explanation for Correct Answer E : Choice (E) is correct. "Righteous indignation" suggests just or correct anger. The passage states, "Well, my mother marched me and my armload of buoyant materials right back into the school" (lines 34-35). This shows the mother's actions as determined and forceful; they are part of an angry response to what she sees as an injustice committed against her daughter.

Explanation for Incorrect Answer A : Choice (A) is incorrect. The mother's decisive action in seeking out the teacher indicates not bewilderment but anger and determination.

Explanation for Incorrect Answer B : Choice (B) is incorrect. The mother is not disappointed but angry and determined to rectify the situation. Further, there is no sign that the mother is weary of standing up for her daughter.

Explanation for Incorrect Answer C : Choice (C) is incorrect. The mother is certainly protective of her daughter's interests, but her attitude is not one of generosity. Rather, she shows her determination to right a wrong.

Explanation for Incorrect Answer D : Choice (D) is incorrect. The mother is dismayed but hardly overwhelmed, as her actions in immediately returning to the school to confront the teacher clearly indicate.

eg

AND EXPLANATIONS

is te

(A)

sudden bewilderment

re

Explanation for Incorrect Answer E : Choice (E) is incorrect. The experiment calls for weighing objects in water; this does not require liquid to be transported from place to place.

file://E:\\c8.htm

2006-11-12

The Official SAT Online Course

12/16

12

The mother most probably intended the questions in lines 36-40 to


(A)

underscore the absurdity of the teachers position

(B)

request clarification from the child about the incident

(C) express concern over her daughters forgetfulness

(D) lessen the childs preoccupation with how her project would be received

(E)

help herself understand her childs defensiveness about the box issue

ANSWERS

AND EXPLANATIONS

Explanation for Correct Answer A : Choice (A) is correct. The mother's questions, "The box was the only problem? Just the box? Nothing wrong with the experiment? An excited eight year old had forgotten a lousy, stinking box" and "for that she was out of the running?" (lines 36-40), are part of her doubting query. The mother clearly believes that the teacher has placed undue importance on the box. Her questions are intended to highlight the "absurdity of the teacher's position."

13

nR
(A)

Explanation for Incorrect Answer E : Choice (E) is incorrect. The mother does not need help understanding her child's defensiveness about the box, since her questions indicate that she finds the issue ridiculous.

Between the mention of a hypothetical box in line 23 and its characterization in line 38, the box has changed from a

requirement to something that is no longer needed

(B)

necessity to something that has little inherent value

(C) diversion to something that requires a desperate search

(D) tool to something that is a source of entertainment

(E)

puzzle to something that provides clarity and strength

ANSWERS

Explanation for Correct Answer B : Choice (B) is correct. In line 23, the box as the teacher sees it is a necessary element for the presentation of the experiment at the science fair. In line 38, it is, as the mother sees it, a "lousy, stinking box that you can get from the supermarket," and its degree of importance has shifted from necessary to minor.

Explanation for Incorrect Answer A : Choice (A) is incorrect. The box is still a requirement, as indicated by the fact that the mother and daughter find a box for the experiment materials before the experiment proceeds to the fair.

eg

Explanation for Incorrect Answer D : Choice (D) is incorrect. The questions are being asked of the teacher. The mother is more interested in securing the author's place in the competition than in reassuring the author about the project's reception.

AND EXPLANATIONS

is

te

Explanation for Incorrect Answer C : Choice (C) is incorrect. The mother's disdain for the idea that a box alone should keep her daughter's experiment out of the competition indicates that she is not concernedor at least not interested in expressing concern to the teacherabout her daughter's forgetfulness.

re

Explanation for Incorrect Answer B : Choice (B) is incorrect. The questions are being put to the teacher and not to the child. This is clearly shown by the continuing dialogue which occurs between mother and teacher in lines 40-45.

file://E:\\c8.htm

2006-11-12

The Official SAT Online Course

13/16

Explanation for Incorrect Answer C : Choice (C) is incorrect. Neither the teacher nor the author considered the box a "diversion," or something that is amusing, and it does not later require a "desperate search." As the mother points out, a box can be found almost anywhere (line 39).

Explanation for Incorrect Answer D : Choice (D) is incorrect. The box is never a tool, but a required carrier for the experiment. At no point is the box considered a source of entertainment for anyone involved.

Explanation for Incorrect Answer E : Choice (E) is incorrect. The box is never presented as either a puzzle or as something that provides clarity and strength. It is a requirement that later becomes, as the mother indicates, a petty impediment to her daughter's potential success.

14

In line 55, the author uses the word plug primarily to emphasize her feeling that
(A)

the conversational void was nearly intolerable

(B)

the boarding school had been highly overrated

(C) her mother had gone too far in promoting her

(D) her mothers words and actions were entirely at odds

ANSWERS

AND EXPLANATIONS

U
15

nR
(A)

Explanation for Incorrect Answer A : Choice (A) is incorrect. The author is not embarrassed because of a pause in conversation; she is embarrassed because her mother is openly promoting her.

Explanation for Incorrect Answer B : Choice (B) is incorrect. In this passage, the author does not indicate what her feelings are about the boarding school.

Explanation for Incorrect Answer D : Choice (D) is incorrect. The author feels that her mother's words and actions are in harmony; both are intended to promote and protect her daughter.

Explanation for Incorrect Answer E : Choice (E) is incorrect. The author uses the word "plug" to describe her feelings about what she sees as her mother's shameless promotion. She worries that this promotion will have the unintended effect of causing the interviewers not to praise her, sincerely or otherwise, but to see her family as "forward and pushy" (line 56).

In line 59, blinding suggests all of the following EXCEPT

unswerving

(B)

dazzling

(C) overpowering

(D) determined

(E)

sudden

ANSWERS

Explanation for Correct Answer E :

eg

AND EXPLANATIONS

is te

Explanation for Correct Answer C : Choice (C) is correct. In this context, the word "plug" means promote or boost. The author uses the word "plug," a word associated with selling a product, to express her sense that her mother is overpromoting her.

re d

(E)

the interviewers praise would prove to be insincere

file://E:\\c8.htm

2006-11-12

The Official SAT Online Course

14/16

Choice (E) is correct because "blinding" is not used to mean "sudden" in this context. We can infer that the mother's instinct is always to defend her daughter and that her sense of purpose can therefore not be described as "sudden."

Explanation for Incorrect Answer A : Choice (A) is incorrect because the mother's "blinding" sense of purpose can be described as "unswerving," or steady.

Explanation for Incorrect Answer B : Choice (B) is incorrect because the mother's "blinding" sense of purpose was sufficiently effective to overrule, and thus stun or dazzle, the teacher.

Explanation for Incorrect Answer C : Choice (C) is incorrect because "overpowering" is a word that can be used to describe the mother's "blinding" sense of purpose. Indeed, her argument overpowers the teacher's objections.

Explanation for Incorrect Answer D : Choice (D) is incorrect because the mother's "blinding" sense of purpose can certainly be described as "determined."

(A)

desire to create a comfortable life for her family

(B)

eagerness to learn the results of the interview

(D) need to capture everyones full attention

(E)

sense of responsibility toward her daughter

ANSWERS

17

nR

Explanation for Correct Answer E : Choice (E) is correct. The phrase "somebody was home" is part of a sentence in which the author describes her mother defending her. For the author, her mother's vigorous defense is a way of conveying that the author is important to and supported by her family. This clearly captures the mother's "sense of responsibility toward her daughter."

Explanation for Incorrect Answer A : Choice (A) is incorrect. Undoubtedly the mother wants to create a comfortable life for her family, but the phrase in line 62 refers specifically to the mother's support and defense of her daughter.

Explanation for Incorrect Answer B : Choice (B) is incorrect. Although we can assume that the mother was eager to learn the results of the interview, the passage describes only part of the interview. The passage does not discuss the results of the interview or the feelings of the participants as to those results. Further, the phrase in line 62 refers to an earlier event, not to the interview.

Explanation for Incorrect Answer C : Choice (C) is incorrect. The mother's story indicates that she feels not despair but determination to confront injustice toward her daughter.

Explanation for Incorrect Answer D : Choice (D) is incorrect. Although the passage indicates elsewhere that the mother enjoys attention, the phrase "somebody was home" refers specifically to her defense of her daughter.

The details presented in lines 63-66 primarily serve to

eg is

AND EXPLANATIONS

te r

(C) despair over the unfairness of the teachers instructions

ed

16

The phrase somebody was home (line 62) captures the mothers

file://E:\\c8.htm

2006-11-12

The Official SAT Online Course

15/16

(A)

illustrate the various reactions to the mothers story

(B)

satirize the pomposity of the school officials

(C) emphasize the sense of unity among parents of prospective students

(D) convey a sense of unease

(E)

clarify a preceding statement

ANSWERS

AND EXPLANATIONS

Explanation for Correct Answer A : Choice (A) is correct. These lines indicate the various reactions, ranging from approval to amusement, of the story's listeners: "[t]he other mothers nodded approvingly. My father gave me a wide, clever-girl smile. The officials from the school looked at me deadpan" (lines 63-65).

Explanation for Incorrect Answer B : Choice (B) is incorrect. The details in lines 63-66 are straightforward rather than satirical. The author interprets the school officials' "deadpan" reaction as amused, not pompous or arrogant.

Explanation for Incorrect Answer E : Choice (E) is incorrect. Lines 63-66 do not clarify a previous statement; instead, they indicate the listeners' reactions to the mother's story.

nR
(A)

18

The final paragraph presents the authors assessment of her mothers story as a

pronouncement about forgiveness and understanding

(B)

lesson and forewarning for school officials

(C) personal argument for the importance of hard work

(D) defense of a theory about social attitudes

(E)

parting message to her daughter

ANSWERS

Explanation for Correct Answer B : Choice (B) is correct. In the final paragraph, the author calls her mother's story "a message about her maternal concerns, and a way to prove that racism was not some vanquished enemy" (lines 69-71). She adds, "[w]hen I was in third grade, Mama could do her maternal duty" (lines 73-74), and asks, "[w]ho at this new school would stand up for her child in her stead?" (75-76). The author sums up the purpose of her mother's story as a challenge to the school officials to protect her daughter as fiercely as she has done and will, if necessary, continue to do.

Explanation for Incorrect Answer A : Choice (A) is incorrect. The author indicates that her mother's story was "an answer (part rebuke and part condolence) to the school stories that the admissions people told, where no parents figured at all" (lines 67-69). Forgiveness and understanding do not figure prominently in the mother's story.

Explanation for Incorrect Answer C :

eg
AND EXPLANATIONS

is

te

Explanation for Incorrect Answer D : Choice (D) is incorrect. Although the author expresses embarrassment, the point of lines 63-66 is not to show her unease, but to show the reactions of everyone in the room to her mother's story.

re

Explanation for Incorrect Answer C : Choice (C) is incorrect. Although the other mothers nod in approval, possibly signaling a sense of unity with the author's mother, lines 63-66 also stress how the author herself and the school officials reacted to the mother's story.

file://E:\\c8.htm

2006-11-12

The Official SAT Online Course

16/16

Choice (C) is incorrect. The author sees the story in terms of her mother's determination to defend her daughter rather than as her mother's way of conveying her personal ideas on hard work.

Explanation for Incorrect Answer D : Choice (D) is incorrect. Although the author describes a theory about a particular social attitudeone that might imagine that "racism was not some vanquished enemy, but a real, live person, up in your face, ready, for no apparent reason, to mess with your kid" (lines 71-73)this theory is one part of her mother's story. The author sees the story as her mother's way of expressing her reaction to this theory rather than as a defense of the theory.

Explanation for Incorrect Answer E : Choice (E) is incorrect. As the author sees it, the message of the mother's story is intended for the school officials and not for her daughter.

19

A central purpose of the passage is to


(A)

illustrate the character of the authors mother

(B)

portray the admissions process for boarding schools at that time

(C) show the authors repressed hostility toward her mother

(D) comment on examples of racism in the United States

(E)

reveal how the author became skeptical of human nature

ANSWERS

AND EXPLANATIONS

nR

Explanation for Incorrect Answer C : Choice (C ) is incorrect. Although the author feels embarrassed by her mother's story, the passage does not suggest that she feels hostility, repressed or otherwise, toward her mother. Rather, she feels admiration and thanks for her mother and her actions.

Explanation for Incorrect Answer D : Choice (D) is incorrect. The mother's story indicates that she is well aware of racism; however, this is not the central purpose of the passage. The central purpose of the passage is to describe the mother's response to life and to her family.

Explanation for Incorrect Answer E : Choice (E) is incorrect. The author does not indicate or suggest that she has become skeptical of human nature.

eg

Explanation for Incorrect Answer B : Choice (B) is incorrect. The boarding school admissions process is barely mentioned in the passage.

is

te

Explanation for Correct Answer A : Choice (A) is correct. The main point of the passage is to show what kind of person the author's mother is.

re d
Privacy Policy

Back to Score Report

Copyright 2006 The College Board. All rights reserved.

Terms of Use

Contact Us

file://E:\\c8.htm

2006-11-12

The Official SAT Online Course

1/13

Help | Profile | My Organizer | My Bookmarks | Logout

Answers and Explanations

Test Sections

Back to Score Report

Section 1

View Answers and Explanations


Online - Practice Test #3

Section 2

Section 3

Section 4

If
(A)

what is the value of

Section 5

Section 7

Section 8

(B)

Section 9

Section 10

(C)

(D)
(E)

Choice (B) is correct. If

then

Explanation for Incorrect Answer A :

eg is
not

te
Choice (A) is not correct. If

re

Explanation for Correct Answer B :

d
which simplifies to

ANSWERS

AND EXPLANATIONS

were equal to

then the expression

would

equal

nR

Explanation for Incorrect Answer C :

Choice (C) is not correct. If

were equal to

then the expression

would

equal

not

Explanation for Incorrect Answer D :

Choice (D) is not correct. If

were equal to

then the expression

would

be undefined, and not equal to

Explanation for Incorrect Answer E :

Choice (E) is not correct. If

were equal to

then the expression

would

equal

not

Which of the following is the best estimate of the length of segment number line above?

on the

file://E:\\c9.htm

2006-11-12

The Official SAT Online Course

2/13

(A)

(B)
(C)

(D)
(E)

ANSWERS

AND EXPLANATIONS
and

Explanation for Correct Answer A : on the number line has a value of about Choice (A) is correct. Point

point

has a value of about

. The length of

is approximately

Explanation for Incorrect Answer B : on the number line has a value of about Choice (B) is not correct. Point

and point

has a value of about

. While

is close to the length of

the value

is a better estimate.

re
and

and point

has a value of about

. The length of

Explanation for Incorrect Answer C : on the number line has a value of about Choice (C) is not correct. Point

is greater than the

between the points labeled distance of better estimate.

on the number line, so

is a

Explanation for Incorrect Answer D : is to the left of Choice (D) is not correct. Since point

is te

and point

is to the

the distance between the two points must be at least right of better estimate.

so

is a

nR eg

Explanation for Incorrect Answer E : is to the left of Choice (E) is not correct. Since point

and point

is to the

the distance between the two points must be at least right of better estimate.

so

is a

For which of the following lists of than the median?


(A)

numbers is the average (arithmetic mean) less

(B)
(C)
(D)

(E)

ANSWERS

AND EXPLANATIONS
numbers are

Explanation for Correct Answer A : Choice (A) is correct. The median is the middle number when the The average is listed in order, so the median is

The question asks for a list of numbers

for which the average is less than the median, and

is less than

Explanation for Incorrect Answer B : Choice (B) is not correct. The median is the middle number when the

numbers

file://E:\\c9.htm

2006-11-12

The Official SAT Online Course

3/13

are listed in order, so the median is

The average is

For this list, the average is equal to the

median.

Explanation for Incorrect Answer C : Choice (C) is not correct. The median is the middle number when the The average is are listed in order, so the median is

numbers

For this list, the average is equal to the

median.

Explanation for Incorrect Answer D : Choice (D) is not correct. The median is the middle number when the The average is are listed in order, so the median is

numbers

For this list, the average is equal to the

median.

Explanation for Incorrect Answer E : Choice (E) is not correct. The median is the middle number when the The average is are listed in order, so the median is

numbers

For this list, the mean is greater than

the median.

nR eg
(A)

is
If the perimeter of the rectangle above is

(B)

(C)

(D)

(E)

ANSWERS

AND EXPLANATIONS

Explanation for Correct Answer C : Choice (C) is correct. The formula for the perimeter of this rectangle is is the width. Therefore, is the length and where

so

Explanation for Incorrect Answer A : Choice (A) is not correct. This would be the value of rectangle rather than the perimeter.

te re
what is the value of
and

Explanation for Incorrect Answer B : were equal to Choice (B) is not correct. If

d
if

were the area of the

then the perimeter of the

rectangle would be

This cannot be

the case since it is given that the perimeter is

Explanation for Incorrect Answer D : were equal to Choice (D) is not correct. If

then the perimeter of the

file://E:\\c9.htm

2006-11-12

The Official SAT Online Course

4/13

rectangle would be

This cannot be

the case since it is given that the perimeter is

Explanation for Incorrect Answer E : were equal to Choice (E) is not correct. If

then the perimeter of the

rectangle would be

This cannot

be the case since it is given that the perimeter is

For which of the following functions is it true that


(A)

(B)

(C)

(D)
(E)

ANSWERS

AND EXPLANATIONS

Explanation for Correct Answer D :

Explanation for Incorrect Answer A :

eg i

st er e
This is the only choice where

Choice (D) is correct. For this function,

d
and

Choice (A) is not correct. For this function,

and

so

nR

Explanation for Incorrect Answer B :

Choice (B) is not correct. For this function,

and

so

Explanation for Incorrect Answer C :

Choice (C) is not correct. For this function,

and

so

Explanation for Incorrect Answer E :

Choice (E) is not correct. For this function,

and

so

Wayne would like to buy a school jacket priced at

but the price of the jacket is

more than he has. In which of the following equations does number of dollars Wayne has?
(A)

represent the

(B)

(C)
(D)
(E)

file://E:\\c9.htm

2006-11-12

The Official SAT Online Course

5/13

ANSWERS

AND EXPLANATIONS
jacket costs

Explanation for Correct Answer D :

Choice (D) is correct. Since the

more than Wayne has,

Wayne has

dollars. Thus, if

represents the number of dollars Wayne

has, then

or

Explanation for Incorrect Answer A : would have to be a Choice (A) is not correct. For this equation to be true negative number. However, Wayne has a positive amount of money.

Explanation for Incorrect Answer B : Choice (B) is not correct. In this equation,

Wayne would have if the price of the jacket were

represents the number of dollars less than the amount Wayne

has. However, the jacket costs

more than the amount of money that Wayne

has.

Explanation for Incorrect Answer C : would have to be a Choice (C) is not correct. For this equation to be true negative number. However, Wayne has a positive amount of money.

Wayne would have if the price of the jacket were

has. However, the jacket costs

more than the amount of money that Wayne

has.

eg is
and

te r
Which of the following must be true for all integers
I.

II.

nR

III.

(A)

I only

(B)

II only

(C) III only

(D) I and II

(E)

II and III

ANSWERS

AND EXPLANATIONS
will always result in the does not hold for

Explanation for Correct Answer A : Choice (A) is correct. Subtracting zero from any integer so I must be true. On the other hand, integer

ed
and

Explanation for Incorrect Answer E : Choice (E) is not correct. In this equation,

represents the number of dollars less than the amount Wayne

and

all integers. For example, if

while

so II is not true. Also,

does not hold

for all integers. For example, if

then

while

so

III is not true.

Explanation for Incorrect Answer B : Choice (B) is not correct. II is not true for all integers. For example, if

and

then

while

file://E:\\c9.htm

2006-11-12

The Official SAT Online Course

6/13

Explanation for Incorrect Answer C : Choice (C) is not correct. III is not true for all integers. For example, if

and

then

while

Explanation for Incorrect Answer D : Choice (D) is not correct. I is true, but II is not true for all integers. For example, if while then and

Explanation for Incorrect Answer E : Choice (E) is not correct. Neither II nor III is true for all integers. For example, if and while then and

while

If
(A)
(B)

and if

and

are positive numbers, then

(E)

eg

Explanation for Correct Answer D :

is

ANSWERS

AND EXPLANATIONS
and

te
were equal to

Choice (D) is correct. If

nR

Explanation for Incorrect Answer A :

re
and

(D)

d
are positive, then

(C)

Choice (A) is not correct. If

then

would be equal to

which is not the same expression as the given

Explanation for Incorrect Answer B :

Choice (B) is not correct. If

were equal to

then

would be equal to

which is not the same expression as the given

Explanation for Incorrect Answer C :

Choice (C) is not correct. If

were equal to

then

would be equal to

which is not the same expression as the given

Explanation for Incorrect Answer E :

Choice (E) is not correct. If

were equal to

then

would be equal to

which is not the same expression as the given

file://E:\\c9.htm

2006-11-12

The Official SAT Online Course

7/13

On a map, the length of the road from Town

to Town

is measured to be

inches. On this map,

inch represents an actual distance of

miles. What is the

actual distance, in miles, from Town


(A)
(B)
(C)
(D)
(E)

to Town

along this road?

ANSWERS

AND EXPLANATIONS

Explanation for Correct Answer A :

Choice (A) is correct. Since

inch represents

miles,

inch represents

miles. Therefore,

inches represent

miles.

Explanation for Incorrect Answer B :

miles. If the actual distance were

st er
or

Choice (B) is not correct. Since

inch represents

ed
miles,

inch represents

miles, it would be represented

on the map by

inches, which is less than

inches, the

value given in the problem.

Explanation for Incorrect Answer C :

eg i
or

nR

Choice (C) is not correct. Since

inch represents

miles,

inch represents

miles. If the actual distance were

miles, it would be represented

on the map by

inches, which is less than

inches, the value given in

the problem.

Explanation for Incorrect Answer D :

Choice (D) is not correct. Since

inch represents

miles,

inch represents

miles. If the actual distance were

miles, it would be represented

on the map by

inches, which is less than

inches, the

value given in the problem.

Explanation for Incorrect Answer E :

Choice (E) is not correct. Since

inch represents

miles,

inch represents

miles. If the actual distance were

miles, it would be represented

on the map by

or

inches, which is less than

inches, the

value given in the problem.

file://E:\\c9.htm

2006-11-12

The Official SAT Online Course

8/13

10

In the figure above, if the area of triangle

is equal to the area of rectangle

what is the length of segment

(A)

(B)

(C)

(D)
(E)

eg

Explanation for Correct Answer C : Choice (C) is correct. The area of rectangle

is

ANSWERS

AND EXPLANATIONS
is

te

re
The area of a

d
where

triangle is

is the length of the base of the triangle and

is the

height of the triangle. Therefore, the area of triangle

is equal to

nR

Since the areas of the triangle and the rectangle are the same,

This simplifies to

Thus, line segment

has length

Explanation for Incorrect Answer A : Choice (A) is not correct. The area of rectangle

is

The area of

triangle

is equal to

If the length of line segment

were equal to

then the area of the triangle would equal

not

Explanation for Incorrect Answer B : Choice (B) is not correct. The area of rectangle

is

The area of

triangle

is equal to

If the length of line segment

were equal to

then the area of the triangle would equal

not

Explanation for Incorrect Answer D : Choice (D) is not correct. The area of rectangle

is

The area of

triangle

is equal to

If the length of line segment

were equal to

then the area of the triangle would equal

not

file://E:\\c9.htm

2006-11-12

The Official SAT Online Course

9/13

Explanation for Incorrect Answer E : Choice (E) is not correct. The area of rectangle

is

The area of triangle

is equal to

If the length of line segment

were equal

to

then the area of the triangle would equal

not

11

Six points are placed on a circle. What is the greatest number of different lines that can be drawn so that each line passes through two of these points?
(A)
(B)
(C)
(D)
(E)

ANSWERS

AND EXPLANATIONS

nR U

Explanation for Incorrect Answer A : different lines so that Choice (A) is not correct. It is possible to draw more than lines that can each line passes through two of the points. The figure shows the be drawn.

Explanation for Incorrect Answer C : Choice (C) is not correct. There are only the figure.

eg

Explanation for Incorrect Answer D : Choice (D) is not correct. You can draw

is te r
lines that can be drawn, as shown in

ed

Explanation for Correct Answer B : Choice (B) is correct. Five lines can be drawn from each point through another point different lines from the first point, but only on the circle. You can draw different lines from the second point, since one of the lines to that point has already been drawn. Thus, the greatest number of lines that can be drawn so that each line passes through two of these points is

lines from each of the

points, but

since the line through a point

and another point,

is the same as the line

and point through point as shown in the figure.

there are only

different lines that can be drawn,

Explanation for Incorrect Answer E : Choice (E) is not correct. There are only shown in the figure.

different lines that can be drawn, as

12

file://E:\\c9.htm

2006-11-12

The Official SAT Online Course

10/13

is the point with the greatest Point What is the -coordinate of point
(A)
(B)

-coordinate on the semicircle shown above.

(C)
(D)

(E)

ANSWERS

AND EXPLANATIONS
-coordinate on the semicircle,

Explanation for Correct Answer C : has the greatest Choice (C) is correct. Since

the radius of the circle equals the

-coordinate of

which is equal to

The

center of the circle has the same

-coordinate as

so the center has coordinates

The distance between the center of the circle and

is the radius of the

circle, so the

-coordinate of

is equal to

Explanation for Incorrect Answer A :

re
were

Choice (A) is not correct. The center of the circle has coordinates

and the

circle has radius

If the

-coordinate of

then the distance

between the center of the circle to the point

on the circle would be equal to

te

which is greater than the radius. Therefore, the

-coordinate of

cannot be

equal to

Explanation for Incorrect Answer B :

is

Choice (B) is not correct. The center of the circle has coordinates

and the

nR eg
circle has radius

If the

-coordinate of

were

then the distance between

the center of the circle to the point

on the circle would be equal to

which is

greater than the radius. Therefore, the

-coordinate of

cannot be equal to

Explanation for Incorrect Answer D :

Choice (D) is not correct. The center of the circle has coordinates

and the

circle has radius

If the

-coordinate of

were

then the distance between

the center of the circle to the point

on the circle would be equal to

which is

less than the radius. Therefore, the

-coordinate of

cannot be equal to

Explanation for Incorrect Answer E :

Choice (E) is not correct. The center of the circle has coordinates

and the

circle has radius

If the

-coordinate of

were

then the distance

between the center of the circle to the point

on the circle would be equal to

which is less than the radius. Therefore, the

-coordinate of

cannot be equal to

13

plants are plants in each row. If parallel rows of plants, with A garden has and added to each row, how many plants will then be in the garden, in terms of
(A)

(B)

(C)

(D)

(E)

file://E:\\c9.htm

2006-11-12

The Official SAT Online Course

11/13

ANSWERS

AND EXPLANATIONS

Explanation for Correct Answer C : Choice (C) is correct. The total number of plants will be the number of rows,

multiplied by the total number of plants per row,

which gives

Explanation for Incorrect Answer A : Choice (A) is not correct. This expression corresponds to a situation in which the The question asks for an expression number of plants per row is multiplied by plants to each row of the garden. that corresponds to adding

Explanation for Incorrect Answer B : Choice (B) is not correct. This expression corresponds to the situation in which plants are added to the entire garden. The question asks for an expression that plants to each row of the garden. corresponds to adding

Explanation for Incorrect Answer D : Choice (D) is not correct. This expression corresponds to the situation in which plants each are added to the garden. The question asks for an rows with plants to each row of the garden. expression that corresponds to adding

Explanation for Incorrect Answer E : Choice (E) is not correct. The number of plants per row in the garden,

needs to

not involve this necessary multiplication of

(A)

Three

(B)

Four

(C) Five

nR
(E)

(D) Six

Seven

ANSWERS

Explanation for Correct Answer E : Choice (E) is correct. Since there are three different intersection points, the three lines form a triangle. They also separate the remaining space in the plane into six regions, for a total of seven non-overlapping regions. See figure below.

Explanation for Incorrect Answer A : Choice (A) is not correct. The lines separate the plane into more than three regions. The figure shows the plane divided into seven regions.

Explanation for Incorrect Answer B : Choice (B) is not correct. The lines separate the plane into more than four regions. The figure shows the plane divided into seven regions.

Explanation for Incorrect Answer C :

eg
AND EXPLANATIONS

is

14

Three lines are drawn in a plane so that there are exactly three different intersection points. Into how many non-overlapping regions do these lines divide the plane?

te re
and

to get the total number of plants in the be multiplied by the number of rows, does new plants are added to each row. The expression garden before

file://E:\\c9.htm

2006-11-12

The Official SAT Online Course

12/13

Choice (C) is not correct. The lines separate the plane into more than five regions. The figure shows the plane divided into seven regions

Explanation for Incorrect Answer D : Choice (D) is not correct. The lines separate the plane into more than six regions. The figure shows the plane divided into seven regions.

15

In the figure above, side

of

is on line

What is

in terms of

(A)

(B)

(C)

(E)

nR eg

Explanation for Correct Answer A : Choice (A) is correct. The measure of

is
so

ANSWERS

AND EXPLANATIONS
is

te r
and the

(D)

measure of

is

Explanation for Incorrect Answer B :

ed
Therefore,

and

Choice (B) is not correct. If

then

which means

that

must be

and that

must be a

right triangle.

However, there are other values of

that satisfy the conditions given in the figure.

Explanation for Incorrect Answer C :

Choice (C) is not correct. If

then

which means that

must be

and that

must be an equilateral triangle.

However, there are other values of

that satisfy the conditions given in the figure.

Explanation for Incorrect Answer D :

Choice (D) is not correct. If

then

which is equivalent to the false statement

Therefore,

cannot equal

Explanation for Incorrect Answer E : Choice (E) is not correct. If

then

which means that

must be

and

but from the figure,

is positive. Therefore,

cannot equal

16

file://E:\\c9.htm

2006-11-12

The Official SAT Online Course

13/13

If

denotes the sum of the integers from

to

inclusive, and

denotes the sum

of the integers from


(A)

to

inclusive, what is the value of

(B)

(C)
(D)
(E)

ANSWERS

AND EXPLANATIONS
and

Explanation for Correct Answer E : Choice (E) is correct. If

then

and

Another way to solve this

problem is to notice that

is the sum

of

numbers each of which is equal to

so

equals

Explanation for Incorrect Answer A : and Choice (A) is not correct. Since

it is not true that

Explanation for Incorrect Answer B : and Choice (B) is not correct. Since

eg

Explanation for Incorrect Answer D : and Choice (D) is not correct. Since

is t

Explanation for Incorrect Answer C : and Choice (C) is not correct. Since

er

nR

ed
Privacy Policy

it is not true that

it is not true that

it is not true that

Back to Score Report

Copyright 2006 The College Board. All rights reserved.

Terms of Use

Contact Us

file://E:\\c9.htm

2006-11-12

The Official SAT Online Course

1/9

Help | Profile | My Organizer | My Bookmarks | Logout

Answers and Explanations

Test Sections

Back to Score Report

Section 1

View Answers and Explanations


Online - Practice Test #3

Section 2

Section 3

Section 4

A recent discovery is the finding that people who both drink and smoke are greater cancer risks than those who do only one of these things.
(A)

Section 5

Section 7

A recent discovery is the finding that

Section 8

(B)

A recent discovery came when they learned that

Section 9

(C) Recently, a finding is that

Section 10

(D) It has recently been discovered that

(E)

It is a recent discovery that

ANSWERS

AND EXPLANATIONS

Explanation for Correct Answer D : Choice (D) is correct. It avoids the error of the original by eliminating the unnecessary phrase "the finding."

Explanation for Incorrect Answer B : Choice (B) uses a vague pronoun. The sentence contains no noun to which the pronoun "they" can refer.

U
2

nR
(A)

Explanation for Incorrect Answer C : Choice (C) involves an inappropriate idiom. In context, the adverb "recently" is less effective than the adjective "recent" ("A recent finding is that").

Explanation for Incorrect Answer E : Choice (E) contains an unnecessary word. With a minor change in word order ("A recent discovery is that"), the vague pronoun "it" can be deleted.

The notion that a biography should be full of praise and free of criticism prevailed during most of the nineteenth century.

The notion that a biography should be full of praise and free of criticism prevailed during most of the nineteenth century.

(B)

The notion that prevailed about a biography during most of the nineteenth century was that of being full of praise and free of criticism.

(C) During most of the nineteenth century, they had a prevalent notion that a biography should be full of praise and free of criticism.

(D) Prevalent as a notion during most of the nineteenth century was for a biography to be full of praise and free of criticism.

(E)

Prevalent during most of the nineteenth century, the notion prevailed that a biography should be full of praise and free of criticism.

ANSWERS

Explanation for Correct Answer A : Choice (A) is correct. It uses appropriate words to express a complete idea.

eg is

Explanation for Incorrect Answer A : Choice (A) displays wordiness. The phrase "the finding" is not needed, since it merely repeats the idea of the earlier noun "discovery."

AND EXPLANATIONS

te

re d

file://E:\\c10.htm

2006-11-12

The Official SAT Online Course

2/9

Explanation for Incorrect Answer B : Choice (B) uses an inappropriate idiom. The phrase "that of being" is not suitable to introduce a phrase describing the prevalent notion.

Explanation for Incorrect Answer C : Choice (C) includes a vague pronoun. Since the sentence contains no noun to which the pronoun "they" can refer, its meaning is not clear.

Explanation for Incorrect Answer D : Choice (D) creates a sentence fragment. Without a noun, a phrase, or a clause to serve as the subject of the verb "was," this group of words does not state a complete thought.

Explanation for Incorrect Answer E : Choice (E) displays wordiness. The adjective "prevalent" is not needed, since it conveys the same idea as the verb "prevailed."

A native New Yorker, Gloria Naylors first novel won an American Book Award in 1983.
(A)

A native New Yorker, Gloria Naylors first novel

(B)

A native New Yorker, the first novel by Gloria Naylor

(D) Gloria Naylor, a native New Yorker, wrote her first novel thus having

(E)

Gloria Naylor wrote her first novel and the native New Yorker

ANSWERS

AND EXPLANATIONS

Explanation for Correct Answer C : Choice (C) is correct. It avoids the dangling modifier of the original by using a correctly placed prepositional phrase ("by native New Yorker Gloria Naylor") to embed details describing the "first novel."

nR
(A)

Explanation for Incorrect Answer A : Choice (A) contains a dangling noun phrase. Although the two noun phrases ("A native New Yorker" and "Gloria Naylors first novel") may appear at first to be parallel appositives, the first phrase refers to a person and the second to that person's work. Thus, the first phrase has no grammatical connection to the rest of the sentence.

Explanation for Incorrect Answer B : Choice (B) displays improper word order. Since the noun phrase "A native New Yorker" obviously refers to a person (not to that person's "first novel"), the phrase should be placed immediately before or after the name "Gloria Naylor."

Explanation for Incorrect Answer D : Choice (D) uses improper tense sequence. The present perfect tense of the verbal "having won" illogically suggests that Naylor received the award before she wrote the novel.

Explanation for Incorrect Answer E : Choice (E) exhibits ineffective coordination. The coordinating conjunction "and" merely links two complete thoughts without appropriate subordination or embedding to indicate their relationship and relative importance.

Charlie Chaplin developed definite ideas about the art of comedy and as a result sentiment, satire, and social criticism were introduced into his work.

sentiment, satire, and social criticism were introduced

(B)

sentiment, satire, and social criticism were introduced by him

(C) having introduced sentiment, satire, and social criticism

eg

is te r

ed

(C) The first novel by native New Yorker Gloria Naylor

file://E:\\c10.htm

2006-11-12

The Official SAT Online Course

3/9

(D) introduced sentiment, satire, and social criticism

(E)

the introduction of sentiment, satire, and social criticism

ANSWERS

AND EXPLANATIONS

Explanation for Correct Answer D : Choice (D) is correct. It avoids the error of the original by using two parallel active verbs, "developed" and "introduced," with "Charlie Chaplin" as their subject.

Explanation for Incorrect Answer A : Choice (A) fails to maintain parallelism. The first independent clause uses active voice ("Charlie Chaplin developed"), but the second shifts awkwardly to passive voice ("were introduced").

Explanation for Incorrect Answer B : Choice (B) uses excess words. The phrase "by him" is not needed.

Explanation for Incorrect Answer C : Choice (C) exhibits improper modification. The phrase "having introduced sentiment, satire, and social criticism" cannot coherently modify any part of the sentence, and it incorrectly suggests that this introduction came before the development of Chaplin's ideas about comedy.

Someone living in a technological, consumption-oriented culture probably taxes the environment at a rate many times that of a country such as Myanmar.

(B)

that of someone living in a country like Myanmar

(C) what you find in Myanmar, for instance

nR
(E)

(D) the rate in a country such as Myanmar

a citizen of Myanmar, for instance

ANSWERS

Explanation for Correct Answer B : Choice (B) is correct. It avoids the error of the original by adding the phrase "someone living in" so that the effects of the actions of two individuals in two different countries are compared.

Explanation for Incorrect Answer A : Choice (A) presents an illogical comparison. The effect of a single person ("someone") on the environment is compared to the effect of an entire country ("Myanmar").

Explanation for Incorrect Answer C : Choice (C) has an inappropriate pronoun. The pronoun "you" is used loosely to mean any person.

Explanation for Incorrect Answer D : Choice (D) suggests an illogical comparison. The rate at which an individual taxes the environment in a technological culture is compared to the rate in the whole country of Myanmar.

Explanation for Incorrect Answer E : Choice (E) makes an illogical comparison. Without the phrase "that of" immediately before "a citizen of Myanmar," the statement compares a rate (of taxing the

eg

(A)

that of a country such as Myanmar

AND EXPLANATIONS

is te

re

Explanation for Incorrect Answer E : Choice (E) violates parallelism. Since the noun "introduction" is not parallel with the earlier clause, "Charlie Chaplin developed definite ideas," the second half of the sentence has no verb and is left awkwardly incomplete.

file://E:\\c10.htm

2006-11-12

The Official SAT Online Course

4/9

environment) with a person.

Airport runways must be constantly swept clear of trash and other debris that could be sucked into a jet-engine intake or it could cause a serious accident.
(A)

intake or it could cause a serious accident

(B)

intake, this causes a serious accident

(C) intake and cause a serious accident

(D) intake, preventing a serious accident

(E)

intakes and avoiding a serious accident

ANSWERS

AND EXPLANATIONS

Explanation for Correct Answer C : Choice (C) is correct. It avoids the error of the original by using the appropriate conjunction "and" to link a clear cause (debris being sucked into a jet engine) with its expected effect (a serious accident).

nR
(A)

Explanation for Incorrect Answer E : Choice (E) violates parallelism. The verbal "avoiding" is neither grammatically nor logically parallel with the earlier verb phrase "could be sucked," since this earlier phrase describes a likely cause of an accident, not a method of preventing one.

In believing that firsthand experience would enhance the credibility of his biography of Columbus, Professor Morison retraced the route of Columbus first voyage.

In believing that

(B)

Believing that

(C) In his belief that

(D) He believed that

(E)

By believing that

ANSWERS

Explanation for Correct Answer B : Choice (B) is correct. It avoids the error of the original by eliminating the unnecessary preposition "in."

Explanation for Incorrect Answer A : Choice (A) exhibits wordiness. The preposition "in" is not needed.

Explanation for Incorrect Answer C : Choice (C) displays wordiness. Four words ("In his belief that") can be reduced to two ("Believing that").

eg is

Explanation for Incorrect Answer D : Choice (D) exhibits improper modification. The dangling phrase, "preventing a serious accident," apparently indicates a reason for sweeping runways, but it is not in a position to modify correctly any part of the sentence.

AND EXPLANATIONS

te

Explanation for Incorrect Answer B : Choice (B) displays incorrect coordination. A comma is used improperly to join two complete thoughts ("Airport runways . . . into a jet-engine intake" and "this causes a serious accident").

re

Explanation for Incorrect Answer A : Choice (A) contains the erroneous singular pronoun "it," which appears to refer to the plural antecedents "trash and other debris."

file://E:\\c10.htm

2006-11-12

The Official SAT Online Course

5/9

Explanation for Incorrect Answer D : Choice (D) involves improper coordination. Two complete thoughts ("He believed . . . of Columbus" and "Professor Morrison retraced . . . first voyage") are joined with only a comma.

Explanation for Incorrect Answer E : Choice (E) uses an inappropriate idiom. The preposition "by" incorrectly suggests a means by which Morrison retraced the route, rather than the mental state in which he did so.

Except in mathematics, absolute proof is more often an ideal to be sought than a goal to be reached, a fact that the courts recognize by setting varying standards of proof for different kinds of cases.
(A)

a fact that the courts recognize by setting

(B)

which the courts recognize and set

(C) and this is recognized when the courts are setting

(D) and it is recognized by the courts when they set

(E)

and the courts recognize this fact setting

ANSWERS

AND EXPLANATIONS

nR
(A)

Explanation for Incorrect Answer C : Choice (C) contains a vague pronoun. The meaning of the pronoun "this" remains unclear, since it apparently refers to the general idea expressed in the preceding clause, not to any specific noun in the sentence.

Explanation for Incorrect Answer D : Choice (D) exhibits vague and ambiguous pronoun use. Since the pronoun "it" cannot logically refer to any of the singular nouns in the sentence ("proof," "ideal," or "goal"), its meaning (apparently the general idea expressed in the preceding clause) is not clear.

Explanation for Incorrect Answer E : Choice (E) displays ineffective coordination. Using the conjunction "and" to connect two independent clauses is grammatically correct, but such a linkage does not indicate the logical relationship between the two clauses.

My grandfather never learned to use a calculator, as he shops he can accurately compute his grocery bill in his head to within a dollar.

as he shops

(B)

while shopping

(C) but as he shops

(D) therefore, when shopping

(E)

however, he shops so that

ANSWERS

Explanation for Correct Answer C : Choice (C) is correct. It avoids the error of the original by adding an appropriate

eg

Explanation for Incorrect Answer B : Choice (B) uses a vague pronoun. Since the pronoun "which" refers loosely to the idea expressed in the entire preceding clause (but not to any specific noun), the meaning is not clear.

AND EXPLANATIONS

is te

re

Explanation for Correct Answer A : Choice (A) is correct. It uses a specific noun, "fact," instead of a vague and ambiguous pronoun.

file://E:\\c10.htm

2006-11-12

The Official SAT Online Course

6/9

conjunction, "but," to join the two independent clauses that express contrasting ideas.

Explanation for Incorrect Answer A : Choice (A) displays improper coordination. It uses only a comma to join two complete thoughts ("My grandfather . . . calculator" and "as he shops . . . within a dollar").

Explanation for Incorrect Answer B : Choice (B) uses improper coordination. Two complete thoughts ("My grandfather . . . calculator" and "while shopping . . . within a dollar") are joined by only a comma.

Explanation for Incorrect Answer D : Choice (D) exhibits improper coordination. Since the transition word "therefore" does not serve as a coordinating conjunction, this choice incorrectly links two complete thoughts ("My grandfather . . . calculator" and "therefore, when shopping . . . within a dollar") with only a comma.

Explanation for Incorrect Answer E : Choice (E) results in improper coordination. Two independent clauses ("My grandfather . . . calculator" and "however, he shops . . . within a dollar") are connected by only a comma.

10

The first African American woman to win the Pulitzer Prize for poetry, Gwendolyn Brooks with her special interest in encouraging young poets.

(B)

had a special interest in encouraging

(C) having had a special interest, which was to encourage

(E)

she had a special interest to encourage

11

nR eg
ANSWERS
(A)

AND EXPLANATIONS

Explanation for Correct Answer B : Choice (B) is correct. It avoids the error of the original by providing a verb, "had," so that the sentence states a complete idea.

Explanation for Incorrect Answer A : Choice (A) produces a sentence fragment. With no verb (only the verbal phrases "to win" and "encouraging"), the main clause fails to state a complete idea.

Explanation for Incorrect Answer C : Choice (C) creates a sentence fragment. Although this choice has a verb, "was," the verb is part of a dependent clause introduced by "which." The main clause has no verb to complete the thought.

Explanation for Incorrect Answer D : Choice (D) is a sentence fragment. The only verb, "had," is part of a dependent clause ("who had a special interest in encouraging young poets"), so the main clause cannot state a complete thought.

Explanation for Incorrect Answer E : Choice (E) includes an excess word. The pronoun "she" simply repeats the name immediately before it, "Gwendolyn Brooks," and is, therefore, unnecessary.

Although fascinated by chance and coincidence, Paul Austers novels are written with careful attention to style and balance.

Paul Austers novels are written

is

(D) who had a special interest in encouraging

te r

(A)

with her special interest in encouraging

ed

file://E:\\c10.htm

2006-11-12

The Official SAT Online Course

7/9

(B)

Paul Austers novels were written

(C) Paul Auster writes his novels

(D) Paul Auster is a writer

(E)

Paul Auster had wrote

ANSWERS

AND EXPLANATIONS

Explanation for Correct Answer C : Choice (C) is correct. It avoids the error of the original by placing the proper noun "Paul Auster" immediately after the introductory phrase that modifies it.

Explanation for Incorrect Answer A : Choice (A) displays improper modification. The introductory phrase, "Although fascinated by chance and coincidence," cannot logically modify the noun that follows, "novels," since it clearly describes the author rather than his books.

Explanation for Incorrect Answer B : Choice (B) exhibits improper modification. The introductory phrase, "Although fascinated by chance and coincidence," logically modifies the author Paul Auster, not the novels that he wrote.

12

Early American factories did not so much replace household manufacturing but complement it.

nR
(B)

(A)

but complement

as complement

(C) but they complemented

(D) and they complemented

(E)

as they were to complement

ANSWERS

Explanation for Correct Answer B : Choice (B) is correct. It avoids the error of the original by using the appropriate conjunction "as" to complete the idea of the introductory phrase "so much" and thus properly link two parallel verbs ("replace" and "complement").

Explanation for Incorrect Answer A : Choice (A) uses an inappropriate idiom. After the phrase "so much," the conjunction "but" is improper because it suggests not a limited degree of difference but a more complete contrast.

Explanation for Incorrect Answer C : Choice (C) contains an idiom that is not suitable. The conjunction "but" is not appropriate for use with the phrase "so much" because "but" implies a more absolute contrast.

Explanation for Incorrect Answer D : Choice (D) employs an improper idiom. The conjunction "and" suggests no contrast but simply indicates that additional information will follow.

eg

AND EXPLANATIONS

is te

Explanation for Incorrect Answer E : Choice (E) uses an incorrect verb form. After the helping verb "had," the correct form of this irregular verb is the past participle, "written," not the past, "wrote."

re

Explanation for Incorrect Answer D : Choice (D) involves inappropriate modification. The prepositional phrase, "with careful attention to style and balance," functions as an adverb telling how the novels were written, not as an adjective describing the preceding noun, "writer."

file://E:\\c10.htm

2006-11-12

The Official SAT Online Course

8/9

Explanation for Incorrect Answer E : Choice (E) displays wordiness. The phrase "they were to" is unnecessary.

13

After teaching, becoming involved in several fashion enterprises, and after she founded the Harlem Institute of Fashion, Lois Alexander Lane launched the Black Fashion Museum.
(A)

after she founded

(B)

after the founding of

(C) founding

(D) she had founded

(E)

having founded

ANSWERS

AND EXPLANATIONS

Explanation for Correct Answer C : Choice (C) is correct. It avoids the error of the original by using a gerund, "founding," that is parallel with the earlier gerunds, "teaching" and "becoming."

Explanation for Incorrect Answer A : Choice (A) fails to maintain parallelism. The clause "after she founded" is not parallel with two earlier items in the seriesthe gerunds "teaching" and "becoming."

Explanation for Incorrect Answer B : Choice (B) uses excess words. The words "after," "the," and "of" are not needed.

14

nR
(A)

Explanation for Incorrect Answer E : Choice (E) has an error in tense sequence. The present perfect tense of the verbal "having founded" is not consistent with the present tense of the earlier verbals "teaching" and "becoming."

In eighteenth-century France, economic inequalities made many people angry, and a violent revolution was fueled.

angry, and a violent revolution was fueled

(B)

angry; it fueled a violent revolution

(C) angry, and this anger fueled a violent revolution

(D) angry, that anger fueled a violent revolution

(E)

angry; thus fueling a violent revolution

ANSWERS

Explanation for Correct Answer C : Choice (C) is correct. It avoids the error of the original by using two parallel clauses ("economic inequalities made" and "this anger fueled"), both in active voice.

Explanation for Incorrect Answer A : Choice (A) fails to maintain parallelism. The first independent clause uses active voice ("economic inequalities made many people angry"), but the second shifts to ineffective passive voice ("and a violent revolution was fueled").

Explanation for Incorrect Answer B : Choice (B) contains a vague pronoun. The singular pronoun "it" cannot logically

eg i

Explanation for Incorrect Answer D : Choice (D) violates parallelism. The clause "she had founded" is not parallel with the gerunds "teaching" and "becoming"the two earlier items in the series.

AND EXPLANATIONS

st er

ed

file://E:\\c10.htm

2006-11-12

The Official SAT Online Course


refer to the noun "France" (the only singular noun that precedes it), and it does not agree with the plural nouns "inequalities" and "people."

9/9

Explanation for Incorrect Answer D : Choice (D) uses improper coordination. It uses only a comma to join two complete thoughts ("In eighteenth-century France . . . many people angry" and "that anger fueled a violent revolution").

Explanation for Incorrect Answer E : Choice (E) exhibits inappropriate coordination. It incorrectly uses a semicolon to link parts of unequal grammatical rank (a complete thought before the semicolon and only a phrase after it).

Back to Score Report

Copyright 2006 The College Board. All rights reserved.

Privacy Policy

Terms of Use

Contact Us

nR

eg

is

te

re
file://E:\\c10.htm

d
2006-11-12

You might also like